Vous êtes sur la page 1sur 116

CONTENTS TESTS AND CLINICAL CASES FOR 5th YEAR STUDENS DISEASES WITH PRIMARY AIRDROPLET WAY OF TRANSMISSION

Influenza. ARVI (1-30) Acute tonsillitis. Diphtheria (31-62) Infectious mononucleosis (63-69)

Lime-borreliosis (321-329) Louse-born typhus. Brills disease (330-342) Malaria (343-361) Helminthes (362-368)

INFECTIOUS DISEASES WITH INOCULATION AND PARENTERAL MECHANISM OF TRANSMISSION HIV-infection and AIDS (369-378)

Meningococcal infection (70-102) Rabies (379-392) Legionellosis (103-131) Tetanus (393-405) Measles (132-145) Erysipelas (406-412) DISEASES WITH PRIMARY ALIMENTARY WAY OF TRANSMISSION Typhoid fever. Paratyphoid A and B (146-163) Salmonellosis (164-172) Haemorrhagic fevers (445-459) Food poisonings (173-185) Haemorragic fever with renal syndrome (460-474) Botulism (186-192) Herpesvirus infection (475-500) Pseudotuberculosis (193-204) Plaque. Tularemia (501-516) Intestinal yersiniosis (205-212) Antrax (517-526) Echerichiosis (213-216) Brucellosis (527-536) Shigellosis (217-232) Q-fever (537-541) Cholera (233-243) Toxoplasmosis (542-549) Rotaviral diarrheas (244-249) Sepsis (550-559) Viral hepatitis (250-271) Enteroviral diseases (272-273) Poliomyelitis (274-283) Protozoal diseases. Amoebiasis (284-293) Balantidiasis. Lambliosis (294-300) TESTS AND CLINICAL CASES FOR 6th YEAR STUDENS Clinical cases Clinical tasks (1-58) Skin changes in some infectious diseases (59-64) Laboratory method of diagnostics (65-108) DISEASES WITH TRANSMISSIVE MECHANISM OF TRANSMITTION Arboviral encephalits (301-320) Leptosprosis (413-444)

DISEASES WITH MULTIPLE WAYS OF TRANSMITTION

Urgent states in infectious pathology (109-144)

DISEASES WITH PRIMARY AIR-DROPLET WAY OF TRANSMISSION INFLUENZA. ARVI 1. What is etiological agent of influenza? a) virus; b) chlamydia; c) bacteria; d) mycoplasmas; e) protozoa. 2. What is the most typical localization of pathological process of influenza? a) nasopharynx; b) larynx; c) trachea; d) bronchi; e) lung. 3. What type of inflammation of the respiratory tract is typical for the uncomplicated influenza? a) catarrhal; b) fibrinogenous; c) purulent; d) fibrinohemorrhagic. 4. Choose the most effective medicine for etiological treatment of influenza: a) Interferon; b) Aciclovire; c) Remantadine; d) Neovire; e) Zovirax; 5. The patient was admitted to the hospital on the 2nd day of disease with complaints on general weakness, T 37.1C, headache, difficulties with nasal breathing, discharging of abundant serous secretion from the nose. O/E: maceration of nostrils skin, moderate hyperemia of the back wall of the throat. What is the most likely diagnosis in this patient? a) influenza; b) parainfluenza; c) adenoviral infection; d) RS-viral infection; e) rhinoviral infection. 6. Choose the drug for treatment of mycoplasma infection: a) Ribavirin; b) Eritromicyn; c) Penicillin; d) Riboxin; e) Oxolin oilment. 7. Patient was admitted on the 3-rd day after acute beginning of the disease. The patient complained on chills, headache, rhinitis and increasing of temperature from 37,80 up to 38.2C, sore throat, feeling and irritation of eyes. O/E: dropsical face, conjunctiva is edematous and hyperemic. The palatine tonsils, back wall of the throat are edematous, hyperplasia of

lymphatic follicles of back wall of throat is seen, and submandibular lymph nodes are enlarged. What is the most likely diagnosis? a) influenza; b) parainfluenza; c) adenoviral infection; d) RS-viral infection; e) rhinoviral infection 8. Patient was admitted on the 3rd day of the disease. His temperature was 37.3-37.5C. The patient complained on weakness, mild rhinitis, sore throat, dry cough. O/E: hyperemia of the mucous membrane of back wall of the throat, hoarse voice, normal lung examination. What is the most likely diagnosis? a) influenza; b) parainfluenza; c) adenoviral infection; d) RS-viral infection; e) rhinoviral infection 9. What is the most typical manifestation of the RSinfection? a) tracheatis; b) tonsillitis; c) bronchiolitis; d) laryngitis; e) conjunctivitis. 10. Mail of 27 year-old was admitted to the hospital during the epidemic of influenza. Onset of disease was acute, temperature was high up to 39.0-39.7C. Chills, pain in muscles and joints, headache and irritation of eyes were present. Treatment with aspirin, paracetamol was not effective. On the 2nd day of disease temperature was 39.8C, loss of orientation, cramps, psychomotor excitement were noted. O/E: meningeal symptoms are negative, hyperemia of the skin and mucous membrane were present. What complication of disease took place in this case? a) infectious-toxic shock; b) edema of the brain; c) intracranial bleeding; d) meningitis; e) encephalitis. 11. What are the 2 most frequent manifestation of the hypertoxic form of the influenza? a) meningitis; b) edema of the brain; c) myocarditis; d) edema of the lungs; e) croup; f) bronchoobstructive syndrome. 12. What are the methods of laboratory diagnostics of the influenza and ARVI? a) immunofluorescent test (IFT); b) complement fixation test (CFT) ; c) culturing on embrioneted eggs; d) indirect hemagglutination test; e) all mentioned.. 13. The influenza virus has tendency to affect (choose correct answer): a) cells of the ciliated epithelium and lymph tissue;

cells of the ciliated epithelium; epithelium of the gut; epithelium of the conjunctive; ciliated epithelium and epithelium of conjunctive. 14. Adenoviruses have tendency to affect (choose correct answer): a) ciliated epithelium of respiratory tract; b) epithelium of the gut; c) epithelium of the conjunctive; d) lymph tissue; e) all mentioned. 15. What are the 2 most frequent complications of rhinoviral infection? a) myocarditis; b) sinusitis; c) bronchitis; d) pneumonia; e) otitis; 16. What is the most frequent cause of the acute respiratory insufficiency as a complication of severe form of the influenza? a) croup; b) bronchocbstructive syndrome; c) edema of the lung; d) myocarditis; e) fit of spastic cough with apnea; 17. Which 2 drugs may be recommended for treatment of the lung edema? a) Norfloxacin; b) Mannitol; c) Dexametazone; d) Heparin; e) Aminasin 18. Which 2 drugs are not recommended for treatment of the brain edema? a) dexametasone; b) strichnin; c) Na oxibutirat; d) lasics; e) promedole. 19. Susceptibility of humans to influenza is: a) high in infants; b) *high in all age groups; c) high in immunocompromised persons; d) high in aged persons; e) high in young persons. 20. Acquired immunity after influenza is: a) Weak, short-living, that is the cause of repeated epidemics; b) Strong for life, repeated diseases impossible; c) Strong, but strain-specific, repeated epidemics are connected with genetic drift of the virus; d) Repeated epidemics are connected with short-living immunity response and genetic drift of the virus. 21. What is the most effective antiinfluenza medication from mentioned below? a) -Interferon; b) Remantadine; c) Oxolin oilment 0.25%

b) c) d) e)

d) Aminocapronic acid 5%; e) Aspirin. 22. What is the most accepted scheme for treatment of the influenza with remantadine? a) 1st day 300 mg, 2nd 200 mg, from 3rd 150 mg/day; b) 100 mg 3 t. d. for whole period of the disease; c) 50 mg 4 t.d. for whole period of the disease; d) *50 mg 3 t.d. for whole period of the disease; e) Single dose of 300 mg daily. 23. the main factor of the development of the lung edema in patients with influenza is: a) increased pressure in the pulmonary system; b) disturbance of K+ and Na+-pump of alveolar epithelium and transfer of liquid to the alveolus gap; c) severe disturbances of microcirculation and disseminated intravascular coagulation, mainly in the lung vessels; d) changing of the oncotic properties of blood; e) all mentioned. 24. High frequency of such complication of influenza as pneumonia is conditioned by all mentioned below factors except of: a) disturbance of the ciliated epithelium clearance; b) hyperproduction of the mucus in bronchi and increasing of its viscosity; c) viral affection of alveolar epithelium; d) T-cell immunity decreasing; e) *All mentioned.. 25. What white cell count is characteristic for influenza? a) leucopoenia, lymphocytosis, decreasing of ESR; b) leucopoenia, lymphocytosis, increasing of ESR; c) leukocytosis, neutrophylosis, high ESR; d) leucocytosis, insignificant lymphocytosis, increasing of ESR; e) expressed leucopoenia (L 2.0*109 /l), neutropoenia, lymphocytosis, high ESR. 26. Using of antiviral drugs for treatment of influenza is: a) effective during whole period of the disease; b) ineffective; c) effective only when it is used in the first 3 days of the disease; d) most effective when it is used after the 4 day of the disease; e) effective after the 7 day of the disease. 27. What complication of influenza is not connected with secondary bacterial infection? a) pneumonia; b) diencephalitis; c) otitis; d) bronchitis; e) sinusitis. 28. What complications of influenza are not connected with specific viral affection? a) diencephalitis; b) neuritis of the facial cranial nerve; c) myocarditis; d) sinusitis;

e) ganglionitis. 29. The general mechanisms of transmission of adenoviral infection are all following beside of: a) through the bites of insects; b) fecal-oral; c) air-droplet; d) contact. 30. What peculiarities of blood cell count are typical for climax period of influenza? a) Leucopoenia, neutropoenia, eosinopoenia, relative lymphocytosis; b) Normal count or insignificant leucocytosis, tendency to neutrophyls increase, drum - stick shift to the left, monocytosis, finding of plasmatic Turks cells; c) Leucopoenia, aneosinophylia, lymphocytosis, thrombocytopoenia, ESR elevation; d) Leucocytosis, neutropoenia, lympho monocytosis, atypical mononuclear cells.

ACUTE TONSILLITIS AND DIPHTHERIA. 31. To what genus belongs pathogen of diphtheria? a) Neisseria; b) Corynebacteria; c) Campilobacteria; d) Streptococci; e) Staphylococci. 32. What are the properties of Cl. Diphtheria? a) spore-forming and Gram-negative bacteria; b) Gram-negative and nonspore-forming bacteria; c) nonspore-forming and Gram-positive bacteria; d) Gram-positive and spore-forming bacteria; 33. What is the stability of Cl. Diphteria in environment? a) unstable; b) poorly stable; c) considerably stable; 34. What is a main source of infection with diphtheria? a) healthy bacteria carriers; b) patients with atypical forms of disease; c) patients with typical forms of disease; d) patients with severe forms of disease. 35. What are the two main way of transmission of diphtheria? a) *contact; b) *air-droplet; c) transplacenter; d) alimentary; e) through the bites of insects. 36. What is the type of immunity response acquired after diphtheria? a) active antibacterial; b) active antitoxic; c) passive antibacterial; d) passive antitoxic; e) all mentioned.

37. What is the main factor of the pathogenesis of diphtheria? a) exotoxinemia; b) endotoxinemia; c) bacteriemia; 38. What is the specimen for bacteriological investigation in diphtheria? a) blood; b) sputum; c) nasopharyngeal secretion; d) membrane from the tonsils; e) all mentioned. 39. What location of pathological process is most typical for the modern diphtheria? a) nose; b) oropharyngeal region; c) larynx; d) trachea; e) skin. 40. What kind of inflammation is characteristic for oropharyngeal form of diphtheria? a) fibrinous; b) purulent; c) serous; d) productive; e) all mentioned. 41. What is a cause of enlargement of palatine tonsils in patients with oropharyngeal form of diphtheria? a) hyperplasia of lymph tissue; b) edema of tonsils; c) productive inflammation; d) hemorrhages; e) all mentioned. 42. What is the principal cause of death on early stages of diphtheria (2-3 days)? a) myocarditis; b) polyneuritis; c) croup; d) infectious - toxic shock. e) hemorrhagic shock. 43. Which one of the following affections of nervous system is characteristic for the late period of diphtheria (2-nd week and later)? a) meningitis; b) polyneuritis; c) encephalitis; d) mental disorders; e) ganglionites. 44. Which one of the following affections of kidneys is characteristic for severe cases of diphtheria? a) pyelonephritis; b) glomerulonephritis; c) toxic nephroso-nephritis; d) hydronephrosis; e) all mentioned. 45. What disease should be first differentiated with membranous form of oropharyngeal diphtheria? a) lacunar form of tonsillitis;

b) chemic and burns of pharynx; c) Simanovsky - Vincent`s tonsillitis; d) mycotic affections of mucous of pharynx; e) herpetic stomatitis 46. Subtoxic (moderate) form of oropharygeal diphtheria is most similar to: a) epidemic parotitis; b) purulent parotitis; c) peritonsillitis or paratonsillar abscess); d) purulent submandibular lymphadenitis; e) tuberculosis of cervical lymph nodes. 47. What is the basic method of diagnostics of the diphtheria? a) epidemiological; b) clinical; c) bacteriological; d) serological; e) all mentioned. 48. What is the basic method of therapy of diphtheria? a) systemic treatment with antibiotics; b) local treatment with antiseptics; c) detoxication specific; d) detoxication nonspecific; e) antiviral drugs. 49. What is the dose of diphtheria antitoxin for treatment of localized oropharyngeal form of diphtheria? a) 20 000 IU; b) 40 000 IU; c) 60 000 IU; d) 80 000 IU; e) 100 000 IU. 50. What is the initial dose of horse hyperimmune antidiphtherial serum for treatment of severe oropharyngeal form of diphtheria at first 2 days of disease? a) 40 000 IU ; b) 60 000 IU; c) 80 000 IU; d) 100 000 IU; e) 150 000 IU. 51. Who is the main reservoir of diphtheria? a) ill persons and carriers of bacteria; b) cattle; c) rodents; d) birds; e) insects. 52. Which type of inflammation is most typical for oropharyngeal form of diphtheria? a) Serous-gemorragic; b) fibrinous; c) diphtheric; d) productive; e) purulent. 53. Which type of inflammation is most typical for the diphtheria of larynx and trachea? a) serous-gemorragic; b) fibrinous;

c) diphtheric; d) productive; e) purulent; 54. What are the first therapeutic measures in case of diphtheria: a) inhalations of O2; b) diphtheria antitoxin; c) penicillin; d) removing of membranes; e) glucocorticoids. 55. What is one of the most important clinical indexes of gravity (level of toxicosis): a) spreading of a patches; b) dyspnea; c) enlargement of regional lymph nodes; d) edema of the subcutaneous layer of neck; e) tachycardia. 56. What is the most frequent complication of the combined form of diphtheria (diphtheria of oropharyngeal region and of larynges? a) toxic shook; b) acute renal insufficiency; c) croup; d) bronchoobstructive syndrome; e) acute cardio-vascular insufficiency. 57. What is the therapeutic measure for treatment of croup in patients with diphtheria? a) Tracheostomy; b) glucocorticoids; c) inhalation of oxygen; d) bemegrid; e) euphillin. 58. Which antibiotic is the drug of choice for treatment of diphtheria? a) penicillin; b) erythromycin; c) ciprofloxacin; d) amfotericin B. e) all mentioned. 59. What is the mechanism of the development of polyneuritis complicated diphtheria? a) reversible dystrophic changes of the cranial nerves nuclei and the anterior roots of spinal cord; b) severe irreversible neuropathy; c) demyelinisation of sheaths of nerves; d) disturbance of acetylcholine synthesis in synapses. e) No one from mentioned. 60. What is the dose of diphtheria antitoxin for treatment of cutaneous form of diphtheria?: a) *10 20000 ME; b) 40 60000 ME; c) 80 10000 ME; d) 80 10000 ME; e) 100 150000 ME; 61. What are the medications for treatment of diphtheric polyneuritis? a) glucocorticoids; b) potassium orotate;

c) nootrops; d) proserin; e) massage. 62. What drug are not recommended for treatment of diphtheric myocarditis? a) strophantin; b) glucocorticoids; c) rhiboxin; d) strichnin; e) tocopherol. E a s i e r s a i d t h a n d o n e .

INFECTIOUS MONONUCLEOSIS 63. What is the causative agent of the mononucleosis? a) Hepadnavirus; b) Klebsiella pneumoniae; c) Epstine Barr virus; d) Myxovirus; e) Paramyxovirus. 64. Which diseases from mentioned below are connected with Epstine-Barr viral infection (2)? a) Caposi carcinoma; b) Burkitts lymphoma; c) acute lympholeucosis; d) nasopharingeal carcinoma; e) carcinomatosis of the ovaries. 65. What are the two from mentioned below methods, which are effective for diagnostic of mononucleosis? a) blood cell count; b) PHAT of patients sera with specific antigen; c) detection of IgM antibodies to the capsular antigen of EBV; d) detection of IgG antibodies to the capsular antigen of EBV; e) detection of IgA antibodies to the early antigen of EBV; 66. Treatment of mild and moderate cases of the mononucleosis include all drugs except of: a) penicillin; b) vitamin C; c) gargling of the throat with antiseptics; d) dimedrol; e) glucocorticoids. 67. What are the main clinical signs of the mononucleosis? a) prolonged fever, diarrhea, abdominal pain; b) prolonged fever, generalized lymphoadenopathy, liver and spleen enlargement; c) prolonged fever, meningeal syndrome, liver and spleen enlargement; d) prolonged fever, generalized lymphoadenopathy, diarrhea. e) generalized lymphoadenopathy, petechial rush, kidney affection. 68. What are the typical for the mononucleosis changing in the blood cell count? a) neutrophylic leucocytosis, eosinophylia, anemia; b) leucocytosis, lymphocytosis, atypical mononuclears; c) neutrophylic leucocytosis, atypical mononuclears, eosinophylia; d) neutrophylic leucocytosis, drum stick shift to left, eosinophylia; e) all mentioned. 69. Chose symptoms, typical for icteric form of infectious mononucleosis: a) fever, lymphoadenopathy, oliguria, significant serum ALAT elevation;

b) lymphoadenopathy and fever are absent, function of kidneys is normal, hepatomegaly, significant serum ALAT elevation; c) fever, generalized lymphoadenopathy, function of kidneys is normal, liver and spleen are enlarged, moderate serum ALAT elevation; d) generalized lymphoadenopathy; fever is absent; the function of kidneys is normal, significant serum ALAT elevation. Promise little, but do much.

MENINGOCOCCAL INFECTION 70. What is the main source of meningococcal infection? a) patients with generalized forms of disease; b) patients with a meningococcal nasopharyngitis; c) carriers of meningococci; d) convalescents of meningococcal meningitis 71. What is the site of entrance for the meningococcal infection? a) nasopharynx; b) conjunctiva; c) damaged skin; d) trachea; e) gastrointestional tract. 72. What kind of exanthema is typical for meningococcemia? a) maculopapular; b) papular; c) erythematic; d) hemorrhagic; e) All mentioned. 73.On which day of disease the rush appears in case of meningococcemia? a) 1 2; b) 3 4; c) 5 6; d) 7 8; e) 10 - 12. 74. For which form of meningococcal infection the exanthema is the most typical sign? a) meningococcal nasopharyngitis; b) meningococcal meningitis; c) meningococcal meningoencephalitis; d) meningococcemia; e) all mentioned. 75. Which data are the most informative for diagnostics of meningitis? a) meningeal syndrome; b) symptoms of the general intoxication; c) inflammatory changes in spinal fluid; d) epidemiological data; e) Brudzinskys sign. 76. Which method is usually used for early specific diagnostics of meningococcal infection? a) biological; b) bacteriological; c) serological; d) allergological; e) all mentioned. 77. What is the main way of spreading of meningococci in human organism? a) hematogenous; b) lymphogenous; c) per contact; d) per neural; e) all mentioned. 78. What is the most typical causative agent of primary purulent meningitis?

a) Ps. aeruginosa; b) N. meningitidis; c) S. pneumoniael; d) Candida albicans; e) H. influenzae 79. What etiology is most typical for the primary seasonal meningitis? a) arboviral; b) pneumococcal; c) tubercular; d) brucellous; 80. Which specimens from patients may contain N. meningitides? a) blood; b) sputum; c) nasal secretion; d) spinal fluid; e) all mentioned. 81. Which characteristics of spinal fluid are typical for the purulent meningitis? a) colorless, transparent, 10 cells106/l, protein 0.12 g/l, all cells - lymphocytes; b) white with sediment, muddy; 10000 cells106/l, protein 2.3 g/l, 80% of cells neutrophyls; c) transparent, rust-colored; 15 cells106/l, protein 1.2 g/l, lymphocytes and erythrocytes. d) colorless, twinkling; 110 cells106/l, protein 3.5 g/l, all cells - lymphocytes; e) colorless, transparent, 50 cells106/l, protein 2.5 g/l, all cells - lymphocytes; 82. What characteristics of spinal liquor typical for the serous meningitis? a) colorless, transparent; 10 cells106/l, protein 0.12 g/l, all cells - lymphocytes; b) white with sediment; muddy, 10000 cells106/l, protein 2.3 g/l, 80% of cells neutrophils; c) transparent, rust-colored; 15 cells106/l, protein 1.2 g/l, lymphocytes and erythrocytes. d) colorless, twinkling; 110 cells106/l, protein 3.5 g/l, all cells - lymphocytes; e) colorless, transparent, 50 cells106/l, protein 2.5 g/l, all cells - lymphocytes; 83. What is the most effective antibiotic for treatment of the meningococci carriers? a) Ampicillin; b) Gentamycin; c) Biseptole; d) Rifampycin; e) Tetracycline. 84. Which two antibiotics are the drugs of choice for treatment of generalized forms of meningococcal infection? a) Doxyciclin; b) Penicillin G; c) Streptomycin; d) Ryphampicin; e) Laevomycetin.

85. What is the most serious complication of the early period of meningitis: a) toxic shock; b) edema of brain; c) ependimatitis; d) myocarditis; e) encephalitis. 86. What is the most serious complication of the meningococcemia? a) toxic shock; b) Waterhouse-Fridericksens syndrome; c) edema of the brain; d) edema of the lungs; e) endocarditis. 89. Despite antibacterial and pathogenic therapy of the patient with meningococcemia sudden reducing of the hemodynamic indexes :(BP 40/0 mm Hg, PS 140 beats/min) registered. Intensive therapy with high doses of dopamine and glucocorticoids was ineffective. What the most likely complications does develop in this case? a) toxic shock; b) acute cardiovascular insufficiency; c) collapse; d) acute adrenal failure; e) edema of the brain. 90. What are the causes of the development of ependimatitis in patients with purulent meningitis? a) late hospitalization; b) ineffective antibacterial therapy; c) it is obligatory stage of the pathological process; d) immunodeficiency; e) all mentioned. 91. Indicate the most essential symptoms, characteristic for meningoencephalilis a) presence of local neurological symptoms; b) comatose state; c) psycho-motor excitement; d) development of convulsive syndrome; e) all mentioned. 92. Which symptoms are not typical for the syndrome of wedging the medulla oblongata into the foramen magnum in patients with comatose state? a) arterial hypotension; b) midriasis, absence of the pupil reflex; c) bradypnoe; d) arterial hypertension; e) muscular atonia. 93. Which antibiotic is the drug of choice for initial treatment of severe form of meningococcemia? a) Penicillin; b) Cephtriaxon; c) Laevomycetin; d) Riphampicin; e) Vibramicin. 94. What is the optimal dose of penicillin for the treatment of meningococcal meningitis in adults? a) 250 500.000 units/kg/day; b) 50 100.000 units/kg/day;

c) 1 000 000 units /kg/day; d) 600.000 units/kg/day; e) 700.000 units/kg/day. 95. What is the optimal dose of laevomycetin for the treatment of meningococcal meningitis in adults? a) 50 60 mg/kg/day; b) 10 30 mg/kg/day; c) 30 50 mg/kg/day; d) 60 80 mg/kg/day; e) 80 100 mg/kg/day. 96. What is the most frequent form of meningococcal infection: a) nazopharingitis; b) meningitis; c) meningococcemia; d) pneumonia; e) germ-carrierity. 97. Which serogroups of N. meningitidis are the most frequent causes of generalized forms of diseases? a) A, B, C, Y; b) A,X,Y, 29C; c) B, C, X, Z; d) A, B, C, W-135; e) A, B, C, Z. 98. Whish cultural properties are not typical for N. meningitidis (2)? a) Gram cocci; b) Gram+ cocci; c) beanshaped appearance, frequently as a dyplococci; d) oval, frequently chainsequenced; e) aerobes; 99. Which factors are not characteristic for N. meningitidis (2): a) presence of O-antigen; b) presence of spore; c) presence of pili; d) presence of lipopolysaccharide; e) presence of exotoxin. 100. What methods are used for express diagnostics of meningococcal infection (2)? a) latexagglutination; b) bacterioscopy of thick drop of blood; c) culture of blood; d) serological test; e) skin allergic test. 101. Which symptoms are not typical for meningeal syndrome? a) severe headache; b) rigidity of neck; c) vomiting; d) affection of cranial nerves; e) diarrhea. 102. Which symptoms are not typical for meningeal syndrome? a) rigidity of neck; b) Brudzinskys symptom; c) Bechterevs symptom; d) Kernigs symptom; e) Lessages symptom.

Deeds not words.

10

LEGIONELLOSIS 103. Legionellosis is not connected with: a) L. pneumophilia; b) L. micdadei; c) L. longbeache; d) L. bozemanii; e) L. icterohaemorrhagica. 104. What media are used for cultivation of legionella? a) Agaragar; b) Leffler agar; c) Yeastcharcoal agar; d) Ascite agar; e) KittTarocci agar. 105. There are the main antigens of legionella, which produce antibodies in human organism: a) superficial antigen; b) lipopolysacharide; c) nucleic acid; d) Oantigen; e) Capsule antigen. 106. What are the main localizations of legionella in enviorment (2)? a) intestines of animals; b) water reservoirs; c) soil; d) human nasopharinx; e) human intestines. 107. What factors are not facilitate colonization of legionella? a) t 25 - 42C; b) t 10 - 20C; c) presence of amoeba, green algae; d) presence of sediment in water systems; e) chlorination of water. 108. What ways of transmission are not typical for legionellosis (2)? a) air-droplet; b) drinking of unboiled water; c) alimentary; d) during excavations; e) direct human to human contact; 109. What are the factors of risk for legionellosis (2)? a) immunosupression; b) allergic diseases; c) chronic diseases of stomach; d) young age; e) smoking. 110. What are the factors of risk for legionellosis (2)? a) local anesthesia; b) endotracheal narcosis; c) allergic diseases; d) conditioners of air; e) hemophilia. 111. Typical clinical forms of legionellosis are the following (2): a) pneumonia; b) systemic form; c) acute infection of upper respiratory tract; d) gastro-intestinal form;

e) meningeal form. 112. Factors, contributing to development of legionellosis are the following (2): a) depressing of humoral mechanisms of immune system; b) depressing of cellular part of immune system; c) decreasing of fagocyts activeness and neutropoenia; d) persistence of pathogen. 113. Which express methods of laboratory diagnostics of legionellosis is most effective? a) gram staining; b) hematoxilin-eosin staining; c) direct immunofluorescent test; d) culturing; e) detection of antibodies in urine. 114. What clinical sighs are not typical for legionellosis (2)? a) high temperature; b) diarrhea; c) ineffective treatment with cephalosporins; d) ineffective treatment with macrolids; e) development of the disease 10 days after discharge from hospital. 115. What clinical sighs are not typical for legionellosis? a) Massive infiltration in lungs; b) Hyponatriumemia (less than 130 mmol/l); c) ineffective tratment with penicillin; d) development of the disease 10 days after stay in intensive care unit; e) ineffective treatment with azytromicin. 116. What clinical symptom is not typical for Pontiac fever? a) high temperature; b) diarrhea; c) catarrhal symptoms d) meningeal signs; e) artralgia. 117. Incubation period of legionellosis numrers: a) 24-48 h. b) 2-5 days; c) 2-10 days; d) 2-14 days; e) about 1 month. 118. For extrapulmonary legionellosis is typical development of: a) myocarditis; b) pyelonephritis; c) otitis; d) meningitis; e) sinusitis. 119. Typical X-ray signs of legionellosis are the following (2): a) massive infiltration in lungs; b) small-focal infiltration; c) pleuritis; d) bilateral affection of lungs; e) abscesses;

11

120. What specimens have be taken from patient for laboratory diagnostics of legionellosis (3)? a) sputum; b) blood; c) feces; d) spinal fluid; e) endotracheal aspirate. 121. Witch test from mentioned below does not prove legionellosis? a) culturing; b) immune enzyme analysis; c) seroconversion with 4 fold increasing of antibodies titer; d) titer of antibodies 1:64 or more in a single sample; e) detection of legionella antigen in urine. 122. Most effective and cheap method of diagnostics of legionellosis is: a) culturing; b) detection of antigen in tracheal aspirate with PCR; c) detection of antibodies with IHAT; d) detection of antigen in urine with direct immunofluorescent test; e) detection of antigen in sputum with direct immunofluorescent test. 123. What antibiotics from mentioned below are not effective for treatment of legionellosis (2)? a) Ciprofloxacin; b) Penicillin; c) TTrimetoprim-sulfametoxazol; d) Ceftriaxone e) Gentamicin 124. What drug is effective for disinfections of water tanks as prophylactic of legionellosis? a) Detergents; b) Chloramines; c) Calcium hypochlorid; d) Lysol; e) Potassium permanganate. 125. For what season epidemic outbreaks of legionellosis are the most typical? a) winter; b) spring; c) summer; d) autumn; e) not connected with season. 126. Which factor is out of risk for legionellosis? a) young age; b) chronic lung diseases; c) administration of glucocorticoids; d) AIDS; e) alcoholism. 127. Which sign is not included into criterions of early diagnostics of legionellosis? a) presiding disease with toxicosis and hyperthermia during 4-5 days; b) cough; c) diarrhea; d) limphomonocytosis;

e) elevation of serum KFK and LDG levels. 128. Laboratory indices, typical for legionellosis are all following except of: a) hyponatriumemia; b) 4-5 fold AlaT eleveation in blood; c) hypoalbuminemia; d) elevation of creatinine and urea levels in blood; e) ESR 50 mm/h. 129. Which complication from mentioned below is not typical for legionellosis? a) toxic shock; b) acute renal failure; c) edema of lungs; d) hepatic coma; e) edema of brain. 130. Which epidemiological factor from mentioned below can be connected with legionellosis: a) haymaking; b) excavations; c) cafeteria feeding; d) contact with animals; e) drinking of unboiled water. 131. Which antibiotic is most effective for treatment of legionellosis? a) Oxacillin; b) Cepfriaxon; c) Vancomicin; d) Meticillin; e) Rovamicin.

Hope is the good breakfast, but a bad supper.

12

MEASLES 132. What is the ethiological agent of the measles? a) Candida, b) Fusospirilles, c) Myxovirus; d) Rickketsia. e) Bacillus. 133. Is the causative agent of measles pathogenic for monkeys? a) yes; b) no. 134. Who is the reservoir of measles infection? a) monkeys; b) humans; c) cattle; d) rodents; e) all mentioned. 135. What is the route of transmission of measles? a) alimentary; b) air-droplet; c) contact with rodent excreta; d) sexually transmitted disease; e) through the mosquito bites. 136. The susceptibility of humans to measles is: a) not high; b) absolute; c) very high (susceptibility index is 0.96). 137. Post-infective immunity after measles is: a) steady, life - long; b) steady, during one year; c) absent. 138. What is the portal of entry for agent of measles? a) skin; b) mucous membrane of the upper respiratory tract and possibly the conjunctiva; c) tonsils; d) gastrointestinal tract; e) all mentioned. 139. What is the most severe complication of measles? a) pneumonia; b) gangrenous stomatitis; c) encephalitis; d) bronchitis; e) purulent otitis. 140. Where are the principal pathological changes of measles localized? a) liver; b) cardiovascular system; c) nasopharynx, respiratory tract and skin; d) gastrointestinal tract; e) all mentioned. 141. What are the Belsky-Filatov-Koplik spots? a) rash on skin; b) spots scattered over mucous membrane of a mouth in the initial stage of measles; c) rash on the conjunctiva; d) all mentioned. 142. When do the Belsky-Filatov-Koplik spots disappear?

a) in first days after initial catarrhal period; b) in the eruptive stage; c) during convalescence; 143. What is duration of incubation period of measles? a) from 1 to 5 days; b) from 10 to 45 days; c) from 9 to 10 days d) from 10 days to 1 month. e) 1 year. 144. Where are the first elements of the measles rash usually localized? a) at the conjunctiva; b) *at the skin of neck behind the ears and in center of face; c) at the skin of chest; d) at the skin of extremities; e) all mentioned. 145. What are the most common complications of measles (2)? a) *measles croup; b) serous meningitis; c) *pneumonia; d) gangrenous stomatitis or noma; e) encephalitis.

In the evening one may praise the day.

13

DISEASES WITH PRIMARY ALIMENTARY WAY OF TRANSMISSION TYPHOID FIVER, PARATYPHOID A AND B 146. Pathogens of typhoid and paratyphoids A and B belong to the genus: a) Shigella; b) Salmonella; c) Escherichia; d) Klebsiella; e) Enterobacteria; 147. Pathogens of typhoid and paratyphoids A and B are: a) Gram-negative rods; b) Gram-positive rods; c) Gram-negative cocci; d) Gram-positive cocci; e) All mentioned; 148. Which one of following is the reservoir of typhoid infection? a) live-stock; b) rodents; c) humans; d) wild carnivores; e) all mentioned; 149. The most common way of transmission of typhoid infection, paratyphoids A and B is: a) contact through the injured skin; b) air-droplet; c) transmissible; d) sexual; e) * alimentary. 150. Toxins, produced by pathogens of typhoid and paratyphoids A and B belong to: a) exotoxins; b) endotoxins; c) exo- and endotoxins; d) No one from mentioned. 151. Immunity after typhoid fever is: a) antibacterial, short term; b) antibacterial,; c) antitoxic, short term; d) antitoxic long-term, e) all mentioned; 152. Which systems are most affected in case of typhoid and paratyphoids A and B (2): a) respiratory; b) cardiovascular; c) digestive; d) lymphatic; e) nervous. 153. What is the fever pattern, characteristic for typhoid in climax of disease? a) remittent; b) intermittent; c) constant; d) undulating; e) no one of mentioned;

154. Specific complication of typhoid and paratyphoids A and B is: a) intestinal bleeding; b) arthritis; c) otitis; d) intestinal obstruction; e) hypovolemic shock; 155. Specific complication of typhoid and paratyphoids A and B is: a) intestinal obstruction; b) perforation of a thin intestine; c) pulmonary bleeding; d) encephalitis; e) polyradiculoneuritis 156. Which antibiotic is the drug of choose for treatment of typhoid and paratyphoids A and B? a) Penicillin; b) Metronidazole; c) Furazolidone; d) Laevomycetin; e) Sulfadimesine; 157. At patient H. on the 20th day from beginning of typhoid doctor has suspected the intestinal bleeding. What is further medical tactics? a) urgent operative measures; b) hemotransfusion and infusion of the blood substitutes; c) increasing in a daily dose of antibiotics; d) prescription of glucocorticoids (prednisolone, etc.); e) no one mentioned. 158. What is the most typical cause of death in patients with the typhoid and paratyphoids A and B during the first week of disease? a) intestinal bleeding; b) hypovolemic shock; c) infectious - toxic shock; d) peritonitis; e) encephalitis; 159. Typical onset of clinical manifestations of typhoid in nonimmune persons is: a) acute; b) subacute; c) gradual; d) all mentioned; e) no one mentioned. 160. The early symptom of the typhoid and paratyphoids A and B is: a) exanthema; b) enlargement of spleen; c) relative bradycardia; d) diarrhea; e) fever; 161.What is most typical exanthema in the typhoid fever? a) urticaria; b) petechial; c) roseolar; d) erythematous; e) vesicular;

14

162. The specimen for early bacteriological investigation of typhoid and paratyphoids A and B is: a) blood; b) urine; c) feces; d) sputum; e) nasopharyngeal secret. 163. What peculiarities of blood cell count are typical for climax period of typhoid fever? a) leucopoenia, neutropoenia, eosinopoenia, relative lymphocytosis; b) normal count or insignificant leucocytosis, tendency to neutrophils increasing, drum stick shift to the left, monocytosis, plasmatic Turk cells; c) leucopoenia, aneosinophylia, lymphocytosis, thrombocytopoenia, ESR elevation; d) leucocytosis, neutropoenia, lymphomonocytosis, atypical mononuclear.

Soon learnt soon forgotten.

15

SALMONELLOSIS 164. The main way of transmission of salmonelloses: a) alimentary; b) through the using of unboiled water; c) air-droplet; d) contact; e) All mentioned. 165. The specimen from patient with localized form of the salmonellosis for bacteriological diagnostics is: a) blood; b) feces; c) urine; d) bile; e) all mentioned. 166. What is the most typical clinical symptom of generalized forms of salmonelloses? a) repeated vomiting; b) polylymphoadenopathy; c) erythematous exanthema; d) high and long fever; e) All mentioned 167. What is the main specimen for bacteriological diagnostics of generalized form of salmonellosis? a) blood; b) feces; c) urine; d) bile; e) cerebrospinal fluid; 168. What is the most typical clinical sign of gastrointestinal form of salmonellosis? a) fever; b) hepatosplenomegaly; c) dyspeptic disorders; d) headache, a giddiness; e) rash. 169. What is the basic method of treatment of patients with gastrointestinal form of salmonellesis? a) antibacterial; b) rehydration; c) symptomatic; d) detoxication; e) dehydration. 170. What is the basic method of treatment of patients with generalized forms of the salmonellosis: a) diet; b) antibacterial therapy; c) rehydration; d) activating of immune system; e) symptomatic. 171. Which diseases it is necessary to differentiate gastro-intestinal form of the salmonellosis with at first? a) intestinal amebiasis; b) pseudotuberculosis; c) food poisonings; d) typhoid fever and paratyphoids; e) no one from mentioned. 172. What is the method of specific diagnostics of gastrointestinal form of salmonellosis? a) Culturing of feces;

b) c) d) e)

Bacterioscopy of feces; latex agglutination test; rectosygmoidoscopy no one from mentioned;

Wealth is nothing without health.

16

FOOD POISONINGS 173. What is duration of the food poisonings incubation period? a) 1-12 hours; b) 2-3 days; c) 4-6 days; d) 7-14 days; e) 12-24 hours. 174. What complication is characteristic for food poisonings? a) infectious-toxic shock; b) hypovolemic shock; c) edema of the brain; d) pneumonia; e) all mentioned. 175. The is the main factor of the pathogenicity of food poisonings? a) activity of microorganism; b) production of toxin; c) allergic. 176.What is the basic method of food poisonings therapy? a) antibacterial; b) symptomatic c) lavage of stomach; d) diet; e) immunocorrection. 177. The medication for treatment of food poisonings: a) furazolidone; b) glucosaline; c) tetracycline; d) biseptole; e) all mentioned. 178. The volume of fluid for rehydration in case of acute intestinal diseases is determined with: a) intensity of intoxication; b) intensity and duration of fever; c) degree of dehydration; d) frequency of defecations; e) all mentioned. 179. The most effective measure for treatment of food poisoning is using of: a) antibiotics; b) antidiarrheal drugs; c) lavage of stomach; d) analgetics; e) diet. 180. Lavage of stomach is effective when we use such volume of fluid as: a) 500 ml; b) 1000 ml; c) 1,5 l; d) 3 l; e) 4-5 l; 181. Water for lavage of stomach must be: a) of room temperature; b) hot; c) cold; d) worm;

e) of any temperature; 182. Clinical manifestation of food poisoning is characterized with: a) high temperature and intensive watery diarrhea; b) nausea, repeating vomiting, subfebrile temperature; c) protracted signs of the disease; d) admixture of mucus in stool; e) spastic pain in the lover part of abdomen; 183. Incubation period of food poisoning is: a) 30 min-6 h; b) 6-12 h; c) 12-24 h; d) up to 2 days; e) up to 5 days. 184. For bacteriological investigation of patients with food poisoning we have use such specimens as: a) contents of vomiting, lavage water of stomach, stool; b) stool, blood; c) stool duodenal drainage; d) contents of vomiting, stool; e) contents of vomiting, blood. 185. What measures of treatment are not effective in case of food poisoning? a) antibiotics; b) antidiarrheal drugs; c) lavage of stomack; d) spasmolitics; e) diet.

Deeds not words.

17

BOTULISM 186. The general factor of the pathogenesis of the botulism is: a) toxemia; b) bacteremia; c) bacteremia and toxemia; d) inflammatiive processes in organs; e) dehydration. 187. The toxin, formed by the pathogen of botulism, belongs to: a) endotoxin; b) exotoxin; c) exo-and endotoxin; d) no one from mentioned 188. System, that is selectively affected with toxin of botulism is: a) cardiovascular; b) digestive; c) nervous; d) respiratory; e) urinary. 189. On reception to the doctor the patient complained on acute development of disease with repeated vomiting soon followed with disorders of vision. At physical examination of the patient mydriasis, bilateral ptosis, and dryness of mucous of the mouth are detected; body temperature was normal and abdomen was slightly inflated. What is most likely diagnosis? a) poisoning with belladonna; b) viral encephalitis; c) botulism; d) poisoning with methyl alcohol; e) poisoning with mushrooms. 190. What is the medication for specific detoxication in case of botulism? a) trivalent botulinus antitoxin; b) interferon; c) vaccine; d) reopolyglucine e) Ringer solution. 191. What clinical symptoms are most important for early diagnostics of botulism: a) dryness of mouth; b) nausea, vomiting, diarrhea; c) progressive weakness and disorder of vision; d) bulbar symptoms (dysphony, disorders of swallowing, dysarthria etc.); e) high fever, intoxication. 192. What is the main method of treatment of the botulism? a) antibacterial; b) symptomatic; c) antitoxic specific; d) nonspecific detoxication; e) all mentioned.

18

PSEUDOTUBERCULOSIS. 193. What is the reservoir of pseudotuberculosis? a) humans; b) ticks; c) birds; d) rodents; e) All mentioned. 194. What is the main factor in pathogenesis of a pseudotuberculosis? a) bacteremia; b) dehydration; c) immunosupression; d) toxicosis and specific allergy; e) proliferation. 195. What is the main mechanism of rush development in pseudotuberculosis? a) drift of pathogens in a skin lymph vessels with development of perifocal inflammation; b) toxic and allergic affection of the vessel wall; c) generalized proliferative and destructive trombovasculitis; d) fixation of immune complexes in the capillaries of a skin; e) all mentioned. 196. What is the type of rush, which appears on soles and palms: in patients with pseudotuberculosis? a) urtical; b) vesicular; c) erythematous; d) roseolar; e) petechial. 197. Clinical form of pseudotuberculosis with presence of rush is more often necessary to differentiate with: a) measles; b) chickenpox; c) rubella; d) scarlet fever; e) all mentioned. 198. What is the most typical clinical form of a pseudotuberculosis? a) arthritic; b) mixed; c) generalized; d) abdominal; e) icteric. 199. What specimens from patients with pseudotuberculosis may be used for bacteriological investigation? a) blood; b) stool; c) material obtained at surgical exploration d) rectal swabs; e) all mentioned;. 200. Which two groups of antibiotics Y. pseudotuberculosis are generally susceptible to? a) penicillins; b) * aminoglycosides; c) * tetracycline;

d) nitrofurans; e) carbenicillin. 201. Which two groups of antibiotics Y. pseudotuberculosis are generally susceptible to? a) macrolids; b) * third-generation cephalosporins; c) * fluoroquinolones; d) first- generation cephalosporins e) ampicillin. 202.Which methods of laboratory investigation are usually used for diagnostics of pseudodotuberculosis? a) culturing of sputum; b) bacterioscopy; c) skin allergic test; d) * serological; e) all mentioned. 203.What diseases from following which manifest with exanthema are more often necessary to differentiate with pseudotuberculosis? a) typhoid fever; b) louse-born typhus; c) medicamentous toxicodermitis; d) scarlet fever; e) herpes zoster. 204. What of the following clinical forms of pseudotuberculosis are observed more often? a) articular; b) mixed; c) generalized; d) abdominal; e) icteric;

First think, than speak.

19

Idleness is the mother of all evils INTESTINAL YERSINIOSIS 205. The most often clinical form of the intestinal yersiniosis is: a) appendicular; b) gastritic; c) enterocolitic; d) gastroenteritic; e) generalized. 206. Diarrhea in intestinal yersiniosis is caused by: a) strengthened production of water and electrolytes in lumen of intestine; b) inflammation of a mucous of gastrointestinal tract; c) increasing of osmotic pressure in lumen of intestine; d) all mentioned; e) no one from mentioned; 207. What is the most typical complication for transabdominal form of intestinal yersiniosis? a) appendicitis; b) pyelonephritis; c) meningitis; d) myocarditis; e) all mentioned. 208. What place of the pathogen inoculation is typical for intestinal yersiniosis? a) lymphoid formations of oropharyngeal region; b) mucous of a stomach; c) mucous of ileum; d) mucous of rectum and sigmoid colon; e) all mentioned. 209. What is the most frequent clinical form of intestinal yersiniosis? a) apendicular; b) gastritic; c) enterocolitic; d) gastroenteritic; e) meningeal. 210. What is the cause of diarrhea during intestinal yersiniosis? a) amplified production of water and electrolytes into a gut gleam; b) inflammation of the gastro - intestinal mucous; c) increase of the osmolar pressure in a gut contents. 211.What is the most typical complication for abdominal form of intestinal yersiniosis? a) appendicitis; b) pyelonephritis; c) meningitis; d) myocarditis. e) All mentioned. 212. What is the place of introduction of the pathogen of intestinal iersiniosis: a) lymph formations of the throat; b) stomach mucosa; c) jejune mucosa; d) rectum and sigmoid colon mucosa; e) all mentioned.

20

ECHERICHIOSIS 213. What types of Escherichias cause diseases similar to shigellosis? a) enteroinvasive; b) enterotoxigenic; c) enteropathogenic; d) enterohemorrhagic; e) all mentioned. 214. What types of Escherichias cause cholera-like diseases? a) enteroinvasive; b) enterotoxigenic; c) enteropathogenic; d) enterohemorrhagic; e) all mentioned. 215. What is the method of specific laboratory diagnostics of dysentery-like escherichiosis? a) serological; b) bacteriological c) allergological; d) clinical; e) instrumental. 216. What is the basic method of therapy of cholera-like escherichiosis? a) detoxication; b) diet; c) rehydration; d) desensitizing therapy; e) all mentioned.

Self-done is well done.

21

SHIGELLOSIS 217. What is the mechanism of transmission of shigellosis? a) contact; b) air-droplet; c) fecal-oral; d) through the bites of bloodsucking insects; e) all mentioned. 218. Active production of exotoxins is possible for shigellas: a) Sonnei; b) Flexneri; c) Boydii; d) Dysenteriae 1; e) Dysenterie2. 219. The typical site of multiplication of the shigellosis pathogen in human organism is: a) lymphatic formations of a thin intestine; b) mucosa of large bowel; c) parenchymatous organs; d) blood; e) mesenteric lymph nodes. 220. The most severe course of shigellosis causes: a) S. Sonnei; b) S. Flexneri 2; c) S. Boydii 6; d) S. Dysenteriae 1; e) all mentioned. 221.What parts of gastrointestinal tract are most affected in shigellosis? a) stomach; b) duodenum; c) small bowel; d) caecum; e) sigmoid colon and rectum. 222. What is the reservoir of shigellosis? a) domestic birds; b) large and small horned cattle; c) humans; d) rodents; e) cats and dogs; 223. What is the type of immunity acquired after shigellosis? a) antibacterial, long-time; b) antibacterial, short-time; c) antitoxic, long-time; d) antitoxic, short-time; e) no one mentioned. 224. The most effective antibiotic for treatment of shigellosis is: a) Levomycetin (Chloramfenicol); b) Furazolidone; c) Metronidazole; d) Ciprofloxacin (Cifran); e) Biseptole; 225. The main method of treatment of moderate form of shigellosis is: a) antibacterial; b) pathogenetic;

c) symptomatic; d) diet; e) no one from mentioned. 226. What is the basic method of specific laboratory diagnostics of the shigellosis? a) allergological; b) biological; c) bacteriological; d) clinical; e) serological; 227. What is the main factor of the pathogenesis of shigellosis? a) bacteremia; b) toxicosis; c) specific allergy; d) alteration; e) proliferation. 228. What clinical symptom from following is characteristic for the shigellosis? a) indomitable vomiting; b) enlargement of liver; c) liquid stool with slime (and a blood); d) pain in the upper parts of abdomen of girdling character; e) no one mentioned. 229. Affection of what part of gastrointestinal tract is typical for shigellosis? a) stomach; b) small bowel; c) proximal part of large bowel; d) distal parts of large bowel. 230. What are the clinical symptoms of colitic form of shigellosis? a) Gradual onset, absence of intoxication, moderate spastic pains in lower part of abdomen, alternation of constipation and diarrhea, sometimes slime in feces; b) Acute onset with sharp pain in abdomen, symptoms of intoxication on the beginning are absent; stool is bloody, frequently without feces; c) Acute onset, moderate intoxication, diffuse pain in abdomen, vomiting, then liquid stool with admixture of slime (and sometimes of blood); d) Acute onset, moderate intoxication, spastic pain in left lower quadrant of abdomen, poor liquid stool with slime and blood. 231. What character and frequency of stool is typical for colitic form of shigellosis? a) Liquid, plentiful, slime and blood (jelly-like), frequent; b) Liquid, poor, frequent, with admixture of slime and blood; c) Liquid, plentiful, frequent, of greenish color; d) Liquid, plentiful, rice-water-like, frequent; e) Bloody, without feces, frequent. 232. Which antibiotics are effective for treatment of shigellosis? a) Erythromycin;

22

b) First-generation cephalosporins; c) Ciprofloxacin; d) Metronidazol (Trichopol); e) Penicillin; 233. Which data from patient history may not serve as possible source of shigellosis? a). contact with ill persons; b). contact with carriers; c). using of unboiled milk; d). * using of eggs; e). using of unwashed apples. 234. What specimens from patients with shigellosis you have to obtain for bacteriological investigation? (2) a). blood; b). * feces; c). lavage water from stomach; d). * rectal swabs; e). swabs from throat. 235. What characteristic for shigellosis changing from mucosa of intestine does rectosigmoidoscopy reveal? (2) a). * catarrhal inflammation; b). slight hyperemia of mucous membrane and large, deep ulcers; c). * catarrhal inflammation, hemorrhages, not numerous erosions; d). catarrhal and hemorrhagic inflammation with numerous erosions and small ulcers; e). all mentioned.

23

CHOLERA 233. What is the duration of the incubation period of cholera (in days)? a) 1-5; b) 7-14; c) 10-21; d) 25-30; e) 1-2. 234. Typically cholera begin from: a) vomiting; b) abdominal pains; c) fever; d) watery diarrhea; e) all mentioned. 235. The general cause of death of patients with the cholera is: a) infectious - toxic shock; b) hypovolemic shock; c) acute renal failure; d) edema and swelling of a brain; e) all mentioned. 236. Feces of patients with cholera usually contain: a) slime; b) blood; c) slime and blood; d) undigested food e) do not contain neither slime and blood or undigested food.. 237. What is most typical location of V. cholerae in human organism? a) in lumen of intestine; b) penetrate into enterocytes; c) penetrate into mesenteric lymph nodes; d) penetrate into bloodstream; e) penetrate through the BBB. 238. Toxins of V. cholerae are: a) endotoxins; b) exotoxins; c) exo- and endotoxins; 239. The main mechanism of diarrhea in case of cholera is: a) high permeability of intestinal vessels; b) hyperperistalsis of intestine; c) the increased osmotic pressure in a lumen of an intestine (malabsorption); d) breach of function of enzymatic systems of enterocytes (adenylate cyclase-cAMP); e) all mentioned. 240. What is the mechanism of cholerogen action? a) increased peristalsis of an intestine; b) catarrhal inflammation of mucous tunic of intestine; c) electrolytic diarrhea; d) catarrhal inflammation of mucous tunic of small and large bowels; e) all mentioned. 241. What pathogenic mechanisms of acute renal failure are typical for cholera?

Dehydration, hemoconcentration, blocking of microcirculation, reduction of renal filtration; b) Toxemia, loss of water and electrolytes, disturbance of peripheral microcirculation; c) Dehydration, metabolic acidosis, decreasing of potassium level in blood; d) Increase of vessels permeability, hemoconcentration, blocking of microcirculation, 242. What pathogenic mechanisms of tonic cramps are typical for cholera? a) Dehydration, decreasing of arterial pressure, reduction of renal filtration; b) Toxemia, disturbance of peripheral microcirculation; c) Dehydration, metabolic acidosis, decreasing of potassium level in blood; d) Increase of water and electrolytes secretion as a result of activation of adenilatcyclaze and accumulation of cAMP. 243. What are the character and frequency of stool typical for moderate gravity of cholera? a) Liquid, frequent, plentiful, with admixture of slime and blood (jelly - like); b) Liquid, poor, frequent, with admixture of slime and blood; c) Liquid, frequent, plentiful, with admixture of "greens"; d) Liquid, frequent, plentiful, rice-water-like,; e) Liquid, plentiful, "foamy", 3-4 times per day; .244. What is a role of antibiotics in treatment of cholera?(2) a). * reducing of stool output; b). detoxication; c). * shortening of the period of excretion of V. cholere; d). decrease of inflammatiive processes in intestine; e). prevention of bacteremia. 245. What is the main specimen for laboratory diagnostic of cholera? a) blood; b) urine; c) * stool; d) rectal swabs; e) bile. 246. Which saline solution we have to begin use first for treatment of cholera? a) Ringer solution; b) Trisole; c) Disole; d) Acesole; e) Isotonic solution.

a)

24

ROTAVIRAL DIARRHEAS 244. What is the main mechanism of transmission of diarrheas of rotaviral etiology? a) through the bites of bloodsucking insects; b) fecal-oral; c) air-droplet; d) contact; e) all mentioned. 245. Which parts of the gastrointestinal tract are usually affected in case of rotaviral infection? a) stomach; b) upper parts of a small bowels; c) upper parts of large bowels; d) sigmoid colon and rectum; e) all mentioned. 246. The main mechanism of the diarrhea in case of rotaviral infections is: a) breach of an adsorption of a liquid for the account of malabsorption; b) hyperperistalsis of an intestine; c) increased permeability of microvessels of an intestine; d) breach of function of adenylate cyclase-cyclic adenosine monophosphate (cP) system; e) all mentioned. 247.What kind of feces is typical for rotaviral diarrhea? a) mucous with blood; b) watery, plentiful, like "rice broth"; c) foamy, of yellowish color; d) watery, with "greens" e) all mentioned. 248. Abdominal pain in patients with rotaviral gastroenteritis usually is: a) diffuse, over all abdomen, not acute; b) spastic, in the lower areas of abdomen, intensive; c) intensive, in the upper areas of abdomen, of constant character; d) intensive, in the right iliac area, quite often constants; e) all mentioned. 249. What are the methods of specific diagnostics of rotaviral gastroenteritis? (2) a) bacteriological of feces; b) * detection of antibody titer rise with ELIZA; c) bacterioscopy of feces; d) * detection of antigen in feces with latex agglutination test; e) rectosygmoidscopy 250. Which drugs may be used for treatment of rotaviral infection? a) infusions of saline solutions; b) sorbents; c) ferments; d) antidiarrheal drugs; e) * all mentioned. ;

25

VIRAL HEPATITIS 250. What is the superficial antigen of the hepatitis B virus? a) HbcorAg; b) HBsAg; c) HBeAg; d) HbxAg; e) HbyAg. 251. Which one of the following secrets contains maximal quantity of the hepatitis A virus? a) sperm; b) feces; c) urine; d) saliva; e) all mentioned. 252. Affection of which from mentioned cells is most typical for hepatitis B virus? a) epithelial cells of cholangioles; b) hepatocytes; c) Kupffer cells; d) T-helpers; e) macrophages. 253. The long presence of HbeAg in blood serum is evidence of: a) chronization of a viral hepatitis B; b) convalescences phase of acute viral hepatitis B; c) fulminant course of an acute viral hepatitis B; d) healthy "carriage" of a hepatitis B virus; e) all mentioned. 254. What steadiness of the hepatitis B virus on objects of the environment? a) unstable; b) poorly steady; c) high-steady; d) moderate steady. 255. What is the main source of the hepatitis B virus? a) patients with icteric forms of an acute viral hepatitis B; b) patients with the subclinical form of a viral hepatitis B; c) patients with the chronic form of a viral hepatitis B; d) healthy carriers of HBV; e) no one from mentioned. 256. What is the most typical syndrome for the preicteric period of the viral hepatitis A? a) astheno-vegetative syndrome; b) polyarthralgia; c) dyspeptic syndrome; d) influenza-like syndrome; e) mixed; 257. What is the most typical syndrome for the preicteric period of the viral hepatitis B? a) dyspeptic; b) polyarthralgia; c) influenza-like syndrome; d) astheno-vegetative;

e) mixed; 258. What is the most frequent clinical form of viral hepatitis A? a) subclinical; b) unicteric; c) cholestatic; d) icteric; e) fulminant. 259. What is the typical sign for the climax period of acute viral hepatitis? a) erythematic exanthema; b) fever; c) enlargement of liver; d) enlargement of peripheral lymph nodes. e) all mentioned. 260. What is the biochemical parameter, which constantly changes in the preicteric period of acute viral hepatitis? a) activity of alanine aminotransferase (ALaT); b) level of a bilirubin; c) thymol test; d) sublimate titer; e) all mentioned. 261. Decreasing of liver size in climax period of viral hepatitis B is evidence of: a) cholangitis; b) abscess of liver; c) massive necrosis of liver; d) dyskinesis of bile-excreting ways; e) all mentioned. 262. What symptom is most typical for initial period of hepatic failure in acute viral hepatitis B? a) loss of appetite; b) giddiness, sensation of "failure"; c) dermal itch; d) fever; e) all mentioned. 263. What symptoms are most typical for acute hepatic failure? a) increasing of jaundice and hepatomegaly; b) enlargement of level of common bilirubin and cholesterine; c) increase of jaundice and decrease of the sizes of a liver; d) erythematic skin rash and fever; e) all mentioned. 264. What type of viral hepatitis more often cause chronization of disease and formation of cirrhosis of liver? a) A; b) B; c) C; d) ; e) D. 265. In what group of patients lethal outcome of viral hepatitis E are observed more often? a) pregnant women; b) homosexuals; c) dragabused; d) recipients of blood

26

e) immunosupressed patients. 266. In what case of viral hepatitis B formation of the chronic active hepatitis and cirrhosis of the liver is typical? a) superinfection with the virus D (delta); b) coinfection with the virus D (delta); c) superinfection with the virus ; d) coinfection with virus A; e) all mentioned. 267. What is the most often clinical form of viral hepatitis C? a) unicteric; b) icteric; c) fulminant; d) hemorrhagic; e) all mentioned. 268. What is the basic method of therapy of viral hepatitis A? a) symptomatic; b) pathogenetic; c) immunocorrection; d) antiviral; e) all mentioned. 269. What is the basic method of confirmation of acute stage of viral hepatitis B? a) detection with ELISA method of anti-HBs IgG in blood serum; b) detection with ELISA method of anti-HBcore IgM in blood serum; c) detection with ELISA method of HBs Ag in blood serum; d) detection with ELISA method of anti-HBe IgG in blood serum; e) all mentioned. 270. Choose symptoms, typical for icteric form of viral hepatitis A: a) Fever, lymphoadenopathy, oliguria, significant serum ALAT elevation; b) Lymphoadenopathy and fever are absent, function of kidneys is normal, hepatomegaly, significant serum ALAT elevation; c) Fever, generalized lymphoadenopathy, function of kidneys is normal, hepatosplenomegaly, moderate serum ALAT elevation; d) Generalized lymphoadenopathy, fever is absent, the function of kidneys is normal, significant serum ALAT elevation. 271. Chose symptoms, typical for icteric form of viral hepatitis B: a) Gradual onset from nausea, loss of appetite, increasing itch of skin with scratches traces; jaundice; turgor of skin is reduced; hepatomegaly; in gall-bladder projection elastic painless formation 33cm is palpated.. b) Acute beginning from nausea, pain in a right upper quadrant of abdomen, fever up to 38C; moderate jaundice; slight hepatomegaly; palpation of gall-bladder projection is painful.

Gradual onset from nausea, pain in joints, itch of skin, and loss of appetite; presence of jaundice; turgor of skin is normal; .moderate hepatomegaly, palpation of edge of liver is sensitive. d) Acute onset with chill, fever up to 38,8C, nausea, dizziness, weakness; moderate jaundice, temperature is 39C;. hepatosplenomegaly and. anuria within 10 hours. 272. Which one of the following infection routes is most often involved in the neonatal transmission of hepatitis B virus (HBV)? a) Blood transfusion; b) * Fetal contact with infected blood during childbirth; c) Ingestion of the virus via maternal breast milk; d) Transmission of the virus from hospital personnel during childbirth; e) Transplacental transmission of the virus.

c)

27

ENTEROVIRAL DISEASES 272. The most often etiological factor of viral diarrheas: a) Reoviruses; b) Enteroviriuses; c) Adenoviruses; d) Rotaviruses; e) Coronaviruses. 273. The general method of therapy of viral diarrheas: a) therapy with antibiotic; b) symptomatic; c) immunocorrection; d) detoxication; e) antiviral therapy. POLIOMYELITIS 274. Polioviruses are representatives of group: a) Enteroviruses b) Orthomyxoviruses; c) Rhinoviruses d) Adenoviruses; e) Retroviruses. 275. Polioviruses may be cultured in: a) hen eggs; b) monkey kidney cells; c) lymphocyte culture; d) ntracerebral inoculation in multi-nipple mice; e) respiratory epithelium of mice. 276. Specimens, which are commonly used for cultivation of polioviruses, are: a) throat swabs; b) feeces; c) cerebrospinal fluid; d) blood; e) throat swabs and stool. 277. Reservoir of poliomyelitis infection is: a) birds; b) rodents; c) goats and catte; d) humans; e) incects 278. The main route of transmission of poliomyelitis is: a) through the parenteral exposure; b) through the bites of insects; c) alimentary; d) air-droplet way; e) alimentary and air droplet ways. 279. Primary multiplication of poliovirus takes place in: a) tonsils and Payer patches of intestine; b) mucous membrane of nasopharynx; c) blood; d) cerebrospinal fluid; e) mesenteric lymph nodes. 280. Poliovirus usually affects all types of nerve cells beside of: a) the anterior horn cells of spinal cord; b) reticular formation cells; c) vestibular nuclei; d) deep cerebellar nuclei;

e) cortex of brain. 281. Clinical forms of poliomyelitis are all of following beside of: a) asymptomatic form; b) typhoid-like form; c) paralytic form; d) aseptic meningitis; e) bulbar form. 282. In typical cases of poliomyelitis temperature pattern is: a) continual; b) hectic; c) wave-like; d) intermittent; e) remittent. 283. Treatment of uncomplicated poliomyelitis includes all measures except of: a) Interferon; b) Ribavirin; c) Antibiotics; d) Maintenance of respiration; e) Massage.

28

PROTOZOAN DISEASES AMEBIAISIS 284. What is the pathogen of amebiasis: a) Balantidium coli; b) Izospora belli; c) Entamoeba hartmanni; d) Entamoeba histolytica; e) Endolimax nana. 285. What is a reservoir of amebiasis? a) humans; b) livestock; c) pigs; d) rodents; e) insects. 286. The basic method of therapy of intestinal amebiasis is: a) rehydration; b) detoxication;; c) diet; d) chemotherapy; e) symptomatic.. 287. Patient B. complains of presence of pain in right lower part of abdomen and jelly-like liquid stool (mucus stained with blood) up to 4-6 times a day during last 6 months. A day before his condition has sharply worsened, body temperature has risen up to 39,5 C, sharp pain in right upper quadrant, increasing during respiration, has appeared. Mentioned symptoms is connected with: a) exacerbation of an intestinal amebiasis; b) amebic hepatitis; c) amebic abscess of liver; d) amebic pericolitis; e) all mentioned. 288. Symptoms, characteristic for the intestinal amebiasis are: a) liquid stool with mucus and blood, spastic pain in the left lower quadrant of abdomen, temperature 39C; b) liquid stool with mucus and blood as raspberry jelly, discomfort in abdomen, subfebrile temperature; c) plentiful watery stool, rumbling in an abdomen, a normal body temperature; d) plentiful watery stool with "greens", pain and rumbling over all abdomen, a body temperature 39.5C; e) all mentioned. 289. The basic method of laboratory investigation of intestinal amebiasis is: a) parasitoscopy of blood smears; b) detection of specific antibodies; c) parasitoscopy of feces; d) rectosigmoidoscopy; e) culturing of blood..

290. What diseases it is necessary first to differentiate with intestinal amebiasis? a) cholera; b) intestinal yersiniosis; c) shigellosis d) salmonellosis; e) rotaviral gastroenteritis. 291. What part of gastro-intestinal tract is affected typically in case of intestinal amebiasis? a) stomach; b) small intestine; c) proximal part of gut; d) distal parts of large intestine; e) duodenum. 292. What are character and frequency of stools in chronic form of intestinal amebiasis? a) Liquid, plentiful, frequent, with slime and blood (jelly - like), b) Liquid, poor, frequent, with admixture of slime and blood; c) Liquid, plentiful, frequent, with an admixture of "greens",; d) Liquid, plentiful, frequent, rice-water-like; e) Bloody, without feces, frequent. 293. What drag may be used for specific therapy of intestinal amebiasis? a) Erythromycin; b) Remantadin; c) Chlorochin (Delagil); d) Metronidazol (Trichopol); e) Ciprofloxacin.

29

BALANTIDIASIS. LAMBLIASIS (GARDIASIS) 294. What is the reservoir of the balantidiasis? a) humans b) cattle c) pigs d) rodents e) all mentioned. 295. What symptom is characteristic for the lambliasis (gardiasis0? a) fever; b) diarrhea; c) spastic pains in hypogastria; d) repeated vomiting; e) all mentioned. 296. The universal medication for treatment of patients with the intestinal amebiasis, balantidiasis and lambliasis is: a) Metronidazole (Trichopol); b) Yatren (Chiniofon); c) Delagil (Chloroquine); d) Erythromycin; e) Norfloxacine. 297. What materials should be taken in case of suspicion of lambliasis? a) contents of duodenum; b) feces with pathological admixtures; c) feces without pathological admixtures; d) blood; e) vomiting masses.. 299. What form of G. lamblia can infect humans?: a) vegetative form; b) cysts; c) all mentioned; 300. Affection of what part of gastro-intestinal tract is typical for lambliasis: a) stomach; b) small intestine; c) proximal part of large intestine; d) distal parts of large intestine. e) * duodenum and small intestine.

30

DISEASES WITH TRANSMISSIVE MECHANISM OF TRANSMITTION

ARBOVIRAL ENCEPHALITS 301. What is the genus of Russian spring-summer encephalitis ? a) Hantavirus; b) Togavirus; c) Bunjavirus; d) * Flavivirus e) .Bunyavirus. 302. Which two symptoms are not typical for Russian spring-summer encephalitis? a) meningitis; b) encephalitis; c) hepatosplenomegaly; d) polyradiculoneuritis; e) polyuria; f) primary affect. 303. Which two symptoms are not typical for Japanese encephaliti?: a) meningoencephalitis; b) early brain edema; c) polyradiculoneuritis; d) early lungs edema; e) residual dementia. 304. Which drug can not be used for the Japanese encephalitis treatment: a) Albumin; b) Heparin; c) Pentamin; d) Lasics; e) Reopolyglucin. 305. What changes in spinal fluid are typical for fever form of tick-borne encephalitis? a) high neutrophilic pleocytosis; b) high lymphocytic pleocytosis; c) low neutrophilic pleocytosis; d) low lymphocytic pleocytosis; e) changes are not present; 306. Which cells from following are most typically affected in case of poliomyelitic form of tick-borne encephalitis? a) cells of anterior horns in cervical and thoracal parts of spinal cord; b) meninges of brain and spinal cord; c) radices of peripheral nerves; d) nuclei of cranial nerves; e) all mentioned. 307. For what of following forms of tick-borne encephalitis development of epileptic attacks is typical? a) meningeal; b) meningoencephalitic; c) poliomyelitic; d) feverish; e) all mentioned. 308. What is the drug for specific therapy of tick-borne

encephalitis? a) Desoxiribonuclease; b) Remantadine; c) Specific horse serum; d) Donor immunoglobulin; e) Ribavirin. 309. Where should be treated patients with meningeal forms of tick-borne encephalitis? a) in out-patient departments; b) in day time hospital; c) in infectious hospital; d) on the center of consultation and diagnostic; e) at home. 310. Where should be treated patients with local forms of tick-borne encephalitis? a) in out-patient departments; b) in day time hospital; c) in infectious hospital; d) in the center of consultation and diagnostic; e) at home. 311. For what of the following forms of tick-borne encephalitis specific seroprophylactic is used? a) unvaccinated persons at first 3 days after bite of tick; b) vaccinated persons at first 3 days after bite of tick; c) all people in region; d) persons who have history of tick-borne encephalitis; e) persons, who visited endemic zone. 312. How long is supervision for persons, who had attacked by ticks? a) 10 days; b) 14 days; c) 21 days; d) 28 days; e) 30 days. 313.What is the main method of tick-born encephalitis specific diagnostic? a) bacteriological; b) virusologycal; c) serological; d) allergologycal; e) epidemiological 314.Which one of the following group of persons is need in vaccination for prevention of tick-borne encephalitis? a) patients with any form of tick-born encephalitis up to7-th day of disease; b) after tick extraction; c) persons, who works in infectious hospitals and consulting-rooms of infection diseases in medical center; d) persons, who are going to work in natural areas of trick-born encephalitis; e) all mentioned. 315. Which two methods of laboratory investigation are not used for diagnostics of arboviral infection? a) viral isolation; b) detection of specific antibodies;

31

c) detection of antigen with IFT; d) skin allergic tests; e) detection of level of kreatinine kinaze in serum. 316. What is the reservoir of infection for Russian spring-summer encephalitis? a) human; b) mammals and birds; c) ticks; d) * ticks, birds and mammals. e) mosquitoes. 317. What is the main way of transmission for Japanese encephalitis? a) contact with animals; b) contact with infected human; c) alimentary (unboiled milk); d) through the bites of ticks; e) through the bites of mosquitoes. 318. What are the two main ways of transmission for Russian spring-summer encephalitis?: a) contact with affected human; b) alimentary (unboiled milk); c) through the bites of ticks; d) through the bites of mosquitoes; e) all mentioned. 319. What are onset symptoms of arboviral encephalitis (3): a) headache; b) vomiting; c) diarrhea; d) muscular pains; e) cough. 320. What are the diagnostics methods of the arboviral infections: a) detection of antigen with direct immunofluorescent test; b) detection of antibodies in titer 1:200 with complement fixation test (CFT) or hemagglutination inhibition test (HAIT) in single serum sample; c) four-fold rising of antibodies titer with HAIT or CFT in serum of acute and convalescent phase; d) isolation of the virus from blood or spinal fluid; e) * all mentioned.

32

LIME-BORRELIOSIS 321. What clinical sign is very important for diagnostics of Lime-borreliosis? a) presence of skin circular erythema; b) history of ticks bite; c) increase of body temperature and presence of toxic symptoms; d) presence of serous meningitis; e) all mentioned. 322. What is the mechanism of affection of joints in patients with Lime-borreliosis? a) bacteremia; b) immunoallergy; c) toxemia; d) septicopiemia; e) productive inflammation. 323. What are the characteristics of erythema typical for Lime-borreliosis? a) acute painful with fluctuation in the center and unclear borders; b) moderately painful with clear borders and elevations above the level of healthy skin; c) moderately painful with pale cyanotic center and bright festooned borders; d) scratched, painless of red-violet color, with clear borders from surrounded skin e) all mentioned. 324. Which one of the following drugs is used for treatment of Lime-borreliosis? a) Tetracycline; b) Gentamycin; c) Erythromycin; d) Lincomycin; e) Biseptol; 325. How long must be supervision for Lime-borreliosis convalescents? a) 3 month; b) 6 month; c) 1 year; d) 2 years; e) life-long. 326. What type of cardiac disturbances is most typical for Lime borreliosis? a) myocarditis; b) endocarditis; c) infarct of myocardium; d) pericarditis; e) all mentioned. 327. What is the cause of relapses of Lime borreliosis? a) septicemia; b) intracellular persistence of pathogen; c) reinfection; d) superinfection; e) immunosupression. 328. What is the cause of chronic Lime borreliosis course? a) immunosupression; b) intracellular persistence of pathogen;

c) reinfection; d) superinfection; e) all mentioned. 329. Which one of the following symptoms is typical for Limes disease? a) generalized lymphadenopathy; b) glomerulonephritis; c) enterocolitis; d) polyradiculoneuritis; e) otitis. 330. What are the most informative laboratory methods for diagnostic of Lime disease? (2) a) culturing of blood and spinal fluid; b) detection of specific IgG antibodies with ELISA; c) * detection of specific IgM antibodies with ELISA; d) isolation of pathogen from joint fluid; e) * detection of pathogen DNA with PCR.

33

LOUSE-BORN TYPHUS 330. How long is maximal many days are maximal incubation period of louse-born typhus? a) 5 days; b) 14 days; c) 20 days; d) 25 days; e) 30 days 331. What complication is typical for climax period of louse-born typhus? a) pneumonia; b) thromboembolism; c) infectious toxic shock; d) peritonitis; e) all mentioned. 332. What complication is typical for convalescent period of louse-born typhus? a) thromboembolism; b) meningoencephalitis; c) psychosis; d) pneumonia; e) dementia. 333. What type of encephalitis is typical for louse-born typhus? a) diffuse purulent; b) diffuse serous; c) local purulent; d) local serous; e) all mentioned. 334. What type of exanthema is typical for louse-born typhus? a) roseolar and petechial; b) macular and papular c) roseolar and papular; d) vesicular and papular; e) erythematous and papular. 335. What is the main factor of louse-born typhus pathogenesis? a) rickettsiemia; b) toxemia; c) autoallergy; d) immunosuppression; e) necrosis. 336. What is the immunity acquired after louse-born typhus? a) passive natural; b) passive artificial; c) active natural; d) active artificial; e) all mentioned. 337.Tthe patient with louse-born typhus may serve as source of infection in one of following cases: a) from last 1-2 days of incubation period and within all period of fever; b) from a beginning of clinical manifestation and up to the 21-st day of normal body temperature; c) from the end of the incubation period and up to standing out of rash;

338. What type of fever is typical for climax period of louse-born typhus? a) constant; b) intermittent; c) wave-like; d) remittent; e) recurrent. 339. What is the basic method for specific laboratory diagnostic of louse-born typhus? a) isolation of rickettsia from blood; b) detection of specific antibodies in serum; c) skin-allergic test; d) isolation of rickettsia from feces; e) isolation of rickettsia from cerebrospinal fluid. 340. What is the drug of choose for treatment of louseborn typhus and Brills disease? a) Penicillin; b) Streptomycin; c) Furazolidone; d) Tetracycline; e) Delagil. 341. What cardiovascular disturbances are typical for climax period of louse-born typhus and Brills disease? a) relative bradycardia and moderate hypotension; b) tachycardia and hypotension; c) bradycardia and hypertension; d) tachycardia and hypertension; e) all mentioned. 342. What peculiarities of blood cell count are typical for climax period of louse-borne typhus? a) Leukopenia, neutropenia, eosinopenia, relative lymphocytosis; b) Normal count or insignificant leukocytosis, tendency to neutrophilia, drum -tick shift to the left, monocytosis, presence of plasmatic cells (Turk cells); c) Leukopenia, aneosinophylia, lymphocytosis, thrombocytopenia, ESR elevation; d) Leukocytosis, neutropenia, lymphomonocytosis, atypical mononuclears.

34

MALARIA 343. Which one of the following agents is causative agent of malaria tertian? a) Pl. ovale; b) Pl. malariae; c) Pl. falciparum; d) Pl. vivax; e) all mentioned. 344. Which one of the following agents is a causative agent of tropical malaria? a) Pl. ovale; b) Pl. falciparum; c) Pl. malariae; d) Pl. vivax; e) L. donavani. 345. What is the basic mechanism of anemia in patients with malaria? a) spleen hyperplasia and hypersplenism; b) hemolysis of erythrocytes as a result of erythrocytic schizogony; c) autoimmune hemolysis of erythrocytes; d) low osmotic resistance of erythrocytes with subsequent hemolysis; e) inherent deficit of glucose-6-phosphate dehydrogenase in erythrocytes 346. Which one of the following forms of plasmodia is the cause of early malaria relapses? a) sporozoites; b) gametocytes; c) exoerythrocytic schizontes; d) erythrocytic schizontes; e) all mentioned. 347. Which one of the following forms of plasmodia is the cause of late malaria relapses in patients with malaria ovale? a) sporozoites; b) gametocytes; c) exoerythrocytic schizontes; d) erythrocytic schizontes; e) all mentioned 348. Which one of the following types of malaria may develop remote relapses? a) Malaria vivax; b) Malaria ovale; c) Malaria falciparum; d) * Malaria malaria; e) all mentioned. 349. How long is interval between paroxysms in patients with tertian malaria? a) 24 h; b) 48 h; c) 72 h; d) 12 h; e) 2 h. 350. Which one of the following types of malaria may develop malarial coma? a) tertian; b) quartian;

c) tropical; d) ovale; e) all mentioned. 351. What is the main pathogenic mechanism of jaundice in patients with malaria? a) break of capture and conjugation of bilirubin in hepatocytes; b) development of cholangitis; c) autoimmune erythrocytes hemolysis; d) * superfluous hemolysis of erythrocytes as a result of schizogony; e) all mentioned. 352. What one of the following complications is typical for tropical malaria? a) acute renal insufficiency; b) pneumonia; c) pulmonary edema; d) cerebral coma; e) all mentioned. 353. What type of fever is typical for malaria? a) remittent; b) constant; c) intermittent; d) wave-like; e) hectic. 354. Which one of the following mechanisms is the main cause of hemoglobinuric fever in patients with malaria? a) hemolysis of erythrocytes as a result of schizogony; b) hemolysis of erythrocytes as a result of decrease of their osmotic resistance; c) autoimmune mechanisms of hemolysis of erythrocytes; d) hypersplenism; e) all mentioned. 355. What is the main method of laboratory diagnostic of malaria? a) detection of specific antibodies; b) parasitoscopy of thin blood smears; c) cultivation of blood on selective media with subsequent pathogen identification; d) identification of pathogen with PCR; e) parasitoscopy of thin blood film and thick blood drop. 356. What is the level of parasitemia in case of malaria, complicated with hemoglobinuric fever? a) absent; b) insignificant; c) moderate; d) significant; 357. What one of the following drugs is the drug of chooses for therapy of malaria? a) Laevomycetin (Chloramphenicole); b) Penicillin; c) Chloroquine (Delagil); d) Metronidazole (Trichopol); e) all mentioned. 358. Which one of the following drugs is the drug of chooses for the treatment of severe tropical malaria?

35

a) Amodiaqine; b) Chloroquine (Delagil); c) Primaquine; d) Quinine sulfate; e) all mentioned. 359. What one of the following drugs may be used for prevention of malaria remote relapses? a) Chloroquine (Delagil); b) Primaquine; c) Quinine sulfate; d) Amodiaquine; e) all mentioned. 360. What are basic clinical symptoms of climax period of malaria vivax? a) Constant fever, chills, significant sweating, lymphoadenopathy, muscular pain, enlargement of spleen and liver; b) Constant type of fever, weakness, adynamia, pale skin, poor roseolar rush on lateral surfaces of trunk, enlargement of spleen and lever; c) Fever of remitting type, chills, significant weakness, petechial enanthema on conjunctiva, sometimes enlargement of liver; d) Fever of intermitting type, on climax of fever feeling of heat with subsequent plentiful sweating, enlargement of spleen and liver. 361. Which one of the following drags is effective for therapy of paroxysms of malaria tropica? a) * Fansidar; b) Remantadine; c) Chloroquine (Delagil); d) Metronidazol (Trichopol); e) Ciprofloxacin. 362. Which one from the following drugs is the drugmay be used for suppressive prophylaxis of malaria in chloroquine-resistant falciparum areas? a) Chloroquine; b) Primaqine; c) Clindamycin; d) * Mefloquine e) Bigumal

36

HELMINTHES 362. What is the route of transmission of Ascariasis? a) using of soil-contaminated food; b) anal-oral self-contamination; c) through skin; d) using of uncooked fish; e) using of undercooked beef. 363. What is the route of transmission of Teniasis? a) using of soil-contaminated food; b) anal-oral self-contamination; c) using of undercooked meat; d) using of uncooked fish; e) through skin and by endogenous autoreinfection. 364. What is the route of transmission of Hookworms? a) using of soil-contaminated food; b) anal-oral self-contamination; c) using of undercooked meat; d) using of uncooked fish; e) through skin and by endogenous autoreinfection.. 365. Pyrantel Pamoate is effective for treatment of: a) Hookworms; b) Enterobiasis; c) Tapeworms; d) Opisthorchiasis; e) Stronglyloidiasis. 366. Which drug from mentioned may be effective for treatment of cystecircosis? a) Pyrantel pamoate; b) Mebendasole; c) Thiabendasole; d) Albendasole; e) Ivermectin. 367. What drug is effective for treatment of Tapeworms? a) Pyrantel pamoate; b) Mebendasole; c) Praziquantel; d) Thiabendasole; e) Albendasole. 368. For treatment of Hookworms the most effective drug is: a) Mebendasole; b) Albendasole; c) Pyrantel pamoate; d) Ivermectin; e) Praziquantel. 369. What is the cause of anemia associated with ancylostomiasis? Bone marrow depression caused by parasitic invasion; b) Vitamin B12 deficiency caused by competitive absorption by the parasite; c) Malabsorption of folic acid as a result of chronic diarrhea; d) Massive hemoptysis during the initial phases of parasitic maturation; Persistent iron loss. a)

37

38

INFECTIOUS DISEASES WITH INOCULATION AND PARENTERAL MECHANISM OF TRANSMISSION

HIV INFECTION AND AIDS 369. Which envelop glycoproteins submit antigenic structure of the HIV 1 type virus? a) gp41, gp20, gp160; b) gp160, gp120, gp41; c) gp41, gp110, gp120; d) gp120, gp180, gp41; e) all mentioned. 370. Where does replication of the HIV occur? a) plasmocytes and macrophages; b) macrophages and T-helpers; c) T-helpers and -lymphocytes; d) -lymphocytes and plasmocytes; e) No one from mentioned. 371. What opportunistic infections and invasions are typical for AIDS? a) generalized candidiasis and pulmonary coccididomycosis; b) leishmaniasis and toxoplasmosis; c) pneumocytosis and disseminated histoplasmosis; d) extrapulmonary tuberculosis and leishmaniasis; e) shigellosis and amebiasis. 372. What is the screening - method of diagnostics of HIV-infection? a) immune-enzyme analysis (IE); b) radioimmune analysis(RIA); c) immunoblot analysis; d) molecular hybridization (DNA-PROBES); e) polymerase chain reaction. 373. What clinical syndrome is characteristic for dementia in patients with HIV-infection? a) syndrome of liquor hypertension; b) multifocal leukoencephalopathy; c) discirculatory infringements of a cerebral circulation; d) edema of the brain; e) spongiform encephalopathy. 374. What is the coefficient 4/8 (the ratio of subpopulations of -lymphocytes) typical for AIDS? a) 0,5; b) 1,5; c) 2,0; d) 3,0; e) 4,0. 375. What is the AIDS-indicatory disease? a) pulmonary tuberculosis; b) pneumocystic pneumonia; c) candidiasis of a mouth; d) herpes labialis; e) intestinal amebiasis;

376. Which one of the following clinical syndromes is characteristic for HIV-infection on stage of persistent generalized lymphoadenopathy? a) enlargement of any 3 groups of lymph nodes, painful on palpation, tendency to suppuration; b) enlargement of inguinal lymph nodes, soldered and painful on palpation; c) enlargement of any 3 groups of lymph nodes, freely movable and painless on palpation; d) enlargement of inguinal and axillary lymph nodes (more than 3 sm.), soldered and painful on palpation; e) enlargement of mesenteric lymph nodes. 377. What is the basic medication for the specific treatment of HIV-infection? a) Remantadine; b) Pentamidine; c) Interferon; d) Azidothymidine; e) Acyclovlr. 378. When the diagnosis of AIDS can be proved clinicaly, disite of negative results of antibodies to HIV testing? a) diarrheas of isospore etiology; b) Kaposi`s sarcoma in persons senior than 60 years; c) pneumocystic pneumonia; d) histoplasmosis of lungs; e) all mentioned.

39

RABIES ); 379. The causative agent of rabies belong to: a) Myxoviruses; b) Rhabdoviruses; c) Retroviruses; d) Hepadnaviruses; e) Arenaviruses. 380. Where is rabies mostly spread? a) in countries of tropic belt; b) in England; c) in Australia; d) in Antarctica; e) in Ukraine. 381. In which regions rabies is not registered? a) in England; b) in Australia; c) in USA; d) in Antarctica. 382. What is the main source of rabies infection ? a) carnivorous; b) herbivorous; c) fish; d) birds; e) insects. 383. What is the main route of transmission of rabies? a) parenteral; b) through the bites of animals; c) through the wounds, contaminated with soil; d) sexual; e) through the bites of insects; 384. The mean incubation period of rabies is: a) 1-4 days; b) 7 days; c) 14 days; d) 1-2 month; e) 1-2 years. 385. What is the most common form of rabies? a) meningeal; b) furious form; c) asymptomatic form; d) medullar form; e) carriage. 386. What is the main cause of death in the stage of excitement of rabies? a) rupture of spine; b) asphyxia; c) edema of the brain; d) dehydration; e) edema of the lung. 387. What is the main cause of death in paralytic stage of rabies? a) rupture of spine; b) asphyxia; c) edema of the brain; d) dehydration; e) edema of the lung. 388. What clinical sign is characteristic for early period of rabies? a) rigidity of the muscles;

b) difficulty of swallowing of water (hydrophobia c) opisthotonos; d) paralysis of cranial nerves; e) disorders of consciousness. 389. What clinical manifestation is most typical for rabies? a) myocarditis; b) encephalitis; c) laryngitis with croup; d) cardiovascular autonomic disorders; e) dehydration. 390. For diagnostics of rabies all mentioned tests are used except of: a) isolation of virus from spinal fluid; b) isolation of virus from brain; c) detection of virus RNA in PCR; d) visualization of typical cellular changes in Ammon horn (Babesh-Negry bodies); e) isolation of virus from the blood; 391. For treatment of rabies all drugs are used except of: a) Interferon; b) Ribavirin; c) Arduan; d) Reopolyglucine; e) Seduxen. 392. Prophylaxis of rabies includes all measures except of: a) preexposure vaccination with rabies vaccine of persons with high risk of infection; b) postexposure local wound therapy with soap and water; c) passive immunization with rabies either equine or human immune globulin; d) passive immunization with tetanus toxoid; e) postexpposure active immunization with rabies vaccine.

40

TETANUS 393. What pathogen from mentioned is the cause of tetanus? a) Clostridia; b) Corynebacteria; c) Listeria; d) Neisseria; e) Bordetella; 394. What is the main route of transmission of tetanus? a) through the bites of insects; b) trough the wounds; c) parenteral; d) sexual; e) alimentary. 395. Where morbidity from tetanus is higher? a) in Europe; b) in South-East Asia; c) in Arctic; d) in USA; e) in Europe. 396. In which cases risk of development of tetanus is higher? a) cut wounds; b) wounds with active infection; a) stings of insects; b) bites of dogs; c) pulmonary abscesses. 397. Mark the main pathogenic factor of tetanus, which has importance in clinical nanifestation? f) tetanolysin; g) tetanospasinin; h) lipopolysaccaride; i) hemolytic membrane protein; j) capsule. 398. In case of tetanus all parts of nervous system may he involved beside of: a) spinal cord; b) brainstem; c) sympathetic chain; d) parasympathetic centers; e) cortex. 399. What is the main pathogenic mechanism of tetanus? a) releasing of inhibitory neurotransmitters with inhibitory neurons; b) blockage of producing acetylcholine in peripheral synapses; c) releasing of excitatory neurotransmitters with excitatory neurons; d) demyelination of neurons axons; e) necrotic changes in neurons. 400. Which group of muscles are involved in manifestation of tetanus firs? a) muscles of face; b) muscles of hands; c) muscles of abdomen; d) muscles of chest; e) muscles of low extremities 401. Average incubation period of tetanus is:

a) 3-5 days; b) 7 days; c) 14 days; d) 20 days; e) 1 year. 402. What 2 symptoms may indicate severe course of tetanus? a) trismus; b) rigidity of the neck; c) dysphagia; d) rigidity of abdomen muscles; e) spontaneous prolonged spasms. 403. All of following symptoms may be indicators of severe tetanus beside of: a) moderate trismus; b) generalized cramps; c) tachypnoea in excess of 40/min; d) apnoic spells; e) hypertension. 404. The main role in treatment of tetanus belongs to: a) antibiotics; b) human tetanus immune globulin or equine tetanus antitoxin; c) discharging of toxin with infusion therapy; d) glucocorticoids; e) anticoagulants. 405. The main two methods of prevention of tetanus are: a) * active immunization with tetanus toxoid; b) active immunization with humane tetanus immune globulin; c) * active immunization with equine tetanus antitoxin; d) active immunization with attenuated vaccine; e) active immunization with recombinant vaccine.

41

ERYSIPELAS 406. Show the causative agent of erysipelas. a) Corinebacteria; b) Streptococci of group A; c) Frysipelothrix; d) Staphylococci; e) Neisseria. 407. Where does erysipelas localize? a) in skin; b) in skin and mucous membranes; c) in brain; d) in sexual organs; e) in joints and muscles. 408. When ought you to suggest erysipelas as repeated form? a) when it repeats half year after primary disease; b) when it repeats 1 year after primary disease; c) when it repeats 1,5-2 years after primary disease; d) when it repeats 2 weeks after primary disease; e) when it repeats 2-3 month after primary disease. 409. What clinical form is not typical for erysipelas? a) erythematous; b) erythematous-hemorrhagic; c) skin-bubonic; d) necrotic; e) bullous. 410. The most frequent localization of erysipelas is: a) low extremities; b) upper extremities; c) face; d) chest; e) sexual organs. 411. What is the base for diagnostics of erysipelas? a) clinical manifestation; b) isolation of causative agent from blood; c) isolation of causative agent from skin and mucous membranes; d) serologic tests; e) skin allergic test. 412. What is the drug of choice for treatment of erysipelas?: a) Penicillin G; b) Nitrofurans; c) Sulfonamides; d) Metronidazole; e) Albendazole.

42

LEPTOSPIROSIS 413. What is the pathogen of leptospirosis? a) Spirochaeta; b) Listeria; c) Corynebacteria; d) Rickettsia; e) Virus. 414. What type of toxins toxin of Leptospira belongs to? a) exotoxin; b) endotoxin; c) exo-and endotoxins; 415. What is the type immunity acquired after leptospirosis? a) antitoxic unsteady; b) antitoxic steady; c) antibacterial unsteady; d) antibacterial steady; e) all mentioned. 416. Which two from following routes of transmission are typical for leptospirosis? a) through the bites of insects; b) * through the contaminated food; c) air-droplet; d) * through the contact of mucous membranes and breaks in the skin with water of rivers or soil; e) using of unboiled water from water-pipe. 417. What is the reservoir of leptospirosis? a) humans; b) ticks; c) rodents; d) birds; e) mosquitoes. 418. Which two from following factors are typical for pathogenesis of leptospirosis? a) * accomplishing of circulating immune complexes in tissues; b) suppression of immune system; c) * circulating of leptospira in bloodstream and establishing them in parenchymatous organs; d) tropism of leptospira to lymphatic system; e) proliferative processes in organs with formation of granulomas. 419. Which human systems are most likely injured in leptospirosis? a) urogenital and cardiovascular systems; b) cardiovascular and respiratory systems; c) respiratory and digestive systems; d) digestive and urogenital systems; e) nervous and digestive systems. 420. What type of exanthema is most typical for leptospirosis? a) vesicular; b) macular; c) petechial; d) erythematous;

e) papular. 422. Choose from the following pathogens two causative agents of leptospirosis. a) S. enteritidis; b) L. Pomona; c) L. pneumophilia; d) L. icterohaemorrhagica; e) Ch. Psittaci. 424. Leptospirae can for a long time survive in: a) food; b) milk; c) water; d) soil; e) air. 425. Choose two groups of risk for leptospirosis. a) medical workers; b) veterinary workers; c) forest-guards; d) drivers; e) fishermen. 426. What is the possibility of human-to-human transmission of leptospirosis? a) high; b) impossible; c) rare. 427. Which two from following ways of transmission are not typical for leptospirosis? a) alimentary; b) * air-droplet; c) * through the bites of insects; d) through the contact of mucous membranes and breaks in the skin with water of rivers or soil. e) through the contact with animals. 428. What pathogenic mechanism is not typical for of leptospirosis? a) development of systemic infectious process; b) development only local infectious process; c) necrotic changes in liver; d) affection of kidneys; e) penetration of the leptospirae into CNS; 429. What pathogenic mechanism of affection of kidneys is not typical for leptospirosis? a) affection of glomeruli; b) affection of tubular system; c) disturbances of microcirculation in a renal parenchyma; d) edema of intersticious space. 430. What is the cause of affection of lungs in patients with leptospirosis? a) affinity of pathogen to the lung tissue; b) interstitial edema; c) hemorrhagic changes in the lung; d) formation of apneumatosises; e) all mentioned. 431. Leptospirae can survive for a long time in: a) lung tissue; b) tubular system of a kidneys; c) liver; d) CNS; e) eye..

43

432. What is the most typical incubation period of leptospirosis? a) 6-12 h; b) 2-3 days; c) 1-2 weeks; d) 1 month. e) 3 month. 433. What is the most typical clinical form of leptospirosis? a) asymptomatic; b) unicteric; c) icteric; d) severe forms with fatal outcome. e) gastrointestinal. 434. What symptom is not typical for clinical manifestation of unicteric form of leptospirosis? a) muscular pain; b) hyperemia of face and conjunctivas; c) tonsillitis; d) meningitis; e) hepatosplenomegaly. 435. What complication of leptospirosis is nonspecific one? a) gastrointestinal bleeding; b) pneumonia; c) insufficiency of kidneys and liver; d) clouding of vitreous body; e) meningitis. 436.Relapses after first attack of leptospirosis usually develop through the period in: a) 1-2 days; b) week; c) month; d) 1 year; e) 3 years. 437. What changes in laboratory tests are not typical for leptospirosis? a) leukocytosis, neutrophilia, aneosinophylia, thrombocytopenia; b) leukocytosis with neutrophilic shift to left, very high ESR; c) leukocytosis, eosinophylia, thrombocytosis; d) increased level of serum creatinine and urea; e) increased level of serum indirect bilirubin 438. What are the specimens for bacteriological diagnostics of leptospirosis? (2) a) blood; b) vomiting masses; c) sputum; d) feces; e) urine; 439. What is the material for early bacteriological diagnostics of leptospirosis (first 5 days)? a) blood; b) vomiting masses; c) sputum; d) feces; e) urine; 440. What are the materials for diagnostics of leptospirosis after 7-10 th days of the disease (2):

a) blood; b) vomiting masses; c) sputum; d) feces; e) urine sediment. 441. What are the two most important methods of serological diagnostics of leptospirosis? a) agglutination test; b) indirect hemagglutination test; c) reaction of microagglutination and lysis of leptospira; d) immunoferment assay; e) compliment fixation test. 442. What are the medicines for treatment of leptospirosis? (2): a) Penicillin; b) Gentamycin; c) Biseptol; d) Ampicillin; e) Remantadine. 443. What are the medicines for treatment of the acute renal failure in patients with leptospirosis? (2) a) Vicasole; b) 7,5% solution of potassium chloride; c) Lazix; d) Dexazone; e) Mechanical respiration. 444. Choose symptoms, typical for icteric form of leptospirosis. a) Fever, lymphoadenopathy, oliguria, moderate serum ALAT elevation, hepatosplenomegaly; b) Lymphoadenopathy and fever are absent, function of kidneys is normal, hepatomegaly, significant serum ALAT elevation; c) Fever, generalized lymphoadenopathy, function of kidneys is normal, hepatosplenomegaly, moderate serum ALAT elevation; d) Generalized lymphoadenopathy, fever is absent, the function of kidneys is normal, significant serum ALAT elevation.

44

DISEASES WITH MULTIPLE WAYS OF TRANSMITTION

HAEMORRAGIC FIVERS 445. What is the genus of the Congo-Crimean hemorrhagic fever pathogen? a) Arenaviruses: b) Hantaviruses; c) Togaviruses; d) Bunyaviruses; e) Flaviviruses. 446. What is the genus of the hemorrhagic fever with renal syndrome? a) Arenaviruses: b) Hantaviruses; c) Togaviruses; d) Bunyaviruses; e) Flaviviruses. 447. What is the genus of the Ebola fever pathogen? a) Arenaviruses; b) Hantaviruses; c) Filoviruses; d) Togaviruses; e) Bunyaviruses. 448. Which two symptoms are not typical for CongoCrimean hemorrhagic fever? a) stomach and gut bleeding; b) kidneys affection; c) hemorrhagic rush on a skin and mycoses; d) acute beginning; e) severe syndrome of intoxication; f) affection of respiratory tract. 449. What drugs can be used for the treatment of hemorrhagic fevers? a) Lamivudin; b) Ribavirin; c) Aciclovir; d) Streptomycin; e) Cephtriaxon. 450. What drug cannot be used for the treatment of hemorrhagic fevers? a) Cycloferon; b) Lasix; c) Heparin; d) Glucocorticoids; e) Aminoglycosides. 451. Choose two target tissues for pathogen of CongoCrimean hemorrhagic fever. a) vessels; b) kidneys; c) liver; d) nervous system; e) lungs. 452. What are the most typical hematological changes in patients with hemorrhagic fever?

normal erythrocytes count, leukocytosis with shift to left, high ESR; b) erythrocytosis, thrombocytosis, leukocytosis with shift to left, high ESR; c) normal erythrocytes count, leukopenia, lymphocytosis, slightly increased ESR: d) normal erythrocytes count, thrombocytopenia, leukopenia, eosinopenia, shift to left of the leucocytes formula, high ESR; e) normal erythrocytes count, leucocytosis with shift to left, very high ESR, thrombocytopoenia. 453. What is the main reservoir of Marburg hemorrhagic fever? a) rats; b) monkeys; c) birds; d) ticks; e) human. 454. What is the main way of transmission of the Marburg hemorrhagic fever? a) contact with monkeys; b) human-to-human contact; c) contact with rats; d) through the bites of insects; e) alimentary. 455. Which ways of transmission are not typical for Lassa hemorrhagic fever? a) air-droplet; b) contact with rodents; c) through the bites of insects; d) alimentary; e) human-to-human contact 456. What symptom is not typical for Lassa hemorrhagic fever? a) high fever; b) ulcers on the mouth mucous; c) muscular pains; d) jaundice; e) hemorrhagic rush; f) affection of kidneys. 457. What symptom is not typical for Murburg & Ebola hemorrhagic fever? a) high fever; b) hemorrhagic rush; c) nasal, stomach and gut bleedings; d) affection of kidneys; e) sore throat. 458. Which factors are not typical for pathogenesis of arboviral infections? a) reproduction in reticulo-endothelial system; b) virusemia; c) syndrome of intoxication; d) affection of kidneys cells. e) all mentioned. 459. What changes in spinal fluid are typical for arboviral encephalitis? a) cytosis of 500*106/l, with prevalence of lymphocytes;

a)

45

b) cytosis from 10 up to 100*106/l, with prevalence of lymphocytes; c) cytosis up to 1000*106/l or more, with prevalence of neutrophiles; d) cytosis lesser than 10*106/l, with prevalence of lymphocytes. e) all mentioned.

46

HEMORRHAGIC FEVER WITH RENAL SYNDROME 460. What is the source of the hemorrhagic fever with renal syndrome? a) human; b) birds; c) rodents; d) insects; e) fish. 461. What is the main factor of pathogenesis of the hemorrhagic fever with renal syndrome? a) immunosuppression; b) dehydration; c) toxicosis; d) allergic; e) All mentioned. 462. What type of exanthema is typical for hemorrhagic fever with renal syndrome (HFRS)? a) petechial; b) roseolar; c) urtical; d) macular and papular; e) vesicular. 463. Which morphological changes are typical for affection of kidney in patients with HFRS? (2) a) * focal hydronephrosis; b) glomerulonephritis; c) pyelonephritis; d) * nephroso-nephritis; e) urolithiasis. 464. What changes in common urine test are typical for HFRS? a) pyuria; b) albuminuria, hematuria; c) albuminuria, microhematuria, casts (hyaline, granular, epithelial); d) pyuria, albuminuria, casts (hyaline, granular); e) microhematuria, pyuria, casts (hyaline). 465. What complications are the typical for HFRS? a) polyneuritis; b) rupture of kidney capsule; c) renohepatic insufficiency; d) gastrointestinal bleeding; e) edema of the lung. 466. What complications are typical for HFRS? a) acute renal insufficiency; b) polyneuritis; c) acute renohepatic insufficiency; d) acute adrenal failure; e) edema of the brain. 467. What is the main mechanism of occurrence of rash in patients with HFRS? a) sedimentation of immune complexes in skins capillaries; b) carrying of infectious pathogens in skin lymphatic vessels and development there of inflammatory changes; c) toxico-allergic impairment of endothelial cells;

d) generalized destructive-proliferative thrombovasculitis; e) all mentioned. 468. Which one of the following combinations of human systems is the most likely injured with HFRS pathogen? a) lymphatic system and respiratory one; b) respiratory system and cardiovascular one; c) cardiovascular system and kidney; d) kidney and lymphatic systems; e) nervous system and digestive one. 469. What type of affection of kidney is typical for hemorrhagic fever with renal syndrome? a) nephrosclerosis; b) pyelonephritis; c) glomerulonephritis; d) nephrosonephritis; e) all mentioned. 470. Which of the following combinations of symptoms are characteristic for the climax period of hemorrhagic fever with renal syndrome? a) hepatomegaly and fever; b) oliguria and hemorrhagic rash; c) vomiting and cough with purulent phlegm; d) anuria and jaundice; e) diarrhea and vomiting. 471. What changing in urine analysis is typical for hemorrhagic fever with renal syndrome? a) hematouria, proteinuria, hyaline and epithelial casts; b) crystalluria; c) pyuria, hyaline casts; d) proteinuria, hematocyturia; e) pyuria. 472. What is the main specific laboratory test for diagnostic of HFRS? a) skin-allergic test; b) detection of antibodies with indirect immunofluorescent test; c) detection of virus in blood with enzyme immunoassays; d) contamination of laboratory animals (biological test); e) common blood test 473. What is the most typical complication for hemorrhagic fever with renal syndrome? a) edema of the brain; b) acute renal failure; c) meningoencephalitis; d) peritonitis; e) edema of lungs; 474. Which one of the following drugs is used for treatment of HFRS in climax period? a) Panangin; b) Furazolidone; c) Prednisolone; d) Delagil; e) Tymogen.

47

HERPESVIRAL INFECTION 475. Which two pathogens are not belong to group of Herpesviridae? a) Hantavirus; b) HSV-1 and HSV-2; c) Cytomegalovirus; d) Bunjavirus; e) Epstein-Barr virus; 476. Which one of following pathomorphological peculiarities is typical for herpetic encephalitis? a) formation of intracellular insertions (Negri bodies); b) myelinoclasis of nerves; c) hemorrhagic vasculitis; d) detachable neurons affection; e) disturbance of the synthesis of acethylcholine in synapses. 477. What way of transmittion is not typical for HSV-infection? a) contact; b) sexual; c) through the blood; d) transplacentar; e) perinatal. 478. What are the typical course and prognosis of the herpetic encephalitis? a) mild; b) moderate; c) severe; d) favorable; e) unfavorable; 479. What are the typical course and prognosis of the herpetic meningitis? a) mild; b) moderate; c) severe; d) favorable; e) unfavorable; 480. What is the main way of transmission of Varicella zoster-infection? a) through the blood; b) air-droplet; c) alimentary; d) sexual; e) through the bites of ticks. 481. The Epstein-Barr virus has tendency to affect: a) neurons; b) B-lymphocytes; c) T-lymphocytes; d) skin and mucous membranes; e) hepatocytes. 482. What are the two main antiherpetic drugs? a) Ribavirine; b) Acyclovire; c) Arbidole; d) Remantadine; e) Gancyclovire.

483. What is the main course of clinical activation of CMV-infection? a) vitamin insufficiency; b) overcooling; c) immunodeficiency; d) physical exhaustion; e) treatment with antibiotics. 484. CMV-infection affects all below mentioned cells except of: a) neutrophiles; b) monocytes; c) enterocytes; d) endocrine glands cells; e) B-lymphocytes. 485. Clinical manifestation of CMV-infection includes all below mentioned signs except of: a) retinitis; b) thyroiditis; c) pyelitis; d) hepatitis; e) pneumonia. 486. Clinical manifestations of CMV-infection includes all below mentioned signs except of: a) prolonged fever; b) endocarditis; c) enteritis; d) nephritis; e) meningitis. 487. What kind of antibody in high titers shows the affection of newborn with CMV-infection? a) IgA; b) IgG; c) IgM; d) IgE; e) IgF. 488. Which two drugs are the most effective for treatment of CMV-infection? a) Cricsivan; b) Foscarnet; c) Azidotimidine; d) Ganciclovire; e) Zovirax. 489. Human herpes virus HV-6 can be the agent of all mentioned diseases except of: a) exanthema of newborns; b) chronic fatigue syndrome; c) meningoencephalitis; d) B-cell lymphoma; e) all mentioned. 490. Which two symptoms from the mentioned below are not typical for herpes zoster-infection? a) fever; b) polyadenitis; c) acute pain syndrome; d) vesicular rush on the skin streaming along dermatome; e) herpetic tonsillitis. 491. What are the two indications for prescription of Acyclovire in case of herpetic infection? a) herpes of lips;

48

b) chicken pox; c) primary genital herpes; d) nasal herpes; e) herpetic encephalitis. 492. The Tsank cells, typical for herpes simplexinfection are: a) gigantic multinuclear cells with intranuclear insertion; b) cells with dark intranuclear insertions, encircled with enlightenment zone (owls eye); c) cells with cytoplasmic eosinophilic insertions; d) multinuclear syncytium with necrosis of the nuclei; e) multinuclear syncytium with bazophilic incertions. 493. Which method from mentioned is most acceptable for herpetic encephalitis diagnostics? a) clinical and epidemiological investigation; b) detection of specific IgG antibodies in blood; c) detection of viral DNA in the spinal fluid with PCR; d) computer tomography of the brain; e) all mentioned. 494. A 56-year old female under treatment for inoperable breast cancer develops a fever and painful vesicular, hemorrhagic rash on her right buttock. 1). The most likely diagnosis is superinfection with: a) Kaposhis sarcoma;

b) CMV-infection; c) erysipelas;

d) herpes-zoster-infection; e) meningococcal infection.

2). What is the main cause of the disease? a) vitamin insufficiency; b) overcooling; c) immunodeficiency; d) physical exhaustion; e) treatment with antibiotics 495. A 5-year-old child suffers from vesicular rush on the mucous of the mouth lusting for 3 weeks. Such rush was observed several times during lust year. What type of drug is most likely to alleviate this childs symptoms? a) Ribavirine; b) Acyclovire; c) Azidothymidine; d) Lamivudine;

e) Arbidole. 496. The mechanism of action of Acyclovire used to treat HS-infection involves; a) blocking of the active site of the cells DNA-dependent RNA polymerases; b) incorporation of a nucleoside analog into a virus DNA chain; c) induction of the synthesis and release of type I interferons; d) inhibition of the releasing of viral nucleic acid into the host cell; e) viral proteinase inhibition. 497. The Hants syndrome is manifested with: a) combination of facial nerves paralysis, pain and vesicular rush in the ear area; b) combination of back passage and waterworks disturbances with pain and vesicular rush in the anal and genital area; c) combination of acute pain in the mandibular area with vesicular rush on the mucous of the mouth. 498. Herpetic infection is prone to affect all mentioned organs and tissues except of: a) conjunctiva; b) liver; c) suprarenal glands; d) bone marrow; e) bronchi. 499. Varicellaformic Kaposhis eczema usually occurs: a) in HIV- infected persons; b) in aged persons; c) in children ill with eczema; d) in young persons with allergic diseases; e) in all age groups of persons ill with helminthiasis. 500. What drug is not used as etiological treatment of herpetic infection? a) Zoviracs; b) Cycloferone; c) Retabolile; d) Thebrofen ointment; e) all mentioned 501. A sexually active 22-year-old college student presents to the local clinic with a localized vesicular eruption on the shaft of his penis. A scraping of the base of one of the vesicles is positive for Tzank cells. The patient mentions that he had a similar eruption in the same area 2 month earlier. The reappearance of this eruption may be explained by: a) cell-mediated immunity (CMI) deficiency in
the patient; b) a prolonged period of viremia following the initial infection; c) a second infection with a similar virus with a different serotype;

49

d) failure of the patient to comply with therapy prescribed at the initial episode; e) * reactivation of a latent infection.

50

PLAGUE. TULAREMIA 501. What is the main way of transmission of bubonic form of plague? a) through the insects bites; b) alimentary transmission; c) air-droplet transmission; d) transplacentar transmission (from mother to fetus); e) through the contact with livestock; 502.What is the main way of transmission of cutaneus form of plague? a) through the insects bites; b) water way of transmission; c) air-droplet transmission; d) through the contact with patients; e) through the contact with livestock. 503. How long is incubation period of plague? a) 1-6 days; b) 3-7 days; c) 5-10 days; d) 7-14 days; e) 11-21 days; 504. How long is incubation period of tularemia? a) 1-6 days; b) 3-7 days; c) 5-10 days; d) 7-14 days; e) 11-21 days; 505. What is the reservoir of plague? a) cattle; b) wild carnivores; c) rodents; d) poultry; e) ticks. 506. Which one of the following forms of plague is localized one? a) secondary septic form; b) primary septic form; c) bubonic form; d) secondary pneumonic form; e) primary pneumonic form. 507. Which one of the following drugs may be used for urgent prevention of plague? a) Erythromycin; b) Penicillin; c) Polymyxin; d) Streptomycin; e) Vancomicin. 508. Which one of the following groups of lymph nodes is most likely affected in bubonic plague? a) cervical; b) inguinal; c) cubital; d) mesenteric; e) axillaries. 509. Which one of the following groups of lymph nodes is most likely affected in case of pharyngeal form of tularemia?

a) anterior cervical; b) inguinal; c) cubital; d) mesenteric; e) posterior cervical. 510. In which period of pneumonic form of plague the patients are most dangerous for other persons? a) incubation; b) initial; c) climax; d) convalescent; e) no one from mentioned. 511. What clinical form of plague is more often registered on a beginning of epidemic? a) primary pneumonic; b) bubonic; c) primary septic; d) cutaneus.. 512. What is the most often cause of death in case of septic form of plague? a) acute renal insufficiency; b) acute respiratory failure; c) septic shock; d) edema of the brain; e) edema of the lungs. 513. What is the first step in the management of patients with bubonic plague? a) prescription of immune globulin; b) local bubo therapy; c) detoxication; d) antibiotic therapy; e) dehydration therapy. 514. What is the main method of plague treatment? a) antibiotic therapy; b) serotherapy; c) detoxication therapy; d) dehydration therapy; e) rehydration therapy. 515. On examination of patient doctor has suspected a plague. Which one of the following facilities should be informed first? a) city center of state sanitary epidemiological surveillance; b) regional center of state sanitary epidemiological surveillance; c) head of current medical facility; d) police office; e) relatives of a patient. 516. What skin phenomena are typical for skin form of plague? a) There is on the hand painless ulcer 23 cm with a blackened necrotic eschar circled with elevated hyperemic rim, where small satellite vesicles, filled by serous-hemorrhagic contents are seen. Expanding zone of brawny edema, which jelly-like fluctuates during shaking, surrounds this affection.. b) There is ulcer 11 cm with hard yellowish bottom without purulent content at the skin of leg. Ulcer is circled by elevated intensively

51

hyperemic and sharply outlined rim with cyanotic shade. Ulcer is sharply painful. c) Moderate edema of skin. There is crater-like ulcer under gray crust on the highly infiltrated and hyperemic base in the center of edema; pus is excreted during pressing. Lymphangitis is present. Touching of ulcer is painful. d) Skin is markedly hyperemic, shining, smoothed; a zone of affection has precise festooned edges. On the center of affection hyperemia is less expressed, than on edges, where bullas with serous content are seen. 517. For treatment of plague we have use all antibiotics from mentioned below except of: a) Doxycycline; b) Chloramphenicole; c) Streptomycine; d) Cephasoline e) Gentamycine. 518.Wich two of the following methods of laboratory investigation may be used for express diagnostic of plague? a) culture of blood; b) * direct immunofluorescent test; c) detection of specific IgM antibodies with EIA (enzyme immunoassay); d) * bacterioscopy with Grams staining; e) detection of antibodies with indirect hemagglutination test (IHAT). 519. Clinical manifestation of primary pneumonic form of plague is characterized with: a) an acute onset of disease, severe signs of intoxication, sharp pain in the chest from the 2-3 days of disease, cough with abundant watery bloodish sputum, dyspnea; b) an acute onset of disease, moderate signs of intoxication, pain in the chest from the 3-5 days of disease, cough with scanty mucous rusty sputum, dyspnea; c) a gradual onset of disease, mild or moderate signs of intoxication, cough with scanty mucous bloodish sputum, dyspnea; d) an acute onset of disease, moderate signs of intoxication, pain in the chest from the 2-3 days of disease, rare dry cough, dyspnea; e) an acute onset of disease, expressed signs of intoxication, mild pain in the chest from the 46 days of disease, cough with abundant purulent sputum, dyspnea.

52

ANTHRAX 517. What is the source of the infection with anthrax? f) livestock; g) man; h) insects; i) river fish; j) birds. 518. What way of inoculation is most typical for anthrax? a) through respiratory tract; b) through injured skin; c) through gastrointestinal tract; d) through conjunctiva; e) all mentioned. 519. What kind of local inflammation is typical for anthrax? a) fibrinous; b) productive; c) serous-hemorrhagic; d) purulent; e) serous. 520. What is the content of anthrax carbuncle cavity? a) serous liquor; b) pus; c) blood; d) all mentioned. 521. What is the main factor of anthrax pathogenesis? a) autoallergy; b) immunosupression; c) toxemia; d) bacteremia; e) all mentioned; 522. What one of the following manifestations is characteristic for anthrax carbuncle? a) presence of carbuncle, local edema of tissue and acute pain; b) presence of carbuncle, spread edema of tissue, palpation is painless; c) local hyperemia and edema of the skin, fluctuation in the center and acute pain; d) spread hyperemia and edema of the skin, palpation is painful. 523. What is the main method of anthrax diagnostics? a) bacteriological; b) biological; c) serological; d) identification of toxin; e) skin allergic test. 524. Which one of the following manifestations is typical for anthrax regional lymphadenitis? a) painful lymph nodes and rapid purulence; b) painful lymph nodes and absence of purulence; c) painless lymph nodes and absence of purulence; d) painless lymph nodes and rapid prolipheration; e) formation of periadenitis.

525. What is the most typical variant of cutaneous form of anthrax? a) erysipeloid-like; b) edematous; c) bullous; d) carbunculous; e) bubonic. 526. What is the basic treatment of cutaneous anthrax? a) bandage with Vishnevskys ointment; b) antibacterial therapy; c) opening carbuncle and drainage; d) surgical revision of carbuncule; e) supportive therapy. 527. What is the drug of choose for treatment of anthrax? a) Streptomycine; b) *Penicilline c) Tetracycline d) Laevomycetine; e) Erhythromycine. 528. For treatment of cutaneus anthrax we may use all following means except of: a) Ceftriaxone; b) Penicillin; c) specific immunoglobulin; d) surgical removing of carbuncle; e) infusion of saline solutions. 528.What skin phenomena are typical for cutaneous form of anthrax? a) There is on the hand painless ulcer 23 cm with a blackened necrotic eschar circled with elevated hyperemic rim, where small satellite vesicles, filled by serous-hemorrhagic contents are seen. Expanding zone of brawny edema, which jelly-like fluctuates during shaking, surrounds this affection. b) There is ulcer 11 cm with hard yellowish bottom at the skin of leg without purulent content. Ulcer is circled by elevated intensively hyperemic and sharply outlined rim with cyanotic shade. Ulcer is sharply painful. c) Moderate edema of skin. There is crater-like ulcer under gray crust on the highly infiltrated and hyperemic base in the center of edema; pus is excreted during pressing. Lymphangitis is present. Touching of ulcer is painful. d) Skin is markedly hyperemic, shining, smoothed; a zone of affection has precise festooned edges. On the center of affection hyperemia is less expressed, than on edges, where bullas with serous content are seen

53

BRUCELLOSIS 527. What kind of Brucellae is most pathogenic for human? a) Br. abortus; b) Br. suis; c) Br. melitensis; d) Br. canis; e) Br. neotomae; 528. What factors can influence on duration of incubation period of brucellosis? a) pathogen activeness; b) quantity of infecting dose; c) way of transmission; d) reinfection; e) all mentioned. 529. Which one of the following tests reflects sensitivity of tissue to specific brucellar antigen? a) Coombss test; b) Wrights test; c) Heddlsons test. d) Burnets test. e) All mentioned. 530. What is migration of brucellae? a) transfer of brucellae from one organ to another of main host; b) transfer of brucellae from one animal to another of the same species; c) transfer of brucellae from main animal host to another of different species; d) all mentioned. 531. Portal of entry for brucellae into human organism is (2): a) dermal abrasions; b) gastrointestinal tract; c) mucous membranes of respiratory tract; d) urogenital tract e) all mentioned. 532. What laboratory test does allow to diagnose an acute stage of brucellosis?. a) Burnets test; b) culture of bone marrow aspirate; c) detection of specific IgM antibodies with EIA (enzyme immunoassay); d) detection of antibodies with indirect hemagglutination test (IHAT) in titer 1:80; e) all mentioned; 533. Does a cross-immunity after infection with different types of brucellae possible? a) yes; b) no. 534. What is the main reservoir of brucellosis? a) goats, sheep; b) humans; c) birds; d) rodents; e) cattle. 535. Which food products do usually serve as a source of brucellosis?

a) milk, cheese; b) salted fish; c) tinned meat; d) vegetables and fruit; e) eggs. 536. What are the basic clinical symptoms of climax period of acute brucellosis? a) protracted remitting fever, chills, significant sweating, polylymphoadenitis, muscular pain, enlargement of spleen and liver; b) constant type of fever, weakness, adynamia, pale skin, poor roseolar rush on the skin of abdomen, enlargement of spleen and liver; c) fever of remitting type, chills, significant weakness, petechial enanthema on conjunctiva, sometimes enlargement of liver; d) fever of intermitting type, arising of fever is associated with paroxysm of chills fallowed with feeling of heat and subsequent plentiful sweating; enlargement of spleen and liver; e) fever of hectic type, chills and profuse sweating, pustular rash, enlargement of spleen and liver. 537. The group of risk for brucellosis includes: a) meat handlers; b) veterinarians; c) milkmaids; d) cattle-breeders; e) * all mentioned. 538. The drugs of choice for treatment of brucellosis are the two from mentioned below: a) Penicillin; b) Erythromycin; c) Tetracycline; d) Streptomycin; e) Ceftriaxone. 539. Clinical manifestations of chronic brucellosis includes all mentioned below signs except of: a) arthritis; b) endometritis; c) meningitis; d) orchitis; e) * eczematous dermatitis. 540. Clinical variants of chronic brucellosis includes: a) locomotor form; b) urogenital form; c) nervous form; d) visceral form; e) all mentioned.

54

Q-FEVER 537. What is the drug of choice for treatment of Qfever? a) Penicillin; b) Tetracycline; c) Furazolidone; d) Streptomycin; e) Chloroquine. 538. Which cardiovascular disturbances are typical for climax period of Q-fever? a) tachycardia and hypotension; b) relative bradycardia and moderate hypotension; c) bradycardia and hypertension; d) tachycardia and hypertension; e) all mentioned. 539. How long is incubation period of Q-fever? a) up to 5 days; b) up to14 days; c) up to25 days; d) up to 45 days; e) up to 50days. 540. What is the basic method of laboratory diagnostic of Q-fever? a) culturing of blood; b) detection of specific antibodies in sera; c) skin-allergic test; d) culturing of feces; e) culturing of cerebrospinal fluid. 541. Which one of the following systems is typically affected with Q-fever? a) nervous system; b) cardiovascular system; c) respiratory system; d) alimentary system; e) all mentioned. 542. Causative agent of Q-fever is: a) Leptospira icterohemorrhagia; b) Rikettsia prowazechii; c) Coxiella burnetii; d) Rikettsia rikettsii; e) Chlamidia psittaci. 543. What is the reservoir of Q-fever? a) * cattle; b) rodents; c) ticks; d) humans e) birds. 544. Most human infections with Q-fever are acquired by (2): a) * inhalation; b) bytes of insects; c) * ingestion of contaminated food; d) direct contact with ill patients; e) direct contact with ill animals. 545. The main role in pathogenesis of Q-fever belongs to: a) * damaging of reticuloendothelial

tissue, ability of causative agent to multiply in macrophages; b) development of specific allergic reactions; c) systemic vasculitis; d) tropism of causative agent to tissue of liver and kidney; e) formation of necrotic foci in different organs. 546. X-ray signs of Q-fever-pneumonia are characterized with formation of: a) * patchy interstitial infiltrates; b) pleuritisl c) lobar consolidation of lung; d) abscesses; e) two-sided lobar pneumonia and pleuritisl. 547. Clinical classification of Q-fever includes all mentioned forms beside of: a) bronchopneumonia; b) typhoid-like form; c) meningoencephalitis; d) hepatitis; e) * nephrosonephritis 548. Q-fever is geographically distributed in a) South America; b) Europe; c) Asia and Africa; d) North America; e) * Worldwide. TOXOPLASMOSIS 542. What is the source of infection with toxoplasmosis? a) cats; b) birds; c) pigs; d) rabbits e) * all mentioned. 543. Who are the definitive hosts of toxoplasmosis? a) * cats; b) birds; c) cattle; d) rabbits; e) all mentioned. 544.Who are the paratenic hosts of toxoplasmosis? a) humans; b) rodents; c) pigs; d) birds; e) * all mentioned. 545. The two main ways of transmission of toxoplasmosis are those: a) air-droplet; b) * alimentary; c) through the bites of insects; d) * transplacental; e) through the skin/ 546. The group of risk for toxoplasmosis includes (2): a) aged persons; b) * patients with HIV-infection; c) smocked persons;

55

d) * persons receiving glucocorticoids; e) children under 1 year. 547. What is pseudocyst? a) It is oocyst, invaded epithelial cells of the cats intestine; b) It is the group of female and male gametes in the gut epithelial cells; c) * It is the group of infected macrophages containing numerous bradyzoites, surrounded by a thick wall; d) It is macrophages with intracellular tachyzoites; e) All mentioned. 548. What form of T. gondii does usually exist in infected organism of human in latent form of toxoplasmosis? a) oocysts; b) gametes; c) tachyzoites; d) *bradyzoits; e) sporozoites. 549. What is the drug of choice for treatment of toxoplasmosis? a) Chloridin (Pyrimethamin); b) Penicillin; c) Chloroquine (Delagil); d) Metronidazole (Trichopol); e) Laevomycetin. 550. Which combination of drugs is more effective for treatment of toxoplasmosis? a) Fansidar and Metronidazole; b) Pyrimethamin and Tetracycline; c) Fansidar and Spiromycin; d) Bisepthole and Clyndamycin; e) Chloroquine and tetracycline. 551. Toxoplasmosis usually causes in neonates such (2) forms of disease as: a) osteomyelitis; b) * meningoencephalitis; c) thyroiditis; d) glomerulonephritis; e) * chorioretimitis. 552. Which tests from the mentioned below may be used for diagnostics of toxoplasmosis? a) culturing of the blood; b) identifying of IgM and IgG antibodies with indirect fluorescent assay (IFA); c) microscopic examination of biopsy material Giemsas stained; d) PCR; e) * all mentioned. 553. Prevention of toxoplasmosis includes two effective measures: a) * avoiding of close contact with cats; b) boiling of water; c) * deep freezing of meat; d) deratization; e) killing of cats insects.

56

SEPSIS. 550. Choose possible causative agent of sepsis from following: a) Sh. sonnei; b) Str. pneumonie; c) Fr. tularence; d) R. prowazekii; e) M.pneumonie. 551. What is the main route of transmission of sepsis? a) air-droplet; b) alimentary; c) transplacentar; d) contact; e) activation of endogenous infection. 552. Is recovering from sepsis possible without proper treatment? a) Yes, due to formation of stable immunity; b) Yes, unfavorable outcome possible only among children and old persons; c) No; d) Yes, unfavorable outcome possible only in HIV infected persons. 553. Clinical sings of sepsis includes all mentioned below except of: a) petechial rash; b) jaundice; c) high long temperature; d) frequent liquid stool with admixture of mucus and blood; e) hepatosplenomegaly. 554. What method of laboratory investigation may be used as express one for diagnosing of sepsis? a) bacterioscopy of thick drop of Gram stained blood; b) culture of blood; c) serological methods; d) immunofluorescent test; e) skin allergic test. 555. In case of staphylococcal sepsis the most effective antibiotic is: a) Penicillin G in high dose; b) Cephalosporins of III generation; c) Vancomycin; d) Macrolids of III generation; e) Tetracycline. 556. The main complications of sepsis are all from mentioned below except of: a) ARDS; b) septic shock; c) acute renal failure; d) edema of the brain; e) dehydrative shock. 557. Refractory septic shock it is condition when: a) infusion therapy is not effective; b) complex antibiotic treatment is not effective; c) glucocorticoids are not effective; d) immune therapy is not effective. e) infusion therapy and glucocorticoids are not effective 558. Most mediators of inflammation act as:

a) activators of immune system; b) vasodilators; c) suppressors of immune system; d) factors increasing permeability of microvessels; e) vasodilators and factors increasing permeability of microvessels. 559. Which one from following immunocorrectors is most effective for treatment of sepsis? a) Timalin; b) Interferon; c) Ronkoleikin; d) Inductors of endogenous interferon (Amixin); e) T-activin. 560. Intravenous drug-users more frequently develop sepsis caused by: a) E. coli; b) Str. pneumonie; c) * S. aureus; d) S. enteritidisi; e) Ps. aeruginosa. good example is the best sermon.

57

TESTS AND CLINICAL CASES FOR 6TH YEAR STUDENTS

CLINICAL CASES CASE 1 The patient of 44-years-old has admitted to the hospital on the 10-th day of disease with complaints on common weakness, loss of appetite, jaundice, darkening of urine. The disease began gradually with fatigue, nausea. On the 4th day he has noted darkening of urine and the feeling of gravity in the right upper part of abdomen. Current state he had connected with exhaustion. On the 8th day of disease the jaundice has appeared and feces has lightened. On admitting: common state is moderate, temperature is 36,8C. Moderate jaundice of sclera and skin was observed. Pulse rate was 60 bites/min, BP 115/70 mm Hg. Tongue was coated with white fur. Liver was enlarged on 2 cm, its edge was acute and surface was smooth. Palpation of liver was slightly painful. a) What is the diagnosis? b) Make a differential diagnostics. c) Give a plan of laboratory examinations and treatment. d) What outcomes of disease are possible? CASE 2 Patient was admitted to the hospital on the 3-d day of disease after the high temperature (up to 39,0C), which was accompanied with chills, frontal headache and pain in muscle and joints within all this period. He complained also of difficulties of nasal breathing, serous secretion from the nose, irritating nonproductive cough and burning substernal discomfort. On examination temperature was 38,2C, injection of vessels of a sclera and eyelids conjunctiva and hyperemia of soft palate were observed. Isolated dry ronchi were listened on auscultation over the lung. a) What is most likely diagnosis? b) Make a differential diagnostic. c) Make the plan of laboratory examination. d) What urgent states are characteristic for a current disease? e) What medical tactics are preferable? CASE 3 A 39-years-old patient was admitted to the hospital on the 12th day of the disease. Disease started with severe chill followed by arising of temperature up to 39,5C, feeling of heat, headache and vomit. Trough the 6 hours temperature critically decreased up to 36,5C with massive sweating. Trough the one day all symptoms repeated and further such attacks repeated with the same periodicity. On examination: herpes of lips, enlargement of spleen and liver are presented, sclera are slightly icteric. The cardiac sounds are weak. Common

blood test presents moderate anemia, leucopoenia, lymphomonocytosis, and elevated ESR (21 mm /h). a) What is the diagnosis? b) Make a differential diagnostic. c) What are the possible urgent states during treatment? d) Give the plan of laboratory investigation. e) Prescribe a treatment. CASE 4 The 22-year-old student developed a disease with acute onset, febrile temperature, frequent liquid stool with admixture of slime and blood, spastic pain in law parts of abdomen. Week ago he has come back from village where the similar cases of disease had been registered. It is known that he has eaten unwashed apples there. On admission to the clinic on the 4-th day of disease a common state of patient was moderate. Temperature was 37,5C. The abdomen was moderately inflated, painful during palpation of thick sigmoid bowel. Frequency of stool numbered 10 times per day; it was poor, liquid, with slime and blood. a) Formulate the clinical diagnosis. b) Make a differential diagnostic. c) Make a plan of laboratory investigation. d) What are possible complications and urgent states during the current disease? CASE 5 A50-year-old watchman of a storehouse situated near a forest, developed disease with acute onset, high temperature (up to 39C) chills, headache, arthralgias, followed by back pain from the 2-th day of disease and by nasal bleeding from the 3-rd day. He was brought to the emergency room on the 5-th day of disease in severe state. On examination delirium, tremor of limbs, hyperemia of the face and neck and abundant hemorrhagic rush on skin of a trunk and extremities were noted. Diuresis numbered 150 ml. a) Make your preliminary diagnosis. b) Make a differential diagnostic. c) Make a plan of laboratory investigation. d) What urgent states did develop in this case? e) What is your medical tactic? CASE 6 The 25-years-old farmer presents on the 2-th day of disease high temperature, chills, headache, back pain and pain in calf muscles. On examination his state was moderate; hyperemia of face and sclera, slight jaundice, enlargement of liver were registered. Lien was palpated at an edge of a rib arc. Tongue was dry, coated with brown fur. The palpation of calf muscles was painful. Urine was dark and feces were of usual color. Diuresis numbered 600 ml. a) Make your preliminary diagnosis. b) Make a differential diagnostic. c) Make a plan of laboratory investigation. d) What urgent states did develop in this case? e) What is your medical tactic?

58

CASE 7 The inhabitant of the Tyumen region (Russia, West Siberia) used to eat a row fish and a home made salted pike caviar within last months noted common weakness, dizziness, burning of tongue, pain in muscles, loss of weight and discharging with a feces particles of helminthes. The skin of patient is pale; shin and feet are edematous. The cardiac sounds are weak, systolic murmur on the apex of heart is listened. BP is 100/70 mm Hg. The liver is enlarged on 2 sm. Tongue is brightly red, with cracks. CBT: erythrocytes 1,3 1012/l, hemoglobin 50 g/l, CI 1,2; in sample megalocytes, poikilocytes, erythrocytes with Jolis corpuscles and Kebots rings are present. a) What is most likely diagnosis? b) Make diagnostics and treatment plan. c) What complication did develop in this case? CASE 8 A 45-year-old train conductor was brought to emergency department on the 7th day of disease with the signs of acute respiratory failure. The disease began gradually from sore throat, temperature of 37,8-38C and weakness. Then nonproductive cough, hoarseness of a voice developed followed with aphonia, difficult and noisy labored respiration. On admission the patients state is severe. Cyanosis, stridoric breathing with participation of auxiliary muscles sweating, paroxysmal tachycardia were noted. Respiratory rate was 52/min. Cardiac sounds were dummy. In oropharyngeal region cyanosis, edema and hyperemia of a soft palate and tonsils were registered; tonsils were covered with firm grey membranes. During laryngoscopy membranes of grey color were detected on a mucous of larynx and then extracted. a) What is most likely diagnosis? b) Make diagnostics and treatment plan. c) What complication did develop in this case?

provoked such convulsive attacks. Pulse rate-160 beats/min. BP 140/100 mm Hg. a) Make your diagnosis. What is the phase of the disease? b) Make a differential diagnostic. c) Make the plan of laboratory investigation. d) What is your medical tactics? CASE 10 A 11-year-old pupil of school was admitted to the hospital on the 4th day of disease with complaints on lack of motions in the lower extremities and sharp pain in their muscles, high temperature, which began to reduce. With drop of temperature the pain in the lower extremities, loins, arms and neck has aggravated. Within several hours after admission the flaccid paralyses have appeared in the muscles of legs, arms with a dominance of a proximal affection of extremities (femur, shoulders). The function of sphincters is reduced. The hypotension of muscles, absence of reflexes is followed with coldening and cyanosis of legs. The sensitivity is saved. a) Make your diagnosis accounting phase of disease. b) Make the differential diagnosis. c) Make the plan of laboratory diagnostics. d) What complications are possible? e) Your therapeutic tactics? CASE 11 A 48-year-old wool-sorter was admitted to clinic on the 2d day of disease in severe condition with complaints on dyspnea, expectoration of hemorrhagic sputum (about 1 l/day), and pain in the chest, connected with breathing. The disease began suddenly with chill, weakness and cough, at first dry and then with expectoration of a great amount of serous and seroushemorrhagic sputum. It is known that during last time he accepted and unpacked bales with a wool delivered from other countries. Among the employers cases of influenza were registered. On examination: general condition of patient was severe, consciousness was saved, but sometimes patient was inadequate. Skin was cyanotic and wet. A mucous of nasopharyngeal region was slightly hyperemic. Tongue was dry, coated with dirty fur. Respiratory rate accounted 40/min. On percussion over the lungs in interscapular space shortening of a pulmonary sound was registered, on auscultation a lot of dry and varied wet rales were listened. Pulse rate was 120 beats/min, blood pressure 80/40 mm Hg. Spleen was palpated at edge of a rib arc. a) What is preliminary diagnosis? b) Make a differential diagnostic. c) Make the plan of laboratory investigation. d) What complications are typical for the disease? CASE 12 A 18-year-old patient presents on the 8th day of the disease complaints on common fatigue,

CASE 9 A 36-year old track driver was brought to emergency room on the 4th day of disease in a severe condition with complaints on headache, sleeplessness and impossibility to swallow liquids (liquid food, water and even saliva). It was found out that at last month on hunting he has caught the fox, brought it to home, but fox has scratched him and then escaped. On examination: there were no bites, but multiple tracks of scratches on the hands. The persons face was hyperemic, with injection of scleras vessels; exophthalmia, hypersalivation and an abundant sweating were noted. Movements with the lower extremities were restricted. Periodic painful convulsive spasms of throat and larynx muscles were registered, accompanied with stenotic breathing and expression of horror. The bright light, noise of pouring water, movement of air or attempt of swallowing of saliva

59

headache, pain in throat during swallowing, high temperature, sweating. The disease began with malaise, subfebrile temperature, and sore throat. Frequent attacks of tonsillitis in past history were noted. On examination: temperature 37,7C. Skin is pale without a rush. Mucous of throat is moderately hyperemic. The tonsils are enlarged in size, edematous, without purulent membranes. Axillary, submandibular, neck posterior lymph nodes are enlarged in size (0,8 1 cm), slightly painful, mobile. Cardiac sounds are clear, rhythmic and pulse rate is 94 beats/min. On auscultation over the lung vesicular sound is listened. Tongue is wet, coated with white fur. Abdomen is soft, painless. The liver enlarged on 1 cm. a) What is preliminary diagnosis? b) Make differential diagnostic. c) Make the plan of laboratory investigation. d) What complications are possible? e) What is your medical tactics? CASE 13 A 30 year-old nurse of a summer children camp was admitted to hospital on the 3d day of the disease with the complaints on pain in throat, high temperature and frequent liquid stool. Nine teenagers from summer camp with similar symptoms were hospitalized in department. Bright hyperemia of mucous of throat and soft palate, palatine arcs, tonsils and back wall of a pharynx were detected. There are a group of shallow vesicles with transparent contents, enclosed by a red crown on a mucous of soft palate, which in 2-3 days burst with formation of erosion. The sore throat was strengthened during swallowing. On the 7th day the fever has decreased, epithelizationof erosions was observed. General condition was improved, stool normalized. a) What is most likely diagnosis? b) Make the differential diagnosis. c) Make the plan of laboratory diagnostics. d) What is your medical tactic? CASE 14 A 38-year-old female patient developed disease with acute onset, arising of temperature up to 38C, headache, weakness and nausea. To the end of the 3d day pain in left eye, swelling of eyelids with purulent discharging from eye has appeared followed with swelling and pain in the field of left ear and neck. It is known that last 2,5 weeks she worked with thresher. On examination: skin of face is hyperemic, dry. Pulse is rhythmic, 90 beats/min. Eyelids of the left eye are edematous, in the angle of left eye white pus is seen. The conjunctiva is sharply edematous; there are plenty of small ulcers with purulent content here. Asymmetry of the face and neck at the expense of enlarged subauricular and posterior neck lymph nodes is seen. Lymph nodes are painful, tough, with clear contours and movable. Mucous of throat is moderately hyperemic. Tonsils are not enlarged.. Tongue is dry, coated with fur. Abdomen is soft, painless during palpation. Liver and lien are not palpated.

The CBT reveals neutrophilic leucocytosis; ESR is 25 mm/h. a) What is most likely diagnosis? b) What are the methods of laboratory diagnostics and therapy? CASE 15 A 44-years-old female patient was admitted to the hospital at the end of May with the complaints on a strong headache of pulsating character, photophobia, general muscle pains, chill, fever, sleeplessness, lack of appetite. It was found out that 8 days before she has come back from Western Siberia, where she had lived in village near wood during 10 days. She has neither contact with ill people, animals, nor bites of mosquitoes and ticks. On examination on the 3d day of disease: temperature was 39,0C, consciousness was clear, pulse was rhythmic, 80 beats/min, the cardiac sounds were muffled. Over the lung vesicular breathing was listened. Abdomen was soft and painless on palpation. Mucous of the throat was hyperemic. A left-sided neck lymphadenitis was present. In left parietal part of the head a crust of brown color on edematous base painful on palpation was seen. Slight neck rigidity was present. Reflexes of pupils on light were week, convergence was broken, mild asymmetry of the face was noted. a) What is most likely diagnosis? b) Make the plan of differential diagnostic. c) What is your therapeutic tactics? CASE 16 A 40-year-old metal-turner on admission to the hospital presents complains of headache, strong weakness, fever, pain in joints, itching abundant rush on skin of body. One week ago he has wounded the right foot, and received prophylactic treatment with tetanus antitoxin. On examination on the 7th day of the disease: temperature is 38,2C. There is macular and papular itching rash on the skin of trunk, arms, abdomen and lower extremities. Face and fingers of hands are edematous and pale. Disseminated lymphoadenopathy is found. Tachycardia and hypotension are registered. In CBT: leucopoenia with a relative lymphocytosis and accelerated ESR. a) What is most likely diagnosis? b) Make the plan of differential diagnostic. c) What is your therapeutic tactics? CASE 17 A divorced mother of single test positive for HIV-1 infection during investigation of a febrile illness presents disseminated lymphoadenopathy. A second enzyme immunoassay (EIA) is performed, and the result is the same. The woman denies intravenous drug use. She has dated several men since her divorce and

60

cannot be positive about their sexual habits or use of intravenous drugs. a) What is most likely diagnosis

b) What is the appropriate next step in the management of this patient? c) Make a plan of differential diagnostic.

CASE 18 A 32-year-old patient ill with severe acute tonsillitis was brought to the hospital and treatment with Penicillin and Sulfodoxine was undertaken. Soon the patient felt a numbness of tongue, lips, face; giddiness, headache, nausea, shortness of breath has appeared. On examination: patient is excited, his face is hyperemic, sweated; cyanosis and macular inching rush are present. Pulse is rapid and thread-like. BP 40/0 mm Hg. Cardiac sounds are dummy, the 2nd tone on a pulmonary arteria is accentuated. The respiration is frequent and superficial. Over the lungs box-like pulmonary sound, and numerous dry whistling rales were found. Involuntary urination and defecation were noted. a) What is most likely diagnosis? b) Make a treatment plan? CASE 19 A 37-year-old veterinarian within a month is complaining of weakness, sweating, pain in knees, foot joints and subfebrile temperature. Repeatedly sought for medical help, was treated with antipyretics but without effect. In the farmer where he works cases of abortions in sheep were registered On examination: a common patients state is satisfactory, temperature is 37,6 C. Skin and visual mucous are of usual color. Large joints are edematous and painful on palpation and motion; there are dense painful nodules in a hypodermic fat near joints, tendons and muscles. The cardiac sounds are rhythmic, dull. Pulse rate is 80 /min, BP 120/80 mm Hg. Abdomen is soft and painless on palpation. The edge of a liver is palpated on 1,5 cm below of a rib arc. The lien is not enlarged. Stools and urine passage are normal. a) Make your presumable diagnosis. b) Make a plan of differential diagnostic. c) Make a plan of laboratory diagnostic. CASE 20 A 21-year-old schoolteacher developed acute disease with fever up to 38,5C, running nose, nonproductive cough, photophobia and lacrimation. On the 3 rd day temperature increased to 39.5C, headache and nausea intensified and patients condition deteriorated; on the next day rush stood out on the face, neck, upper part of thorax and then spread on whole body. On examination on the 5th day: temperature is 39.5C, patient is languid, adynamic. Face and eyelids are

hyperemic and edematous. Conjunctives are hyperemic, injection of scleras vessels is observed. There is abundant macular and papular rush sometimes merged on the skin of body and face. Throat and palatal mucous is hyperemic; on soft palate enanthema is seen. On chick mucous, opposite molars, small spots of white fur are observed. Disseminated lymphoadenopathy is found. During auscultation rough breathing is listened. Cardiac sounds are moderately dull, pulse rate is 92/min.. a) What is your diagnosis? b) Make a differential diagnosis c) What complications are possible? a) First step treatment measures.

CASE 21 A 61-year-old pensioner within 5 days presents complaints on headache, insomnia, chill, fever 39.3C, and weakness. Treatment with antipyretics was not effective. Today on skin of trunk roseolar and petechial rush come out. It is known that 35 years ago he had been ill with louse-borne typhus and malaria and now he suffers from diabetes. On examination: patient is slightly excited, his face is edematous and hyperemic, eyes glitter. There is enanthema on soft palate. Not numerous roseolar and petechial rush predominantly on the lateral parts of trunk and internal surfaces of upper extremities is seen. Moderate dyspnea is present. On auscultation over the lung rough breathing is listened. Cardiac sounds are dull, systolic murmur on top projection. BP is 100/60 mm Hg. Tongue is coated with fur, trembling during examination. Abdomen is soft, painless, liver and spleen are enlarged on 2 cm. a) What is the diagnosis? b) Make differential diagnostic. c) What complications are possible? d) Make plan of laboratory diagnostics and treatment. CASE 22 A 35-year-old milkmaid within 2 weeks presents remitting fever with every day chills and then profuse sweating, headache, weakness, and pain in joints. Paroxysms of temperature appeared in the afternoon. Treatment with penicillin and antipyretics was not effective. Despite of high temperature, common condition of the patient remained rather satisfactory. It is known that during last year she has contact with ill animals during calving. On examination: temperature is 38.5C. Skin is slightly cyanotic, without rush. Cervical and axillary lymphoadenopathy is present. Over the lungs vesicular breathing is listened. The cardiac sounds are muffled and rhythmic. Pulse rate is 80 beats/min. Tongue is wet and pure. Abdomen is soft and painless; edge of a liver is palpated on 1m below of rib arc. The lien is not palpated. Stools and waterworks is normal a) What is most likely diagnosis?

61

b) Make a differential diagnostic. c) Make the plan of laboratory diagnostic and treatment. CASE 23 A 25-year-old hunter was brought to the emergency department in a severe state on the 3rd day of disease. Onset of the disease was acute with chill, high temperature (up to 39.0-39.5C), headache, vomiting, pain in muscles, and pain in right inguinal region followed with hallucinations. Patient is accurately vaccinated. According to his work, he most part of time spent in a wood and steppe. On examination: temperature is 39C, pulse is weak, 140 beats/min, BP 80/40 mm Hg. Patients consciousness is confused. Skin of face is hyperemic and cyanotic. Cardiac sounds are dull, respiratory rate account 26/min. Over the lung vesicular respiration is listened. Tongue is dry and coated with white fur. The abdomen is soft, painless. The right leg is half bent. In inguinal area the formation with indistinct contours of 15 25 cm in size is visually defined. Skin above it is of blue-crimson color. During palpation of this conglomerate the patient reacts with a groan, tries to repel a hand of the doctor. a) What is the most likely diagnosis? b) What urgent state was developed in this case? c) Make your tactics of treatment.

department in severe state. Within 7 days he complained of fever, sharp headache, and chills. Temperature was high (39,0-40,0C) and only sometimes on mornings was normal. Patient was taking Analgin but had no effect: the headache accrued, sleepiness appeared, and then exaltation, unconsciousness state, and cramps developed. It was found out that month ago he has been ill with malaria. On examination: patients state is severe. Unconsciousness, mild neck rigidity and periodical tonic and clonic seizures are present. Reflexes of pupils on light are week; miosis, increasing of tendinal reflexes, and Chain-Stokss breathing are observed. Skin and visual mucous are subicteric. Pulse is weak, 130 beats/min, BP 90/40 mm Hg. On auscultation of lungs heterogeneous wet crackles are listened. The lien is illegibly palpated. a) What is the most likely diagnosis? b) What agent does cause this disease? c) What is the form of disease? d) What is your therapeutic tactics?

CASE 24 A 21-year-old patient within 7 days presents complaints on chill, febrile temperature, headache, common weakness, and sore throat. Treatment with Tetracycline and Aspirin was not effective. On examination: Patient was sluggish. Skin was pale, without rush. Mucous membrane of a throat is hyperemic; tonsils are enlarged and covered with membranes of green-yellow color. Axillary, anterior and posterior neck lymph nodes were enlarged, slightly painful during palpation. Pulse was rhythmic, 108 beats/min. The cardiac sounds are dull. Harsh breathing is listened on auscultation of the lung. Tongue is dryish, coated with white-grey fur. Abdomen is soft, slightly painful in right part of epigastria on palpation. Lower edge of liver is palpated on 2 cm below of a rib arc. Spleen is palpated at a level of a rib arc. The CBT shows: erythrocytes 4 1012/l, leukocytes 10 109 /l, drum stick 8 %, segmented neutrophiles 32%, lymphocytes 51 %, monocytes 6 %, plasmatic cells 3%. ESR 27 mm/h. Some lymphocytes have wide and vacuolated cytoplasm. a) What is the most likely diagnosis? b) What is your therapy?

CASE 26 A 26-year-old patient within 3 days developed weakness, temperature of 38C, followed with difficulty of swallowing of a saliva and water and hypersalivation. It was found out, that 4 months ago during hunting he has been bitten by the fox in the right hand and has not been treated with vaccine. On examination: patients state is severe, temperature is 38.8C, face is hyperemic and sweaty. In a reception ward at the attempt to swallow water attack of painful spasms of pharyngeal and laryngeal muscles developed, that was accompanied by excitement and disorders of respiration. With cry throws from itself a cup of water, throws back head and trunk. The face is distorted, cyanotic, expresses horror, the pupils are dilated, view stuck in one point. The attack lasted some seconds. Then the attack has repeated after a loud sound and switching of light. Lethal outcome had occurred during the next fit. a) What is most likely diagnosis? b) What is the period of diseases? c) What medical help was need in this case? CASE 27 A 35-year-old migrant worker is admitted to the hospital with a high fever and malaise on the 8 day of disease. When he started feeling weak he checked his temperature and found it to be slightly elevated. Over the last 4 days he has felt progressively worse and his

CASE 25 A 35-year-old businessman, which week ago has come back from trip in Guinea, was brought to the emergency

temperature has reached 39 C. The resident on call in the emergency room observes a scanty roseolar rash on patients trunk. The patient mentions that another

62

worker in his group has been sick wit similar symptoms. a) What is most likely diagnosis? b) Make a plan of differential diagnostic. c) Make a plan of laboratory investigation and treatment. CASE 28 A 18-year-old male was admitted to the hospital on the 3rd day of the disease. On the 1st day his temperature was 37.8C, and he felt irritation in the throat, sneezing and mucous secretion from the nose. On the 2nd day temperature arose up to 38.8C and strong headache, increasing to night and single vomiting appeared. Next morning patient developed cramps and became disoriented. On examination: patients common state is grave; productive contact with him is impossible. Meningeal symptoms are expressed. During examination patient pushes away doctors hand. He periodically develops clonic and tonic cramps. His face is edematous and hyperemic. Moderate hyperemia of posterior wall of a throat is seen. Pulse rate is 120 beats/min, BP 140/90 mm Hg, respiratory rate 30/min. a) What is the most likely diagnosis? b) What is the level of consciousness disorder? c) What complication has develop in this case? CASE 29 A 30-year-old male was admitted on the 2nd day of the disease at night after acute beginning of disease with arise of temperature up to 37.8C and general weakness, with marked worsening of the general state to the evening: temperature arose up to 39C, severe headache, vomiting, absence of effect from analgetics, disorders of conscious. a) What is the most likely diagnosis? b) What is the level of consciousness disorder? CLINICAL TASKS 1. Chose symptoms, typical for icteric form of viral hepatitis B: a) Gradual onset from nausea, gradual loss of appetite, increasing itching of skin. OE: jaundice of skin, with scratches traces. Skin turgor is reduced. Pulse is 86 /min; sizes of liver by Kurlov: 13128 cm, in gall-bladder projection elastic painless formation 33 cm is palpated. Spleen is not enlarged. b) Acute onset from nausea, pain in a right part of abdomen, increasing of fever up to 38C. OE: moderate jaundice; tachycardia. Sizes of liver by Kurlov: 12127 cm., palpation in gallbladder projection is painful. Spleen is not enlarged. c) Gradual onset from nausea, pain in joints, itch of skin, and loss of appetite. OE: jaundice of skin, its turgor is normal. Pulse is 68 /min.

Sizes of liver by Kurlov: 13128 cm., edge of liver is sensitive on palpation. Spleen is not enlarged. d) Acute onset with chill, fever up to 38,8C, nausea, dizziness, weakness. OE: moderate jaundice, temperature is 39C. Tachycardia up to 122 /min. Sizes of liver by Kurlov: 11108 cm., its edge is painless. Spleen is palpated on 3 cm. below left costal arch. Anuria within 10 hours. 2. Chose symptoms, typical for cholelitiasis: a) Gradual onset from nausea, gradual loss of appetite, increasing itching of skin. OE: jaundice of skin, with scratches traces. Skin turgor is reduced. Pulse is 86 /min; sizes of liver by Kurlov: 13128 cm, in gall-bladder projection elastic painless formation 33 cm is palpated. Spleen is not enlarged b) Acute onset from nausea, pain in a right part of abdomen, increasing of fever up to 38C. OE: moderate jaundice; tachycardia. Sizes of liver by Kurlov: 12127 cm., palpation in gallbladder projection is painful. Spleen is not enlarged. c) Gradual onset from nausea, pain in joints, itch of skin, and loss of appetite. OE: jaundice of skin, its turgor is normal. Pulse is 68 /min. Sizes of liver by Kurlov: 13128 cm., edge of liver is sensitive on palpation. Spleen is not enlarged. e) . Acute onset with chill, fever up to 38,8C, nausea, dizziness, weakness. OE: moderate jaundice, temperature is 39C. Tachycardia up to 122 /min. Sizes of liver by Kurlov: 11 108 cm., its edge is painless. Spleen is palpated on 3 cm. below left costal arch. Anuria within 10 hours. 3. Chose symptoms, typical for tumor of head of pancreas: a) Gradual onset from nausea, gradual loss of appetite, increasing itching of skin. OE: jaundice of skin, with scratches traces. Skin turgor is reduced. Pulse is 86 /min; sizes of liver by Kurlov: 13128 cm, in gall-bladder projection elastic painless formation 33 cm is palpated. Spleen is not enlarged b) Acute onset from nausea, pain in a right part of abdomen, increasing of fever up to 38C. OE: moderate jaundice; tachycardia. Sizes of liver by Kurlov: 12127 cm., palpation in gallbladder projection is painful. Spleen is not enlarged c) . Gradual onset from nausea, pain in joints, itch of skin, and loss of appetite. OE: jaundice of skin, its turgor is normal. Pulse is 68 /min. Sizes of liver by Kurlov: 13128 cm., edge of liver is sensitive on palpation. Spleen is not enlarged d) Acute onset with chill, fever up to 38,8C, nausea, dizziness, weakness. OE: moderate

63

jaundice, temperature is 39C. Tachycardia up to 122 /min. Sizes of liver by Kurlov: 11108 cm., its edge is painless. Spleen is palpated on 3 cm. below left costal arch. Anuria within 10 hours.. 4. What pathogenetic mechanisms are typical for cholera? (2) a) Dehydration, hypovolemia, arterial pressure decreasing, reduction of renal filtration; b) Toxemia, disturbance of peripheral microcirculation; c) Dehydration, inflammatory changes in intestine, metabolic acidosis, lack of potassium in blood; d) Increasing of water and electrolytes secretion as a result of activation of adenilatcyclaze and accumulation of cAMP. 5. What basic clinical symptoms are characteristic for colitic form of shigellosis? a) Gradual onset, absence of intoxication, moderate spastic pains in lower part of abdomen, alternation of constipation and diarrhea, sometimes mucus in feces; b) Acute onset with sharp pain in abdomen, symptoms of intoxication at first are absent; stool is bloody, frequently without feces; c) Acute onset, moderate intoxication, diffuse pain in abdomen, vomiting, then liquid stool with admixture of mucus (and sometimes of blood); d) Acute onset, moderate intoxication, spastic pain in left hypogastric region, scanty stool with mucus and blood. 6. What are the basic clinical symptoms of cancer of rectum? a) Gradual onset, absence of intoxication, moderate spastic pains in lower part of abdomen, alternation of constipation and diarrhea, sometimes mucus in feces; b) Acute onset with sharp pain in abdomen, symptoms of intoxication at first are absent; stool is bloody, frequently without feces; c) Acute onset, moderate intoxication, diffuse pain in abdomen, vomiting, then liquid stool with admixture of mucus (and sometimes of blood); d) Acute onset, moderate intoxication, spastic pain in hypogastric region, scanty stool with mucus and blood. 7. What are the basic clinical symptoms of thrombosis of mesenteric vessels? a) Gradual onset, absence of intoxication, moderate spastic pains in lower part of abdomen, alternation of constipation and diarrhea, sometimes mucus in feces; b) Acute onset with sharp pain in abdomen, symptoms of intoxication at first are absent; stool is bloody, frequently without feces; c) Acute onset, moderate intoxication, diffuse pain in abdomen, vomiting, then liquid stool with admixture of mucus (and sometimes of blood);

d) Acute onset, moderate intoxication, spastic pain in left hypogastric region, scanty stool with mucus and blood. 8. What are the basic clinical symptoms of climax period of malaria vivax? a) Remittent fever, chills, significant sweating, lympadenopathy, muscular pain, enlargement of spleen and liver; b) Constant type of fever, weakness, adynamia, pale skin, scanty roseolar rush on lateral surfaces of trunk, enlargement of spleen and liver; c) Fever of remitting type, chills, significant weakness, petechial enanthema on conjunctiva, sometimes liver is enlarged; d) Fever of intermitting type, on climax of fever feeling of heat with subsequent plentiful sweating, enlargement of spleen and liver.. 9. What are basic clinical symptoms of climax period of acute brucellosis? a) Remittent fever, chills, significant sweating, lympadenopathy, muscular pain, enlargement of spleen and liver; b) Constant type of fever, weakness, adynamia, pale skin, scanty roseolar rush on lateral surfaces of trunk, enlargement of spleen and liver; c) Fever of remitting type, chills, significant weakness, petechial enanthema on conjunctiva, sometimes liver is enlarged; d) Fever of intermitting type, on climax of fever feeling of heat with subsequent plentiful sweating, enlargement of spleen and liver.. 10. What are basic clinical symptoms of climax period of staphylococcal sepsis? a) Remittent fever, chills, significant sweating, lympadenopathy, muscular pain, enlargement of spleen and liver; b) Constant type of fever, weakness, adynamia, pale skin, scanty roseolar rush on lateral surfaces of trunk, enlargement of spleen and liver; c) Fever of remitting type, chills, significant weakness, petechial enanthema on conjunctiva, sometimes liver is enlarged; d) Fever of intermitting type, on climax of fever feeling of heat with subsequent plentiful sweating, enlargement of spleen and liver. 11. What symptoms are most typical for staphylococcal food poisoning? a) Acute onset with chill, vomiting, short-term fever, diffuse abdominal pain, frequent liquid stool with greenish stool up to 15-20 times per day; b) Acute onset with chill, repeated vomiting, sharp pains in epigastria, frequent development of collapse, liquid stool up to 4-5 times per day, fast convalescence;

64

c) Acute onset with liquid watery stool, then vomiting; normal body temperature, absence of abdominal pain, oliguria; e) Gradual onset with long high fever, increasing weakness, pale skin, from the 2-d week of disease liquid stool up to 2-4 times per day. 12. What symptoms are most typical for gastrointestinal salmonellosis? a) Acute onset with chill, vomiting, short-term fever, diffuse abdominal pain, frequent liquid stool with greenish stool up to 15-20 times per day; b) Acute onset with chill, repeated vomiting, sharp pains in epigastria, frequent development of collapse, liquid stool up to 4-5 times per day, fast convalescence; c) Acute onset with liquid watery stool, then vomiting; normal body temperature, absence of abdominal pain, oliguria; e) Gradual onset with long high fever, increasing weakness, pale skin, from the 2-d week of disease liquid stool up to 2-4 times per day. 13. What symptoms are most typical for moderate form of cholera? a) Acute onset with chill, vomiting, short-term fever, diffuse abdominal pain, frequent liquid stool with greenish stool up to 15-20 times per day; b) Acute onset with chill, repeated vomiting, sharp pains in epigastria, frequent development of collapse, liquid stool up to 4-5 times per day, fast convalescence; c) Acute onset with liquid watery stool, then vomiting; normal body temperature, absence of abdominal pain, oliguria; e) Gradual onset with long high fever, increasing weakness, pale skin, from the 2-d week of disease liquid stool up to 2-4 times per day. 14. What symptoms are typical for typhoid fever? a) Acute onset with fast increasing of temperature within 1-2 days up to 39-40C; persistent headache, sleeplessness, euphoria. Face is hyperemic, sclera are injected, spleen is increased since the 3-4 days from onset of disease; b) Gradual onset with slow (within 4-5 days) increasing of body temperature, general weakness, sleeplessness, pale skin; tongue is thickened, with teeth prints, relative bradycardia, spleen is enlarged since the 6-7 days from onset of disease; c) Acute onset; within several hours body temperature reaches 38-39C, is accompanied with chills, headache in forehead and temple area, pain in eyes, nasal obstruction, dry cough, tracheal pain; spleen is not enlarged; d) Acute onset with chill, fast increasing of temperature up to 39-40C; weakness, sweating, myalgias, excitation, headache, sometime dry cough, hyperemia of face, injection of sclera vessels, retro

orbital pain, sometimes palatal enanthema, enlargement of spleen is quite often. In WBC leucopoenia, neutropoenia. 15. What symptoms are typical for initial period of louse-born typhus? a) Acute onset with fast increasing of temperature within 1-2 days up to 39-40C; persistent headache, sleeplessness, euphoria. Face is hyperemic, vessels of sclera are injected, spleen is enlarged since the 3-4 days from onset of disease, after 4-5 days occurrence of roseolar-petechial rash on trunk. b) Gradual onset with slow (within 4-5 days) increasing of body temperature, general weakness, sleeplessness, pale skin; tongue is thickened, with teeth prints, relative bradycardia, spleen is enlarged since the 6-7 days from onset of disease; c) Acute onset; within several hours body temperature reaches 38-39C, is accompanied with chills, headache in forehead and temple area, pain in eyes, nasal obstruction, dry cough, tracheal pain; spleen is not enlarged; d) Acute onset with chill, fast increasing of temperature up to 39-40C; weakness, sweating, myalgias, excitation, headache, sometime dry cough, hyperemia of face, injection of sclera vessels, retroorbital pain, sometimes palatal enanthema, enlargement of spleen is quite often. In WBC leucopoenia, neutropoenia. 16. What symptoms are typical for initial period of acute Q-fever; a) Acute onset with fast increasing of temperature within 1-2 days up to 39-40C; persistent headache, sleeplessness, euphoria. Face is hyperemic, vessels of sclera are injected, spleen is increased since the 3-4 days from onset of disease; b) Gradual onset with slow (within 4-5 days) increasing of body temperature, general weakness, sleeplessness, pale skin; tongue is thickened, with teeth prints, bradycardia, spleen is enlarged since the 6-7 days from onset of disease; c) Acute onset; within several hours body temperature reaches 38-39C, is accompanied with chills, headache in forehead and temple area, pain in eyes, nasal obstruction, dry cough, tracheal pain; spleen is not enlarged; d) Acute onset with chill, fast increasing of temperature up to 39-40C; weakness, sweating, myalgias, excitation, headache, sometime dry cough, hyperemia of face, injection of sclera vessels, retro orbital pain, sometimes palatal enanthema, enlargement of spleen is quite often. In WBC leucopoenia, neutropoenia.. 17. Choose symptoms, typical for diphtheria: a) Acute onset, general headache, pain in throat, fever 38 -40C, disseminated lymphoadenopathy, mucous of throat is hyperemic; there are easy removed white-yellow purulent membranes in lacunas of tonsils; enlargement of spleen; b) Acute beginning, strong headache, pain in throat on swallowing, fever 39-40C, mucous of

65

throat is hyperemic; there are easy removed grayyellow purulent membranes in lacunas of tonsils; jugular lymph nodes are increased and sharply painful; c) Acute beginning, moderate pain in a throat, fever up to 38C, mucous of throat is hyperemic, there are white colored, hardly removed membranes on tonsils; slightly increased and moderately painful jugular lymph nodes; d) Acute onset, fever up to 38-40C, mucous of throat is moderately hyperemic, there is crater-like ulcer with purulent content on left tonsil; regional lymph nodes are slightly enlarged. 18. Choose symptoms, typical for infectious mononucleosis: a) Acute onset, general headache, pain in throat, fever 38-40C, disseminated lymphoadenopathy, mucous of throat is hyperemic; there are easy removed white-yellow purulent membranes in lacunas of tonsils, that more than 7-14 days does not disappear; enlargement of spleen; b) Acute beginning, strong headache, pain in throat on swallowing, fever 39-40C, mucous of throat is hyperemic; there are easy removed grayyellow purulent membranes in lacunas of tonsils; jugular lymph nodes are increased and sharply painful; c) Acute beginning, moderate pain in a throat, fever up to 38C, mucous of throat is hyperemic, there are white colored, hardly removed membranes on tonsils; slightly increased and moderately painful jugular lymph nodes; d) Acute onset, fever up to 38 - 40C, mucous of throat is moderately hyperemic, there is crater-like ulcer with purulent content on left tonsil; regional lymph nodes are slightly enlarged. 19. Choose symptoms, typical for acute tonsillitis: a) Acute onset, general headache, pain in throat, fever 38 -40C, disseminated lymphoadenopathy, mucous of throat is hyperemic; there are easy removed white-yellow purulent membranes in lacunas of tonsils; enlargement of spleen; b) Acute beginning, strong headache, pain in throat on swallowing, fever 39-40C, mucous of throat is hyperemic; there are easy removed grayyellow purulent membranes in lacunas of tonsils; jugular lymph nodes are increased and sharply painful; c) Acute beginning, moderate pain in a throat, fever up to 38C, mucous of throat is hyperemic, there are white colored, hardly removed membranes on tonsils; slightly increased and moderately painful jugular lymph nodes; d) Acute onset, fever up to 38 - 40C, mucous of throat is moderately hyperemic, there is crater-like ulcer with purulent content on left tonsil; regional lymph nodes are slightly enlarged. 20. What symptoms are typical for initial period of typhoid fever? a) Acute beginning with increasing of body temperature within 1-2 days up to 39-40C, persistent headache, sleeplessness, anxiety, euphoria.

Face is hyperemic, vessels of sclera are injected, spleen is enlarged since the 3-4 day of disease; b) Gradual onset with slow (within 4-5 days) increase of temperature, general adynamia, sleeplessness, pale skin, tongue is thickened, with prints of teeth, relative bradycardia, spleen is enlarged since the 6-7 day of disease; c) Acute beginning of disease, within several hours temperature reaches 38-39C, accompanied with chill, headache in forehead and temple area, pain in eyes, stiffness of a nose, dry cough, pain on trachea projection. Spleen is not enlarged. d) Acute beginning of disease with chill, fast increase of temperature up to 39-40C, weakness, sweating, myalgias, excitation, headache, dry cough, hyperemia of face, injection of sclera, sometimes palatal enanthema, relative bradycardia, spleen is enlarged quite often. 21. What symptoms are typical for acute period of louse-borne typhus? a) The body temperature up to 39-40C, persistent headache, prostration, hallucinations, photophobia, serous meningitis, anxiety, euphoria can be present.. Face is hyperemic, vessels of sclera are injected, and spleen is enlarged since the 3-4 day of disease; rash cone out on the upper part of trunk on the fifth day of fever, involving whole body except of the face, palms, and soles. Initially this rash is macular, maculopapular, petechial. b) Gradual onset with slow (within 4-5 days) increase of temperature, general adynamia, pale skin, tongue is thickened, with prints of teeth, relative bradycardia, spleen is enlarged since the 6-7 day of disease; c) Acute beginning of disease, within several hours temperature reaches 38-39C, accompanied with chill, headache in forehead and temple area, pain in eyes, stiffness of a nose, dry cough, pain on trachea projection. Spleen is not enlarged. d) Acute beginning of disease with chill, fast increase of temperature up to 39-40C, weakness, sweating, myalgias, excitation, headache, dry cough, hyperemia of face, injection of sclera, sometimes palatal enanthema, relative bradycardia, spleen is enlarged quite often. 22. What symptoms are typical for initial period of influenza? a) Acute beginning with increasing of body temperature within 1-2 days up to 39-40C, persistent headache, sleeplessness, anxiety, euphoria. Face is hyperemic, vessels of sclera are injected, spleen is enlarged since 3-4 day of disease; b) Gradual onset with slow (within 4-5 days) increase of temperature, general adynamia, sleeplessness, pale skin, tongue is thickened, with prints of teeth, relative bradycardia, spleen is enlarged since the 6-7 day of disease; c) Acute beginning of disease, within several hours temperature reaches 38-39C, accompanied with chill, headache in forehead and

66

temple area, pain in eyes, stiffness of a nose, dry cough, pain on trachea projection. Spleen is not enlarged. d) Acute beginning of disease with chill, fast increase of temperature up to 39-40C, weakness, sweating, myalgias, excitation, headache, dry cough, hyperemia of face, injection of scleras vessels, sometimes palatal enanthema, relative bradycardia, spleen is enlarged quite often. 23. What symptoms are typical for gastroenterocolitic form of shigellosis: a) Acute onset, increase of temperature up to 39C, repeated vomiting, diffuse abdominal pain, frequent plentiful watery stool without mucus and blood, spasm of calves muscles, oliguria, hypotonia; b) Acute onset, increase of temperature up to 38,5-39C, vomiting 3-4 times, spastic abdominal pain in left hypogastria, plentiful liquid stool with admixtures of mucus and blood c) Acute onset, normal body temperature, vomiting, frequent plentiful watery stool without admixtures, spasm of calves muscles, expressed dryness of mucous, decrease of skin elasticity, oliguria; d) Acute onset, increase of temperature up to 39C, spastic pain in the left half of abdomen, frequent liquid poor stool with mucus and blood, tenesmus. 24. What symptoms are typical for colitic form of shigellosis? a) Acute onset, increase of temperature up to 39C, repeated vomiting, diffuse abdominal pain, plentiful watery stool without mucus and blood, spasm of calves muscles, oliguria, hypotension; b) Acute onset, increase of temperature up to 38,5-39C, vomiting 3-4 times, spastic abdominal pain in left hypogastria, frequent plentiful liquid stool with admixtures of mucus and blood; c) Acute onset, normal body temperature, vomiting, frequent plentiful watery stool without admixtures, spasm of calves muscles, expressed dryness of mucous, decrease of skin elasticity, oliguria; d) Acute onset, increase of temperature up to 39C, spastic pain in the left half of abdomen, frequent liquid poor stool with mucus and blood, tenesmus. 25. What clinical signs are typical for cholera? a) Acute onset, increase of temperature up to 39C, repeated vomiting, diffuse abdominal pain, plentiful watery stool without mucus and blood, spasm of calves muscles, oliguria, hypotension; b) Acute onset, increase of temperature up to 38,5-39C, vomiting 3-4 times, spastic abdominal pain in left hypogastria, frequent plentiful liquid stool with admixtures of mucus and blood; c) Acute onset, normal body temperature, vomiting, frequent plentiful watery stool without admixtures, spasm of calves muscles, expressed dryness of mucous, decrease of skin elasticity, oliguria; d) Acute onset, increase of temperature up to 39C, spastic pain in the left half of abdomen, frequent liquid poor stool with mucus and blood, tenesmus.

26. What hemodynamic parameters are typical for hemorrhagic fever with renal syndrome? a) Tachycardia, hypotension, tendency to collapse; b) Relative bradycardia, dycrotic pulse, hypotension; c) Significant bradycardia, hypotension; d) Tachycardia up to 120-160 /min., cardiac arrhythmia, hypotension. 27. What hemodynamic parameters are typical for typhoid fever? a) Tachycardia, hypotension, tendency to collapse; b) Relative bradycardia, dycrotic pulse, hypotension; c) Significant bradycardia, hypotension; d) Tachycardia up to 120-160 /min., cardiac arrhythmia, hypotension. 28. What hemodynamic parameters are typical for louse-borne typhus? a) Tachycardia, hypotension, tendency to collapse; b) Relative bradycardia, dycrotic pulse, hypotension; c) Significant bradycardia, hypotension; d) Tachycardia up to 120-160 /min., cardiac arrhythmia, hypotension. 29. What are the main symptoms of hepatic coma? a) Patient is unconsciousness. Temperature 39,8C. Skin is pale; slight jaundice. Tachycardia up to 140 /min. Sizes of liver by Kurlov 12119 cm., spleen is palpated on 4 cm. below edge of costal arch; b) Patient is unconsciousness. Temperature 37,2C. Bright jaundice, single petechial rash. Tachycardia up to 98 /min. Sizes of liver by Kurlov 887 cm.. Spleen is not enlarged; c) Patient is unconsciousness. Temperature 36,1C. Skin is pale, dry. Tachycardia up to 110 /min. Sizes of liver by Kurlov: 1098 cm. Spleen is not enlarged; d) Patient is unconsciousness. Temperature 38,5C. Skin is of light-pink color and plentiful hemorrhagic rash is presented. Tachycardia up to 130 /min. Liver and spleen are not enlarged. 30. What are the main symptoms of hyperglycemic coma? a) Patient is unconsciousness. Temperature is 39,8C. Skin is pale; slight jaundice. Tachycardia up to 148 /min. Sizes of liver by Kurlov 12119 cm., spleen is palpated on 4 cm. below edge of costal arch; b) Patient is unconsciousness. Temperature is 37,2C. Bright jaundice, and single petechial rash. Tachycardia up to 98 /min. Sizes of liver by Kurlov 887 cm.. Spleen is not enlarged; c) Patient is unconsciousness. Temperature is 36,1C. Skin is pale, dry. Tachycardia up to 110 /min. Sizes of liver by Kurlov: 1098 cm. Spleen is not enlarged; d) Patient is unconsciousness. Temperature is 38,5C. Skin is of light pink colorand plentiful hemorrhagic rash is present. Tachycardia up to 130 /min. Liver and spleen are not enlarged. 31. What are the main symptoms of malarial coma?

67

a) Patient is unconsciousness. Temperature is 39,8C. Skin is pale, slight jaundice. Tachycardia up to 148 /min. Sizes of liver by Kurlov 12119 cm., spleen is palpated on 4 cm. below edge of costal arch; b) Patient is unconsciousness. Temperature is 37,2C. Bright jaundice and single petechial rash. Tachycardia up to 98 /min. Sizes of liver by Kurlov 887 cm.. Spleen is not enlarged; c) Patient is unconsciousness. Temperature is 36,1C. Skin is pale, dry. Tachycardia up to 110 /min. Sizes of liver by Kurlov: 1098 cm. Spleen is not enlarged; d) Patient is unconsciousness. Temperature is 38,5C. Skin is of light pink color and plentiful hemorrhagic rash is presented. Tachycardia up to 130 /min. Liver and spleen are not enlarged. 32. What are the typical symptoms of initial period of pseudo-tuberculosis? a) Acute onset, in 2-6 hours body temperature reaches 38-39C, expressed headache in forehead and eyes, sore throat, dry cough. Skin of face is hyperemic, scleritis. Lymph nodes and spleen are not enlarged. Urine passage is normal; b) Acute onset with chill, in 12-24 hours temperature reaches 40C, strong headache, expressed muscular pain. Skin of face and neck is hyperemic and icteric; scleritis, petechial rash are present, liver and spleen are enlarged. Oliguria. c) Acute onset, on the 2-3 day of disease temperature reaches 40C, headache, loss of appetite, pain in a throat, liquid stool 3-5 times per day. Skin of face, neck, palms and feet are hyperemic, pale nasal triangle. Liver and spleen are enlarged. Urine passage is normal; d) Acute onset, on the 2-3 day of disease body temperature reaches 39,5-40C, strong headache, anxiety, quite often excitation, euphoria; hyperemia of face, vessels of sclera are injected, spleen is enlarged. Urine passage is normal. 33. What are the typical symptoms of initial period of influenza? a) Acute onset, in 2-6 hours body temperature reaches 38-39C, expressed headache in forehead and eyes, sore throat, dry cough. Skin of face is hyperemic, scleritis. Lymph nodes and spleen are not enlarged. Urine passage is normal; b) Acute onset with chill, in 12-24 hours temperature reaches 40C, strong headache, expressed muscular pain. Skin of face and neck is hyperemic and icteric; scleritis, petechial rash are present, liver and spleen are enlarged. Oliguria. c) Acute onset, on the 2-3 day of disease temperature reaches 40C, headache, loss of appetite, pain in a throat, liquid stool 3-5 times per day. Skin of face, neck, palms and feet are

hyperemic, pale nasal triangle. Liver and spleen are enlarged. Urine passage is normal; d) Acute onset, on the 2-3 day of disease body temperature reaches 39,5-40C, strong headache, anxiety, quite often excitation, euphoria; hyperemia of face, vessels of sclera are injected, spleen is enlarged. Urine passage is normal. 34. What are the typical symptoms of initial period of leptospirosis? a) Acute onset, in 2-6 hours body temperature reaches 38-39C, expressed headache in forehead and eyes, sore throat, dry cough. Skin is hyperemic, scleritis. Lymph nodes and spleen are not enlarged. Urine passage is normal; b) Acute onset with chill, in 12-24 hours temperature reaches 40C, strong headache, expressed muscular pain (predominate in calves muscles). Skin of face and neck is hyperemic and icteric; scleritis, petechial rash are present, liver and spleen are enlarged. Oliguria. c) Acute onset, on the 2-3 day of disease temperature reaches 40C, headache, loss of appetite, pain in a throat, liquid stool 3-5 times per day. Skin of face, neck, palms and feet are hyperemic, pale nasal triangle. Liver and spleen are enlarged. Urine passage is normal; d) Acute onset, on the 2-3 day of disease body temperature reaches 39,5-40C, strong headache, anxiety, quite often excitation, euphoria; hyperemia of face, vessels of sclera are injected, spleen is enlarged. Urine passage is normal. 35. What symptoms are typical for meningococcal nasopharyngitis? a) Granularity and hyperemia of a back wall of throat; b) Hemorrhagic rash; c) Stiffness of neck muscles; d) Plentiful purulent nasal excretion. 36. What symptoms are typical for meningococcal meningitis? a) Granularity and hyperemia of a back wall of throat; b) Hemorrhagic rash; c) Stiffness of neck muscles; d) Plentiful purulent nasal excretion. 37.What symptoms are typical for meningococcemia? a) Granularity and hyperemia of a back wall of throat; b) Hemorrhagic rash; c) Stiffness of neck muscles; d) Plentiful purulent nasal excretion. 38. What symptoms of CNS affection are typical for louse-borne typhus? a) Hyperesthesia, nausea and vomiting, loss of consciousness, headache, positive Kernings symptom, stiffness of neck muscles;

68

b) Euphoria, excitation, anxiety, acoustical and visual hallucinations, persistent headache, symptoms of cranial nerves affection; c) Adynamia, dormancy, dull headache; d) Dizziness, noise in ears, adynamia, headache in forehead and temple areas. 39. What symptoms of CNS affection are typical for typhoid fever? a) Hyperesthesia, nausea and vomiting, loss of consciousness, headache, positive Kernigs symptom, stiffness of neck muscles; b) Euphoria, excitation, anxiety, acoustical and visual hallucinations, persistent headache, symptoms of cranial nerves affection; c) Adynamia, dormancy, dull headache; d) Dizziness, noise in ears, adynamia, headache in forehead and temple areas. 40. What symptoms of CNS affection are typical for purulent meningitis a) Hyperesthesia, nausea and vomiting, quite often loss of consciousness, headache, positive Kernigs symptom, stiffness of neck muscles; b) Euphoria, excitation, anxiety, acoustical and visual hallucinations, persistent headache, symptoms of cranial nerves affection; c) Adynamia, dormancy, dull headache; d) Dizziness, noise in ears, adynamia, headache in forehead and temple areas. 41. What type of rash and its location are typical for pseudo-tuberculosis? a) Roseolar and papular rash arises on the 2-5 day of disease on top of trunk, then spreads on face, extremities and around joints; b) Plentiful, roseolar and petechial rash arises on the 4-5 day of disease, mainly on thorax, extremities, usually is absent on face, palms and feet; c) Little-pointed erythematic rash arises on the 26 day of disease on intact skin, is located mainly on the lateral surfaces of a trunk, axillary areas, abdomen, and extremities, is concentrated on palms and feet as "gloves" and "socks"; face is usually clear; d) Spotty-papular rash arises on the 3-4 day of disease on the face, then on trunk, forearms thighs, and on shins, leaves pigmentation and peeling after recovery; 42. What type of rush and its location are typical for louse-borne typhus? a) Roseolar and papular rash arises on the 2-5 day of disease on top of trunk, then spreads on face, extremities and around joints; c) Plentiful, roseolar and petechial rash arises on the 4-5 day of disease, mainly on thorax, extremities, usually is absent on face, palms and feet; d) Little-pointed erythematic rash arises on the 26 day of disease on intact skin, is located mainly on the lateral surfaces of a trunk, axillary areas, abdomen, and extremities, is

concentrated on palms and feet as "gloves" and "socks"; face is usually clear; e) Spotty-papular rash arises on the 3-4 day of disease on the face, then on trunk, forearms and thighs, then on shins, leaves pigmentation and peeling after recovery. 43. What type of rush and its location are typical for tick-borne North Asian rickettsiasis? a) Roseolar and papular rash arises on the 2-5 day of disease on top of trunk, then spreads on face, extremities and around joints; b) Plentiful, roseolar and petechial rash arises on the 4-5 day of disease, mainly on thorax, extremities, usually is absent on face, palms and feet; c) Little-pointed erythematic rash arises on the 2-6 day of disease on intact skin, is located mainly on the lateral surfaces of a trunk, axillary areas, abdomen, and extremities, is concentrated on palms and feet as "gloves" and "socks"; face is usually clear; d) Spotty-papular rash arises on the 3-4 day of disease on the face, then on trunk, forearms thighs, and on shins, leaves pigmentation and peeling after recovery; 44. What mechanism of rash formation is typical for meningococcemia? a) Damage of capillarys endothelium by microorganisms and toxins, circulating in blood; b) Sedimentation of immune complexes in capillaries of skin; c) Drift of pathogen in lymph vessels of skin with subsequent development of productive inflammatory changes; d) Generalized destructive and prolypherative vasculitis; 45. What mechanism of rash formation is typical for typhoid fever? a) Damage of capillarys endothelium by microorganisms and toxins, circulating in blood; b) Sedimentation of immune complexes in capillaries of skin; c) Generalized destructive and prolypherative vasculitis; d) Drift of pathogen in lymph vessels of skin with subsequent development of productive - inflammatory changes; 46. What mechanism of rash formation is typical for louse-borne typhus? a) Damage of capillarys endothelium by microorganisms and toxins, circulating in blood; b) Sedimentation of immune complexes in capillaries of skin; c) Drift of pathogen in lymph vessels of skin with subsequent development of productive inflammatory changes; d) Generalized destructive and prolypherative vasculitis; 47. What abdominal symptoms are typical for rotaviral infection?

69

a) Spleen and liver are enlarged since the 3-4 day of disease; tendency to constipation, paradoxical ishuria, oliguria; b) Liver and spleen are enlarged since the 6-7 day of disease, stool is liquid 3-4 times per day, meteorism; c) Liver and spleen are not enlarged, abdomen is soft, slightly painful around navel, stool is liquid from 2 up to 10 times per day; d) Liver and spleen are enlarged since the 3-4 day of disease, sensitive on palpation, meteorism, mesenteric lymph nodes are palpated, stool 1-2 times a day. 48. What abdominal symptoms are typical for louseborne typhus? a) Spleen and liver are enlarged since the 3-4 day of disease, soft, tendency to constipation, paradoxical ishuria, oliguria; b) Liver and spleen are enlarged since the 6-7 day, stool liquid 3-4 times per day, meteorism; c) Liver and spleen are not enlarged, abdomen is soft, slightly painful around navel, stool is liquid from 2 up to 10 times per day; d) Liver and spleen are enlarged since the 3-4 day of disease, sensitive on palpation, meteorism, mesenteric lymph nodes are palpated, stool 1-2 times a day. 49.What abdominal symptoms are typical for typhoid fever? a) Spleen and liver are enlarged since the 3-4 day of disease, soft;; tendency to constipation, paradoxical ishuria, oliguria; b) Liver and spleen are enlarged since the 6-7 day, tool is liquid 3-4 times per day, meteorism; c) Liver and spleen are not enlarged, abdomen is soft, slightly painful around navel, stool is liquid from 2 up to 10 times per day; d) Liver and spleen are enlarged since the 3-4 day of disease, sensitive on palpation; meteorism, mesenteric lymph nodes are palpated, stool 1-2 times a day. 50. What symptoms are typical for initial period of leptospirosis? a) Acute onset with increase of body temperature up to 38C, loss of appetite, nausea, weakness and general unwell, sometimes dry cough. Since the 2-4 day of disease liver is enlarged, temperature of body normalizes; b) Acute onset with increase of body temperature up to 39,5C, muscular and joint pains; a headache in forehead area, pain in eyes, dry cough, tracheal pain. Liver is not enlarged. c) Acute onset with increase of body temperature up to 39C, headache, myalgias (mainly in calves muscles), weakness, pain in eyes. Injection of sclera, face is hyperemic, petechial rash in axillary areas. The liver is enlarged, dark urine, oliguria. d) Acute onset with increase of body temperature up to 38C, weakness, running nose, dry cough, pain in eyes. Conjunctiva are hyperemic. Neck,

axillary lymph nodes and liver are enlarged. Urine is normal. 51. What symptoms are typical for initial period of viral hepatitis A? a) Acute onset with increase of body temperature up to 38C, loss of appetite, nausea, weakness and general unwell, sometimes dry cough. Since the 2-4 day liver is enlarged, temperature of body is normalized; b) Acute onset with increase of body temperature up to 39,5C, muscular and joint pains; a headache in forehead area, pain in eyes, dry cough, tracheal pain. Liver is not enlarged. c) Acute onset with increase of body temperature up to 39C, headache, myalgias (mainly in calves muscles), weakness, pain in eyes. Vessels of sclera are injected, face is hyperemic, petechial rash in axillary areas. The liver is enlarged, dark urine, oliguria. d) Acute onset of disease with increase of body temperature up to 38C, weakness, running nose, dry cough and pain in eyes. Conjunctiva is hyperemic. Neck, axillary lymph nodes and liver are enlarged. Urine is normal. 52. What symptoms are typical for initial period of adenoviral infection? a) Acute onset with increase of body temperature up to 38C, loss of appetite, nausea, weakness and general unwell, sometimes dry cough. Since the 2-4 day of disease liver is enlarged, temperature of body normalizes; b) Acute onset with increase of body temperature up to 39,5C, muscular and joint pains; a headache in forehead area, pain in eyes, dry cough, tracheal pain. Liver is not enlarged. c) Acute onset with increase of body temperature up to 39C, headache, myalgias (mainly in calves muscles), weakness, pain in eyes. Injection of sclera, face is hyperemic, petechial rash in axillary areas. The liver is enlarged, dark urine, oliguria. d) Acute onset of disease with increase of body temperature up to 38C, weakness, running nose, dry cough, and pain in eyes. Conjunctiva is hyperemic. Neck, axillary lymph nodes and liver are enlarged. Urine is normal. Diarrhea can be present. 53. What drug may be used for specific therapy of shigellosis? a) Erythromycin; b) Cyprofloxacin; c) Penicillin; d) Metronidazol (trychopol); e) Chloroquin. 54. What drug may be used for specific therapy of intestinal amebiasis? a) Erythromycin; b) Ciprofloxacin; c) Penicillin; d) Metronidazole (Trichopol); e) Chloroquine. 55. What drug may be used for specific therapy of malaria?

70

a) Erythromycin; b) Ciprofloxacin; c) Penicillin; d) Metronidazole (Trichopol); e) Chloroquine. 56. What symptoms are typical for initial period of pseudo-tuberculosis? a) Acute onset with fast increase of temperature of a body up to 39-40, strong headache, euphoria, sleeplessness; injection of scleras vessels, insignificant hyperemia of the soft palate and back wall of throat, lymph nodes are not enlarged, spleen is enlarged since the 3-4 day of disease; b) Acute onset with increase of temperature up to 38,5-40C, edema and pain in ankle and knee joints, pain in a throat, increase of regional lymph nodes, bright hyperemia of the soft palate with precise border, white coated tongue. Spleen and liver are enlarged. c) Gradual onset, from the 3d day increase of temperature up to 38C, running nose, pain in a throat. Tonsils are enlarged, hyperemic, without fur. Neck, axillary lymph nodes are enlarged; conjunctivitis; joints are not changed; sometimes spleen is enlarged; dry cough is quite often; d) Acute onset, for several hours temperature reaches 40C, headache in frontal area, pain in eyes, tracheal pain is present. Hyperemia of soft palate, lymph nodes and spleen are not enlarged, joints are normal. 57. What symptoms are typical for initial period of louse-borne typhus? a) Acute onset with fast increase of temperature of a body up to 39-40 , strong headache, euphoria, sleeplessness; injection of scleras vessels, insignificant hyperemia of the soft palate and back wall of throat, lymph nodes are not enlarged, spleen is enlarged since the 3-4 day of disease; b) Acute onset with increase of temperature up to 38,5-40C, edema and pain in ankle and knee joints, pain in a throat, increase of regional lymph nodes, bright hyperemia of the soft palate with precise border, white coated tongue. Spleen and liver are enlarged. c) Gradual onset, from the 3d day increase of temperature up to 38C, running nose, pain in a throat. Tonsils are enlarged, hyperemic, without fur. Neck, axillary lymph nodes are increased; conjunctivitis; joints are not changed; sometimes spleen is enlarged;; dry cough is quite often; d) Acute onset, in several hours temperature reaches 40C, headache in frontal area, pain in eyes, tracheal pain. Hyperemia of soft palate, lymph nodes and spleen are not enlarged, joints are normal. 58. What symptoms are typical for initial period of adenoviral infection? a) Acute onset with fast increase of temperature of a body up to 39-40, strong headache, euphoria, sleeplessness; injection of scleras vessels, insignificant hyperemia of the soft palate and back wall of

throat; lymph nodes are not enlarged; spleen is enlarged since the 3-4 day of disease; b) Acute onset with increase of temperature up to 38,5-40C, edema and pain in ankle and knee joints, pain in a throat, increase of regional lymph nodes, bright hyperemia of the soft palate with precise border, white coated tongue. Spleen and liver are enlarged. c) Gradual onset, from the 3d day increase of temperature up to 38C, running nose, pain in a throat. Tonsils are enlarged, hyperemic, without fur. Neck, axillary lymph nodes are increased; conjunctivitis; joints are not changed; sometimes spleen is enlarged; dry cough is quite often; d) Acute onset, in several hours temperature reaches 40C, headache in frontal area, pain in eyes, tracheal pain. Hyperemia of soft palate, lymph nodes and spleen are not enlarged; joints are normal.

SKIN CHANGES IN SOME INFECTIOUS DISEASES 59. What skin phenomena are typical for skin form of plague? e) There is on the hand painless ulcer 23 cm with a blackened necrotic eschar circled with elevated hyperemic rim, where small satellite vesicles, filled by serous-hemorrhagic contents are seen. Expanding zone of brawny edema, which jelly-like fluctuates during shaking, surrounds this affection.. f) There is ulcer 11 cm with hard yellowish bottom at the skin of leg without purulent content. Ulcer is circled by elevated intensively hyperemic and sharply outlined rim with cyanotic shade. Ulcer is sharply painful. g) Moderate edema of skin. There is crater-like ulcer under gray crust on the highly infiltrated and hyperemic base in the center of edema; pus is excreted during pressing. Lymphangitis is present. Touching of ulcer is painful. a) Skin is markedly hyperemic, shining, smoothed; a zone of affection has precise festooned edges. On the center of affection hyperemia is less expressed, than on edges, where bullas with serous content are seen.. 60. What skin phenomena are typical for staphylococcal carbuncle? b) There is painless ulcer 23 cm with a blackened necrotic eschar circled with elevated hyperemic rim, where small satellite vesicles, filled by serous-hemorrhagic contents are seen. Expanding zone of brawny edema, which jelly-like fluctuates during shaking, surrounds this affection..

71

There is ulcer 11 cm with hard yellowish bottom at the skin of leg without purulent content. Ulcer is circled by elevated intensively hyperemic and sharply outlined rim with cyanotic shade. Ulcer is sharply painful d) Moderate edema of skin. There is crater-like ulcer under gray crust on the highly infiltrated and hyperemic base in the center of edema; pus is excreted during pressing. Lymphangitis is present. Touching of ulcer is painful. e) Skin is markedly hyperemic, shining, smoothed; a zone of affection has precise festooned edges. On the center of affection hyperemia is less expressed, than on edges, where bullas with serous content are seen. 61. What skin phenomena are typical for skin form of anthrax? a) There is painless ulcer 23 cm with a blackened necrotic eschar circled with elevated hyperemic rim, where small satellite vesicles, filled by serous-hemorrhagic contents are seen. Expanding zone of brawny edema, which jelly-like fluctuates during shaking, surrounds this affection. b) There is ulcer 11 cm with hard yellowish bottom at the skin of leg without purulent content. Ulcer is circled by elevated intensively hyperemic and sharply outlined rim with cyanotic shade. Ulcer is sharply painful c) Moderate edema of skin. There is crater-like ulcer under gray crust on the highly infiltrated and hyperemic base in the center of edema; pus is excreted during pressing. Lymphangitis is present. Touching of ulcer is painful. f) Skin is markedly hyperemic, shining, smoothed; a zone of affection has precise festooned edges. On the center of affection hyperemia is less expressed, than on edges, where bullas with serous content are seen. 62. What character and localization of rush are typical for pseudo-tuberculosis? a) Roseolar and petechial rash occurs on the 4-5 days of disease on lateral surfaces of trunk, internal surfaces of shoulders and forearms (intact face, palms, plants); b) Rash is plentiful, roseolar and papular, sometimes erythematous (face, neck); it arises on the 2-3 days of disease, condenses on skin of joints, palms, feet and resembles there "gloves" and "stocks"; c) Single elements of roseolar rash arise on the 810 day of disease, are located on forward surface of abdomen and thorax, some elements are roseolopapular. 63. What character and localization of rush are typical for typhoid fever? a) Roseolar and petechial rash occurs on the 4-5 days of disease on lateral surfaces of trunk, internal surfaces of shoulders and forearms (intact face, palms, plants); c)

b) Rash is plentiful, roseolar and papular, sometimes erythematous (face, neck); it arises on the 23 days of disease, condenses on skin of joints, palms, feet and resembles there "gloves" and "stocks"; c) Single elements of roseolar rash arise on the 810 day of disease, are located on forward surface of abdomen and thorax, some elements are roseolopapular. 64. What character and localization of rush are typical for louse borne-typhus? a) Roseolar and petechial rash occurs on the 4-5 days of disease on lateral surfaces of trunk, internal surfaces of shoulders and forearms (intact face, palms, plants); b) Rash is plentiful, roseolar and papular, sometimes erythematous (face, neck); it arises on the 23 days of disease, condenses on skin of joints, palms, feet and resembles there "gloves" and "stocks"; d) Single elements of roseolar rash arise on the 810 day of disease, are located on forward surface of abdomen and thorax, some elements are roseolopapular.

LABORATORY METHODS OF DIAGNOSTICS 65. What methods are used for diagnosing of HIV/AIDS? (2) a) Blood thin smear and thick drop microscopy; b) Common blood test; c) Immune enzyme assay; d) Immune blotting; e) Culturing on embryo eggs 66. What tests are used for cytomegalovirus infection diagnostics? a) Compliment fixation test; b) Common blood test; c) Immune enzyme detection of specific IgM and IgG in serum of blood; d) Cytological assay of urine sediment; e) Immunological assay (estimation of absolute and relative amount of lymphocytes subpopulation, titer of IgG, IgM, IgA antibodies in serum of blood). 67. What methods are used for detection of replication activity of HIV? a) Indirect agglutination test; b) PCR; c) Complement fixation test; d) Immunofluorescent assay with specific monoclonal antibodies; e) Western blotting. 68. What is the defect of immunoenzyme assay ( ELYSA) in HIV/AIDS diagnostics? a) Low sensitivity; b) Low specificity; c) Expensive cost; d) High level of false positive results; e) High level of false-negative results. 69. How to treat positive result IEA in case of absence of clinical manifestation of HIV/AIDS?

72

a) to HIV;

Patient has stable protective immunity

b) Patient may be in latent period, acute stage was asymptomatic or wasnt diagnosed; c) It is incubation period; d) It is an acute asymptomatic stage of infection. 70. What is physicians tactic in case of negative result of IEA in patient with suspicion on HIV-infection? a) To perform common blood test; b) To repeat IEA with test-system of other firm; c) To exclude diagnosis of HIV-infection; d) To prescribe antiretroviral therapy; e) To repeat IEA assay through one ear. 71. Which methods from mentioned below are informative for cytomegalovirus infection? a) Blood thin smear and tick drop microscopy; b) Common blood test; c) Detection of antibodies IgM with immunoenzyme assay (IEA); d) Detection of antibodies IgG with IEA; e) Detection of specific-changed cells in urine. 72. What laboratory methods should be applied for proving of herpesviral etiology of meningoencephalitis? a) Investigation of dynamic titer of antibodies IgM and IgG; b) PCR in CSF smears; c) Common blood test; d) Immunoenzyme assay with detection of antibodies IgM, IgG; e) All mentioned. 73. What laboratory method is more informative for proving of chronic brucellosis? a) Culture of blood; b) Bacterioscopy of blood smears; c) Allergological test; d) Detection of antibodies with indirect hemagglutination test. e) Biological method. 74. What smears from mentioned below may be used for bacteriological investigation of patients with pseudotuberculosis? a) Blood; b) Nasal secretion; c) Sputum; d) Urine; e) All mentioned. 75 What changing in CSF is most characteristic for meningococcal meningitis? a) CSF pressure elevation; b) High neutrophilic pleocytosis; c) Proteincell dissociation; d) Normal glucose amount; e) Greenish color of CSF. 76. The main advantage of passive immunization over active immunization is that passive immunization: a) Eliminates the risk of hypersensitivity reactions; b) Is effective against multiple organisms;

Is more cost-effective as a public health measure; d) Provides immediate protection; e) Magnifies the specific immune response to the offending organism. 77. Which one of the following methods is not informative for diagnosing of chronic toxoplasmiasis? a) X-ray examination of brain and muscles; b) Parasitological examination of feces; c) Parasitological examination of blood; d) ECG; e) Examination of eye bottom. 78. Which one of the following markers indicates acute viral hepatitis A? a) HBsAg in blood serum; b) Anti-HBcor IgG in blood serum; c) Anti-HAV IgG in blood serum; d) Anti-HAV IgM in blood serum; e) Anti-HCV IgM in blood serum. 79. Which one of the following methods is more informative for diagnosing of food poisonings? a) Bacteriological investigation of vomiting masses; b) Bacteriological investigation of feces; c) Bacteriological investigation of blood; d) Detection of toxin; e) Serological investigation. 80. What changes in blood count are typical for shigellosis? a) Leukopenia; b) Moderate neutrophilic leukocytosis; c) Relative lymphomonocytosis; d) Drum-sticks shift to left. 81. Which one of the following methods is more informative for early specific diagnosing of ARVI? a) Rhinocytoscopy; b) Culturing on egg embryos; c) Serological; d) Immunofluorescent assay; e) Clinical and epidemiological investigation. 82. Which one of the following methods is more informative for diagnosing of latent forms of toxoplasmosis? a) Compliment fixation test; b) X-ray examination of brain; c) Parasitoscopy of feces; d) Sabin-Feldman test; e) Direct immunofluorescent test. 83. Which one of the following markers is more informative for diagnosing of acute viral hepatitis B? a) Anti-HAV IgM; b) HBs Ag; c) Hbe Ag; d) Anti-HBe; e) Anti-HBc IgM. 84. What methods of specific laboratory diagnostics of shigellosis is usually apply? a) Bacteriological investigation of faeces; b) Bacteriological investigation of urine;

c)

73

c) Indirect hemagglutination test (IHAT); d) Immunofluorescent test; e) Immunoenzyme assay. 85. Which one of the following methods is more informative for diagnosing of cholera? a) Bacterioscopy of feces ; b) Culture of blood; c) Serological investigation; d) Culture of feces and vomit masses; e) Skin allergic test. 86. Which one of the following methods is more informative for diagnosing of botulism? a) Identification of botulinum toxin with inoculation of mice with patients blood; b) Culture of blood; c) Serological method; d) Culture of urine; e) Culture of suspicious food. 87. What serological tests are usually apply for diagnostic of brucellosis? a) Complement fixation test; b) Right and Heddelson agglutination test; c) Indirect hemagglutination test; d) Immunofluorescent test; e) Latex agglutination test. 88. Which one of the following groups of methods is more informative for diagnosing of pseudotuberculosis? a) Epidemiological; 74. Clinical; 75. Serological.; e) Bacteriological; f) All mentioned. 89. Bacteriological investigation of which one of the following smears is more effective for early diagnosing of typhoid and paratyphoid fevers? a) Feces; b) Blood; c) CSF; d) Urine; e) All mentioned. 90. In what case result of immunoenzyme assay for diagnostic of HIV-infection may be false negative? a) In patients with malaria; b) In patients, treated with glucocorticoids; c) In pregnant woman; d) In homosexual persons; e) In drug abused persons. 91. What peculiarities of blood cell count are typical for climax period of typhoid fever? a) Leukopenia, neutropenia, eosinopenia, relative lymphomonocytosis; b) Normal count or insignificant Leukocytosis, tendency to neutrophilic increase, drum-stick shift to the left, monocytosis, plasmatic Turk cells; c) Leukocytosis, eosinophylia, lymphocytosis, ESR elevation;

d) Leukocytosis, neutropenia, lymphomonocytosis, atypical mononuclears. e) Leukocytosis, neutrophilia, drumstick shift to the left, high level of ESR 92. What changes in CBC are typical for influenza? a) Leukocytosis, eosinophylia, neutropenia, lymphocytosis, monocytosis, expressed elevated ESR; b) Leukopenia, neutropenia, lymphocytosis, normal ESR; c) Leukocytosis, neutropenia, monocytosis, lymphocytosis, normal ESR . d) Leukocytosis, neutrophilia, drum-stick shift to the left, moderately elevated ESR. e) Leukopenia, neutropenia, drum-stick shift to the left, eosinopenia, relative lymphomonocytosis, moderately elevated ESR 93. What changes in CBC are typical for infectious mononucleosis? a) a) Leukocytosis, neutropenia, lymphocytosis, monocytosis; b) b) Leukopenia, neutropenia, lymphocytosis; c) c) Leukocytosis, neutropenia, monocytopenia, lymphocytosis. d) d) Leukocytosis, neutrophilia, drum-stick shift to the left. 94. What changes in CBC are typical for acute tonsillitis? a) Leukocytosis, neutropenia, lymphocytosis, monocytosis; b) Leukopenia, neutropenia, lymphocytosis; c) Leukocytosis, neutropenia, monocytopenia, lymphocytosis. d) Leukocytosis, neutrophilia, drum-stick shift to the left. 95. What methods of specific diagnostics are informative for rotoviral gastroenteritis? a) Bacteriological investigation of feces; b) Microscopy of feces; c) Latex agglutination test; d) Rectosigmoidoscopy; e) Revealing of viral antigen in feces with IFA. 96. Which one of the following methods is more informative for specific diagnosing of gastrointestinal form of salmonellosis? a) Culture of feces; b) Bacterioscopy of feces; c) Culture of urine; d) Rectosigmoidoscopy; e) Coprological investigation of feces 97. Which one of the following methods is more informative for specific diagnosing of lambliasis? a) Culture of feces; b) Bacterioscopy of feces; c) Culture of urine; d) Rectosigmoidoscopy; e) Coprological investigation of feces

74

98. Which one of the following appearances of CSF is more typical for meningococcal meningitis? a) Muddy, of white-grayish color; b) Transparent, colorless c) Hemorrhagic; d) Muddy, of greenish color; e) Transparent with fibrinous membrane 99. Which one of the following appearances of CSF is more typical for viral meningitis? a) Muddy, of white-grayish color; b) Transparent, colorless c) Hemorrhagic; d) Muddy, of greenish color; e) Transparent with fibrinous membrane 100. Which one of the following appearances of CSF is more typical for antrax meningitis? f) Muddy, of white-grayish color; g) Transparent, colorless h) Hemorrhagic; i) Muddy, of greenish color; j) Transparent with fibrinous membrane 101. What blood cells count is typical for meningococcemia? a) Neutrophilic hyperleukocytosis with left shift, high ESR; b) Moderate neutrophilic leukocytosis, ESR is moderately increased; c) Leukopenia, lymphomonocytosis, sometimes ESR increase; d) Leukopenia, relative lymphocytosis, increase of plasmatic cells number, increase of ESR. 102. What blood cell count is typical for tick-borne encephalitis? a) Neutrophilic hyperleukocytosis with left shift, high ESR; b) Moderate neutrophilic leukocytosis, ESR is moderately increased; c) Leukopenia, lymphomonocytosis, sometimes ESR increase; d) Leukopenia, relative lymphocytosis, increase of plasmatic cells number, increase of ESR. 103. Choose the methods of specific diagnosing of louse-borne typhus (Brills disease): a) CFT, IHAT with Rickettsia provazekii; b) CFT with Rickettsia burneti; c) CFT, IHAT with Rickettsia sybirica; d) IEA with Borrelia burgdorferi; e) CFT, IHAT with Rickettsia mooseri. 104. Choose the methods of specific diagnosing of North-Asian rickettsiosis: a) CFT, IHAT with Rickettsia provazekii; b) CFT with Rickettsia burneti; c) CFT, IHAT with Rickettsia sybirica; d) IEA with Borrelia burgdorferi; e) CFT, IHAT with Rickettsia mooseri 105. Choose the methods of specific diagnosing of Qfever: a) CFT, IHAT with Rickettsia provazekii; b) CFT with Rickettsia burneti; c) CFT, IHAT with Rickettsia sybirica;

d) IEA with Borrelia burgdorferi; e) CFT, IHAT with Rickettsia mooseri 106. Which one of the following results of CSF investigations is typical for serous meningitis? a) Colorless, transparent, cytosis 0,003 109 /l., all cells are lymphocytes; protein 0,33 /. Test for protein sedimentation is negative. CSF pressure is 250 mm of water; b)Colorless, transparent, cytosis 0,2 109 /l, lymphocytes 80 %, protein 1,0 g/l. Test for protein sedimentation is positive. CSF pressure is 300 mm of water; c) Colorless, transparent, cytosis 0,01 109 /l, lymphocytes 85 %, protein 0,3 g/l, sediment reaction negative, pressure is 300 mm of a water; d)Muddy, white-yellow color, cytosis 15,0 109 /l, neutrophils 100 %, protein 6,6 g/l, sediment reaction are sharply positive, pressure of 350 mm of a water. 107. Which one of the following results of CSF investigations is typical for purulent meningitis? a) Colorless, transparent, cytosis 0,003 109 /l., all cells are lymphocytes; protein 0,33 /. Test for protein sedimentation is negative. CSF pressure is 250 mm of water; b)Colorless, transparent, cytosis 0,2 109 /l, lymphocytes 80 %, protein 1,0 g/l. Test for protein sedimentation is positive. CSF pressure is 300 mm of water; c) Colorless, transparent, cytosis 0,01 109 /l, lymphocytes 85 %, protein 0,3 g/l, sediment reaction negative, pressure is 300 mm of water; d)Muddy, white-yellow color, cytosis 15,0 109 /l, neutrophils 100 %, protein 6,6 g/l, sediment reaction are sharply positive, pressure of 350 mm of water. 108. Which one of the following results of CSF investigations is typical for tuberculosis meningitis? a) Colorless, transparent, cytosis 0,003 109 /l., all cells are lymphocytes; protein 0,33 g/l. Test for protein sedimentation is negative. CSF pressure is 250 mm of water; b)Colorless, transparent, cytosis 0,2 109 /l, lymphocytes 70 %, glucose is significantly decrease, protein 1,0 g/l, fibrinous membrane is formed. c) Colorless, transparent, cytosis 0,01 109 /l, lymphocytes 85 %, protein 0,3 g/l, sediment reaction negative, pressure is 300 mm of a water; d)Muddy, white-yellow color, cytosis 15,0 109 /l, neutrophils 100 %, protein 6,6 g/l, sediment reaction are sharply positive, pressure of 350 mm of a water; 109.Resistance of St.aureus to methicillin is most often caused by: a) b) c) d) e) Alteration of the major target for the drug; Cell membrane impermeability; Decreased uptake of the antibiotic; Inactivation of autolysins; Synthesis of a potent -lactamaze.

75

110. The molecular basis for the effect of cholera toxin on mucosal cells is (2): Activation of adenylate cyclase; Inactivation of G protein; Increased activity of potassium pumps; Increased generation of cyclic adenosine monophosphate (cAMP); e) Ribosylation of a guanosine triphosphate (GTP)binding protein. 111. What is the most common cause of aseptic meningitis of viral etiology? a) Enteroviruses; b) Herpesviruses; c) Arboviruses; d) Retroviruses; e) Orthomyxoviruses. 113. The finding of large, multinucleated clumps of cells in the bronchial secretion of patient with acute bronchopneumonia suggests that this infection is caused by: a) Epstein-Barr virus; b) Str.pneumonia; c) Mycoplasta hominis; d) Rhinovirus; e) * Respiratorysynciytial virus. 114. Which one of the following infection is most likely to be classified as opportunistic? a) b) c) d) e) 115. Which Brucellosis in the child of a migrant worker; Escherichia coli bacteremia in a leukemic patient receiving chemotherapy; Group A streptococcal pharyngitis in a school-aged child; Pulmonary anthrax in a wool sorter; Pulmonary tuberculosis in a physician. one of the following immunizations should a) b) c) d)

c)

The sharing of epitopes between streptococcal and cardiac tissue glycoproteins; d) The tropism of group A streptococci for the cardiac valves. e) Induction of a specific autoimmune response to cardiac tissue;

It is another pare of shoes.

URGENT STATES IN INFECTIOUS PATHOLOGY 118. For what kind of shock increasing of relative density of blood plasma is typical? a) Infectious-toxic; b) Hypovolemic; c) Anaphylactic; d) Cardiogenic; e) Traumatic. 119. What way of drugs administration is most effective in case of infectious-toxic shock ? a) oral administration b) subcutaneously; c) intramuscularly; d) intravenously; e) intraartreioly. 120. What one of the following solutions is most effective for initial treatment of infectious-toxic shock? a) Trisol; b) 4% natrium hydrocarbonate solution; c) Polyglucine; d) Innunoglobuline; e) Regydrone. 121. Choose the clinical criteria of sopor (2): a) Contact with patient is absent; b) Contact with patient is possible only on level of verbal-mimic reactions; c) Directed reactions on pain irritations are saved; d) Reactions of pupils on light are absent; e) Patients answers on questions are monosyllabic and not always adequate. 122. Choose the most typical early clinical signs of brain edema: a) Hemorrhagical rash on the mucous membrane of soft palate; b) Cardiac arrhythmia; c) Psycho-motor excitation; d) Change of consciousness e) Tonic-and clonic cramps. 123. What level of systolic arterial pressure can result in blocking of renal filtration?

be administrated immediately after birth? a) Diphtheria-pertussis-tetanus (DPT) vaccine;

b) Haemophilus influenza type B vaccine; c) Hepatitis B vaccine;

d) HIV-vaccine; e) Oral polio vaccine

116.The most important pathogenic factor leading to the development of carditis in a patients with chronic streptococcal tonsillitis may be: a) Persistent release of streptococcal antigens from silent infectious foci; b) The release of carditoxin;

76

a) 105/70 mm Hg; b) 100/60 mm Hg; c) 70/50 mm Hg; d) 95/60 mm Hg; e) 90/50 mm Hg. 124. What is drug of choose for treatment of anaphylactic shock? a) Rhiboxine; b) Infusion of 5% glucose solution; c) Adrenaline; d) Glucocorticoids; e) Cordiamine. 125. Choose the appropriate remedies for treatment of hypovolemic shock: a) Ringer-Locks solution; b) Isotonic solution of sodium chloride; c) Trisol solution; d) 10% albumin solution; e) Rheopolyglucine. 126. Which clinical signs are typical for pulmonary edema? (2) a) Crepitating rales on auscultation of lungs. b) Severe dyspnea while the patient is at rest in the bed; c) Diffuse rales of large calibers on auscultation of the lung; d) Bradycardia e) Noise of pleura friction 127. Choose the early clinical signs of acute liver failure (2). a) Disorders of color sensation; b) Leucopoenia in CBC; c) Progressive decreasing of liver size; d) Disorders of consciousness; e) Low level of prothrombine index. 128. Choose the drug for treatment of comatose form of malaria caused with resistant to chloroquine plasmodia. a) Primaquine; b) Ribavirin; c) Zydovudin; d) Quinidine gluconate; e) Metronidazole. 129. What correlation of colloid and saline solutions is optimal for intensive treatment of septic shock? a) 1:1; b) 1:2; c) 2:1; d) 1:3; e) 3:1. 130. What daily dosage of dextrans is safe for treatment of infectious toxic shock? a) 3 liters; b) 500 ml; c) 10 ml; d) 800 ml; e) 2 liters. 131. Which complications are possible when dextrans are overdosed? (3) a) Necrosis of large muscles; b) Decreasing of acuteness of hearing;

c) Brain edema; d) Pulmonary edema; e) Necrosis of epithelium cells of renal tubules. 132. What speed of saline solutions infusion you must keep during the first stage of treatment of patient with hypovolemic shock? a) 1 l/h; b) 3 l/h; c) 100-120 ml/min; d) 60 drops/min; e) 50 ml/min. 133. What drugs should be given for patients with septic shock, refractory to volume replacing treatment and to glucocorticoids? a) To continue infusion of high doses of glucocorticoids (up to 1000-1500 mg); b) To begin intravenous infusion of dofamine; c) Treatment with enterosorbents; d) To begin transfusion of blood; e) To use lymposorbtion or plasmosorbtion. 134. Choose 2 of all laboratory signs typical for acute respiratory failure: a) PaO2 <50 mm Hg; b) PaCO2 >50 mm Hg; c) PaO2 40 mm Hg; d) PaCO2 40 mm Hg; e) pH of the blood 7,38. 135. Choose clinical sign that typical for croup of II stage. a) Whistle dry rales over the lungs; b) Inspiratory stridor; c) Comatose state; d) Discharging of plentiful foamy sputum; e) Diffuse moist rales of different calibers on auscultation of lungs. 136. Indicate the most characteristic clinical signs of acute broncoobstruction (2). a) Whistle dry rales over the lung; b) Dry barking cough; c) Tympanic sound on percussion of lungs; d) Discharging of abundant foamy sputum; e) Diffuse moist rales of different calibers on auscultation of lungs. 137. Indicate the most characteristic clinical signs of pulmonary edema (2): a) Whistle dry rails over a lungs; b) Severe pain in the chest on inhale; c) Comatose state; d) Discharging of plentiful foamy sputum; e) Diffuse moist rales of different calibers on auscultation of lungs. 138. Choose appropriate early clinical and laboratory signs of acute respiratory failure (2): a) Dyspnea; b) PaO2 40 mm Hg; c) Diffuse cyanosis; d) PaCO2 40 mm Hg e) The work of additional muscles on breathing 139. Which clinical and laboratory signs are typical for acute renal failure (ARF) (2)?

77

a) Dryness of mucous membranes; b) Level of creatinine in the blood 0,20 mkmol/l; c) Level of urea in the blood 0,08 mkmol/l; d) High temperature; e) Pain in abdomen. 140. Oliguria it is a state when volume of diuresis is: a) up to 1000 ml; b) up to 700 ml; c) up to 500 ml; d) up to 300 ml; e) up to 100 ml. 141. Which volume of diuresis we can classify as anuria? a) 500 ml; b) 300 ml; c) 400 ml; d) 600 ml; e) 450 ml. 142. These are all clinical and laboratory signs of acute renal failure beside of: a) vomiting; b) diarrhea; c) dryness of mucous membranes; d) urea of the blood 0,08 mmol/l; e) creatinine of the blood 0,27 mmol/l.; 143. Which infectious diseases can result in acute renal failure (3)? a) Leptospirosis; b) Brucellosis; c) Haemorrhagical fevers; d) Salmonellosis; e) Cholera. 144.Indications for hemodialysis in patients with acute renal failure are all following beside of (2): a) Potassium of plasma 5,7 mmol/l; b) Urea of plasma 0.24 mkmol/l; c) Creatinine of plasma 0,5 mkmol/l; d) Creatinine of plasma 0,3 mkmol/l; e) Longevity of anuria stage more than 10 days. 145.Which drugs are effective for treatment of acute respiratory failure (2)? a) Infusion of 10% solution of glucose; b) Infusion of 5% solution of glucose; c) Glucocorticoids; d) Prozerinum; e) Artificial pulmonary ventilation. 146. Which drugs can suppress formation of foam in patients with pulmonary edema (2)? a) Inhaling of 78% ethanol alcohol; b) Pentaminum i/m; c) Inhaling of oxygen mixed with air; d) Inhaling of ether; e) Inhaling of antiphomsilanum. 147. What clinical signs are typical for diphtheric croup: a) Sudden beginning with fast development of laryngeal stridor and wavy course;

b) Sudden development of laryngeal stridor during eating or game, absence of fever and catharal symptoms; c) Gradual development of laryngeal stridor, frequently normal body temperature, multiple of dry rales; d) Fever, hoarse voice, consequent development of barking cough, dysphonia, strydor, inspiratory dyspnea, cyanosis of lips, tachycardia. 148. What clinical signs are typical for aspiration of foreign body? a) Sudden beginning with fast development of laryngeal stridor and wavy course; b) Sudden development of laryngeal stridor during eating or game, absence of fever and catharal symptoms; c) Gradual development of laryngeal stridor, frequently normal body temperature, multiple of dry rales; d) Fever, hoarse voice, consequent development of barking cough, dysphonia, stridor, inspiratory dyspnea, cyanosis of lips, tachycardia. 149. What clinical signs are typical for croup as complication of influenza and ARVI: a) Sudden beginning with fast development of laryngeal stridor and wavy course; b) Sudden development of laryngeal stridor during eating or game, absence of fever and catharal symptoms; c) Gradual development of laryngeal stridor, frequently normal body temperature, multiple of dry rales; d) Fever, hoarse voice, consequent development of barking cough, dysphonia, lengthened sonorous breath, inspiratory dyspnea, cyanosis of lips, tachycardia. 150. Chouse main pathogenic mechanism of toxic shock: a) Dehydration; b) Primary decrease of cardiac input; c) Pathological increase of vascular capacity; d) Severe deficiency of steroid hormones; 151. Chouse main pathogenic mechanism of cardiogenic shock: a) Dehydration; b) Primary decrease of cardiac input; c) Pathological increase of vascular capacity; d) Severe deficiency of steroid hormones; 152. Chouse main pathogenic mechanism of hypovolemic shock: a) Dehydration; b) Primary decrease of cardiac input; c) Pathological increase of vascular capacity; d) Severe deficiency of steroid hormones; 153. Which of basic logical Kochs postulates can proof, that a microorganism causes the disease? a) Microorganism must be present in every case of disease, but absent in healthy individuals;

78

b) Suspected microorganism must be isolated from affected person and grown in culture; c) The same disease must result when isolated microorganism is introduced into healthy persons; d) the same microorganism must be isolated again from second affected. e) All mentioned

79

Every why has a therefore.

ADDITION

ANSWERS FOR TESTS FOR 5th YEAR STUDENS N A N A N n n s s . . 1 A 6 B 1 4 D 2 7 2 C 6 A 1 5 C 2 8 3 A 6 A 1 6 2 9 4 C 6 B 1 7 3 0 5 E 6 B 1 8 3 1 6 B 6 1 9 3 2 7 C 7 C 1 0 3 3 8 B 7 A 1 1 3 4 9 C 7 D 1 2 3 5 1 7 A 1 0 3 3 6 1 B 7 D 1 1 D 4 3 7 1 E 7 C 1 2 5 3 8 1 B 7 B 1 3 6 3 9 1 E 7 A 1 4 7 4 0 1 B 7 B 1 5 E 8 4 1 1 C 7 A 1 6 9 4 2 A N A N n n s s . . D 1 A 2 9 5 0 3 B 1 D 2 9 5 1 4 D 1 C 2 9 5 2 5 B 1 D 2 9 5 3 6 E 1 D 2 9 5 4 7 C 1 B 2 9 5 5 8 B 1 C 2 9 5 6 9 B 1 D 2 9 6 7 0 B 1 C 2 9 6 8 1 C 1 D 2 9 6 9 2 A 2 D 2 0 6 0 3 B 2 B 2 0 6 1 4 A 2 C 2 0 6 2 5 C 2 D 2 0 6 3 6 B 2 B 2 0 6 4 7 A 2 C 2 0 6 5 8 A N A N n n s s . . A 3 E 3 1 7 6 9 C 3 E 3 1 8 7 0 B 3 B 3 1 C 8 8 1 D 3 A 3 1 B 8 9 D 2 B 3 C 3 2 8 0 3 B 3 A 3 2 8 1 4 C 3 B 3 2 8 2 5 A 3 C 3 2 8 3 6 C 3 A 3 2 8 4 7 B 3 D 3 2 8 5 8 C 3 A 3 2 8 6 9 C 3 B 3 2 9 7 0 A 3 B 3 2 9 8 1 A 3 D 3 2 9 9 2 A 3 B 3 3 9 0 3 B 3 C 3 3 9 1 4 A N A N n n s s . . B 4 A 5 4 D 0 2 5 A 4 C 5 4 D 0 3 6 C 4 A 5 4 0 4 7 A 4 D 5 4 0 5 8 B 4 B 5 4 0 6 9 D 4 C 5 4 1 7 0 B 4 B 5 4 F 1 8 1 B 4 B 5 4 1 9 2 C 4 C 5 5 1 0 3 B 4 A 5 5 E 1 1 4 B 4 D 5 5 1 2 5 A 4 B 5 5 1 3 6 E 4 A 5 5 1 4 7 A 4 B 5 5 1 5 8 A 4 D 5 5 1 6 9 B 4 E 5 5 2 7 0 A n s . D

1 B 8 E 1 7 C 0 4 3 1 A 8 B 1 8 D 1 4 4 1 B 8 D 1 9 2 4 5 2 C 8 A 1 0 3 4 6 2 B 8 B 1 1 4 E 4 7 2 A 8 B 1 2 5 4 8 2 C 8 B 1 3 6 4 9 2 C 8 1 4 7 5 0 2 A 8 1 5 8 5 1 2 C 8 D 1 6 9 5 2 2 B 9 B 1 7 0 5 3 2 D 9 E 1 8 1 5 4 2 A 9 D 1 9 2 5 5 3 9 C 1 0 3 5 6 3 B 9 A 1 1 4 5 7 3 D 9 C 1 2 5 5 8 3 C 9 A 1 3 6 5 9 3 A 9 D 1 4 7 6 0 3 B 9 B 1 5 8 D 6 1 3 B 9 B 1 6 9 C 6

A 2 0 6 B 2 0 7 E 2 J 0 8 B 2 0 9 A 2 1 0 C 2 1 1 E 2 1 2 B 2 1 3 B 2 1 4 C 2 D 1 5 C 2 1 6 A 2 1 7 B 2 1 8 D 2 1 9 B 2 2 0 C 2 2 1 C 2 2 2 E 2 2 3 C 2 2 4 A 2 2

B 2 6 9 A 2 7 0 C 2 7 1 D 2 7 2 B 2 7 3 A 2 7 4 C 2 7 5 A 2 7 6 B 2 7 7 B 2 7 8 C 2 7 9 C 2 8 0 D 2 8 1 B 2 8 2 D 2 8 3 E 2 8 4 C 2 8 5 B 2 8 6 D 2 8 7 B 2 8

B 3 3 2 C 3 3 3 C 3 3 4 D 3 3 5 B 3 3 6 A 3 3 7 B 3 3 8 E 3 3 9 D 3 4 0 E 3 4 1 A 3 4 2 E 3 4 3 B 3 4 4 C 3 4 5 C 3 4 6 D 3 4 7 A 3 4 8 D 3 4 9 C 3 5 0 B 3 5

A 3 9 5 D 3 9 6 A 3 9 7 B 3 9 8 C 3 9 9 A 4 0 0 A 4 0 1 B 4 0 2 D 4 0 3 B 4 0 4 A 4 0 5 D 4 0 6 B 4 0 7 B 4 0 8 D 4 0 9 C 4 1 0 B 4 1 1 B 4 1 2 C 4 1 3 A 4 1

B 4 5 8 A 4 5 9 B 4 6 0 E 4 6 1 A 4 6 2 A 4 6 3 B 4 6 4 C 4 E 6 5 A 4 6 6 B 4 6 7 A 4 6 8 B 4 6 9 B 4 7 0 C 4 7 1 A 4 7 2 A 4 7 3 A 4 7 4 A 4 7 5 A 4 7 6 B 4 7

D 5 2 1 A 5 2 2 C 5 2 3 B 5 2 4 A 5 2 5 D 5 2 6 C 5 2 7 B 5 2 8 A 5 2 9 C 5 3 0 C 5 3 1 D 5 3 2 B 5 3 3 A 5 3 4 B 5 3 5 B 5 3 6 C 5 3 7 A 5 3 8 B 5 3 9 C 5 4

80

3 A 1 7 0 0 3 C 1 8 0 1 3 B 1 9 0 2 4 A 1 0 0 3 4 B 1 1 0 4 4 D 1 2 0 5 4 B 1 3 0 6 4 C 1 4 0 7 4 A 1 5 0 8 4 C 1 6 0 9 4 C 1 7 1 0 4 C 1 8 1 1 4 A 1 9 1 2 5 A 1 0 1 3 5 A 1 1 1 4 5 B 1 2 1 5 5 C 1 3 1 6 5 B 1 4 1 7 5 D 1 5 1 8 5 C 1

2 A 1 B 6 3 D 1 6 4 C 1 6 5 E 1 6 6 C 1 6 7 D 1 6 8 B 1 C 6 9 E 1 7 0 C 1 E 7 1 A 1 B 7 2 B 1 D 7 3 A 1 C 7 4 A 1 B 7 5 E 1 7 6 D 1 E 7 7 E 1 7 8 B 1 7 9 C 1 8 0 A 1 8 1 A 1

5 C 2 2 6 A 2 2 7 B 2 2 8 D 2 2 9 A 2 3 0 C 2 3 1 B 2 3 2 B 2 3 3 C 2 3 4 A 2 3 5 A 2 3 6 B 2 3 7 B 2 3 8 C 2 3 9 B 2 4 0 C 2 4 1 C 2 4 2 E 2 4 3 A 2 4 4 B 2

8 C 2 8 9 B 2 9 0 C 2 9 1 D 2 9 2 D 2 9 3 B 2 9 4 F 2 9 5 A 2 9 6 D 2 9 7 B 2 9 8 D 2 9 9 A 3 0 0 C 3 0 1 D 3 0 2 C 3 0 3 A 3 0 4 C 3 0 5 D 3 0 6 B 3 0 7 B 3

1 C 3 5 2 C 3 5 3 C 3 5 4 A 3 5 5 C 3 5 6 C 3 5 7 B 3 5 8 A 3 5 9 A 3 6 0 A 3 6 1 B 3 6 2 B 3 6 3 E 3 6 4 E 3 F 6 5 D 3 E 6 6 C 3 6 7 E 3 6 8 A 3 6 9 B 3 7 0 D 3

4 D 4 1 5 C 4 1 6 D 4 1 7 B 4 1 8 A 4 1 9 C 4 2 0 C 4 2 1 B 4 2 2 D 4 2 3 C 4 2 4 A 4 2 5 C 4 2 6 C 4 2 7 B 4 2 8 D 4 2 9 C 4 3 0 B 4 3 1 B 4 3 2 B 4 3 3 C 4

7 C 4 7 8 B 4 7 9 C 4 8 0 D 4 8 1 A 4 8 2 C 4 8 3 B 4 8 4 B 4 D 8 5 C 4 8 6 C 4 8 7 D 4 E 8 8 B 4 8 9 C 4 9 0 B 4 9 1 D 1 9 2 C 4 9 3 B 4 9 4 C 4 9 5 C 4 9 6 C 4

0 E 5 4 1 D 5 4 2 B 5 4 3 B 5 4 4 B 5 E 4 5 C 5 4 6 C 5 4 7 B 5 4 8 B 5 C 4 9 C 5 5 0 D 5 5 1 C 5 5 2 B 5 E 5 3 C 5 5 4 B 5 5 5 C 5 5 6 5 5 7 5 5 8 A 5 5 9 A

6 C 5 A 7 D 5 A 8 A 5 C 9 B 6 B 0 B 6 D 1 C 6 A 2 C 6 C 3 D

1 9 1 2 0 1 2 1 1 2 2 1 2 3 1 2 4 1 2 5 1 2 6

D 8 2 A 1 B 8 E 3 D 1 8 4 D 1 8 5 B 1 E 8 6 C 1 8 7 C 1 8 8 A 1 8 9

4 5 2 4 6 2 4 7 2 4 8 2 4 9 2 5 0 2 5 1 2 5 2

0 8 3 0 9 3 1 0 3 1 1 3 1 2 3 1 3 3 1 4 3 1 5

D E

7 1 3 7 2 3 7 3 3 7 4 3 7 5 3 7 6 3 7 7 3 7 8

3 4 4 3 5 4 3 6 4 3 7 4 3 8 4 3 9 4 4 0 4 4 1

A E C E

9 7 4 9 8 4 9 9 5 0 0 5 0 1 5 0 2 5 0 3 5 0 4

81

It is hors another color ANSWERS FOR CLINICAL CASES AND TESTS FOR 6th YEAR STUDENS

1.

Diagnosis: viral hepatitis Differential diagnosis: hepatitis B, C, D, leptospirosis. 2. Diagnisis: tropical malaria, malarial coma. Differential diagnosis: sepsis, typhoid fever, influenza, leptospirosis, hemorrhagic fevers. 3. Diagnosis: influenza. Differential diagnosis: parainfluenza, adenoviral infection, measles, leptospirosis, paratyphoid A. 4. Diagnosis malaria tertiana. Differential diagnosis: influenza, recurrent typhus. 5. Diagnosis: Shigellosis. Differential diagnosis: amebiazis, salmonellosis, echerichiosis, intestinal iersiniosis. 6. Diagnosis: hemorrhagic fever with renal syndrome. Differential diagnosis: leptospirosis. 7. Diagnosis: Leptospirsis. Differential diagnosis: viral hepatitis, hemorrhagic fever with renal syndrome, pseudotuberculosis. 8. Diagnosis: opistorchosis. Differential diagnosis: typhoid and paratyphoid A, B; viral hepatitis C, B, acute cholecistitis. 9. Diagnosis: diphtheria of larynx; Differential diagnosis: parainfluenza. 10. Diagnosis: rabies. Hydrophobic phase. Differential diagnosis: tetanus, arboviral encephalitis, atropine or strychnine poisoning, delirium tremens. 11. Diagnosis: polyomielitis. Differential diagnosis: botulism, arboviral encephalitis, lymphocytar meningitis; ECHO and Kocksaki diseases. 12. Diagnosis: plague, pulmonary form. Differential diagnosis: pulmonary form of antrax; pneumococcal pneumonia, influenza, complicated with edema of lungs. 13. Diagnosis: Epstine Burr infectious mononucleosis. Differential diagnosis: cytomegaloviral infection, felinosis, acute tonsillitis, localized form of diphtheria, acute HIV infection, leucosis. 14. Diagnosis: enteroviral disease. Differential diagnosis: rotaviral infection, intestinal iersiniosis, echerichiosis, salmonellosis. 15. Diagnosis: tularemia, eye bubonic form. Differential diagnosis: plague, sodocu, felinosis, purulent lymphadenitis.

16. Diagnosis: arboviral tick- borne encephalitis. Differential diagnosis: polyomielitis, rabies, meningococcal meningoencephalitis. 17. Diagnosis: serum disease. Differential diagnosis: measles, iersiniosis. 18. Diagnosis: tularemia, eye bubonic form. Differential diagnosis: plague, sodocu, felinosis, purulent lymphadenitis, infectious mononucleosis. 19. Diagnosis: anaphylactic shock Differential diagnosis: toxic infectious shock, acute poisoning. 20. Diagnosis: acute brucellosis. Differential diagnosis: typhoid fever, tuberculosis, lymphogranulematosis, reumatic fever, iersiniosis. 21. Diagnosis: measles. Differential diagnosis: rubella, enteroviral exantema, serum disease, infectious mononucleosis. 22. Diagnosis: Brills disease (endemic louse born typhus). Differential diagnosis: influenza, arboviral encephalitis, meningococcal infection. 23. Diagnosis: brucellosis. Differential diagnosis: tuberculosis, sepsis, lymphogranulematosis, pseudotuberculosis, reumatic fever, malaria. 24. Diagnosis: plague, bubonic form. Differential diagnosis: tularemia, sodocu, purulent lymphadenitis. 25. Diagnosis: infectious mononucleosis. Differential diagnosis: cytomegaloviral infection, felinosis, acute tonsillitis, localized form of diphtheria, acute HIV infection, leucosis, viral hepatitis A. 26. Diagnosis: tropical malaria Differential diagnosis: sepsis, typhoid fever, influenza, leptospirosis, hemorrhagic fevers. 27. : rabies, hydrophobic form. Differential diagnosis: tetanus, tick born encephalitis, delirium tremens. ANSWERS FOR TESTS FOR 6th YEAR STUDENS N A N A N n n s s . . 1 1 3 7 3 2 B 1 8 3 1 9 2 0 2 3 4 3 5 3 6 3 A N A N A N n n n s s s . . . 4 6 8 9 5 1 5 0 5 1 5 2 5 6 6 6 7 6 8 6 8 2 8 3 8 4 8 A N A N A N n n n s s s . . . 9 1 1 7 1 2 3 9 9 1 1 8 1 3 4 0 9 1 1 9 1 3 5 1 1 1 1 0 1 3 0 6 2 1 1 1 A n s .

82

1 6 2 2 2 3 2 4 2 5 2 6 2 7 2 8 2 9 3 0 3 1 3 2

7 3 8 3 9 4 0 4 1 4 2 4 3 4 4 4 5 4 6 4 7 4 8

3 5 4 5 5 5 6 5 7 5 8 5 9 6 0 6 1 6 2 6 3 6 4

9 7 0 7 1 7 2 7 3 7 4 7 5 7 6 7 7 7 8 7 9 8 0

5 8 6 8 7 8 8 8 9 9 0 9 1 9 2 9 3 9 4 9 5 9 6

1 0 1 1 1 2 1 3 1 4 1 5 1 6

0 1 1 0 2 1 0 3 1 0 4 1 0 5 1 0 6 1 0 7 1 0 8 1 0 9 1 1 0 1 1 1 1 1 2

1 7 1 1 8 1 1 9 1 2 0 1 2 1 1 2 2 1 2 3 1 2 4 1 2 5 1 2 6 1 2 7 1 2 8

3 3 1 3 4 1 3 5 1 3 6 1 3 7 1 3 8 1 3 9 1 4 0 1 4 1 1 4 2 1 4 3 1 4 4

A man of words and not of deeds is like a garden full of weeds. f) g) h) i) inflammatory changes; j) toxicoallergic inflammation of microvessels ; k) generalized destructive-proliferative thrombovasculitis; l) all mentioned. 468. What of the following human systems are most likely injured with HFRS pathogen? a) lymphatic and respiratory systems; b) respiratory and cardiovascular systems; c) cardiovascular and urogenital systems; d) urogenital and lymphatic systems; e) nervous and digestive system. 469. What type of affection of kidney is typical for hemorrhagic fever with renal syndrome? f) nephrosclerosis; g) pyelonephritis; h) glomerulonephritis; i) nephrosonephritis;

j) all mentioned. 470. What are the symptoms of climax period of the hemorrhagic fever with renal syndrome? f) hepatomegaly and fever; g) oliguria and hemorrhagic rash; h) vomiting and cough with purulent phlegm; i) anuria and hepatomegaly; j) diarrhea and vomiting. 471. What changes in common urine test is typical for hemorrhagic fever with renal syndrome? f) hematuria; g) crystaluria; h) leucocyturia; i) oxalaturia; j) piuria. 472. What is the main specific laboratory method for diagnostic of HFRS? f) skin-allergic test; g) antibodies detection in blood by a indirect immunofluorescence method; h) pathogen detection in blood by a method of the immunoenzym analysis; i) infection of laboratory animals (biological test); j) clinical analysis of blood. 473. What is the most typical complication of hemorrhagic fever with renal syndrome? f) edema of the braine; g) acute renalfailure; h) meningoencephalitis; i) peritonitis; j) edema of the lungs; 474. Which one from the following drugs may be recommended for treatment of HFRS in climax period? f) Panangine; g) Furazolidone; h) Prednisolone; i) Delagil; j) Thymogen.

83

HERPESVIRUS INFECTION 475. Which pathogens are not belong to herpesvirus group? (2) f) Hantavirus; g) HSV-1 and HSV-2; h) Cytomegalovirus; i) *Bunyavirus; j) Epstain-Barr virus; 476. What are the herpetic encephalitis peculiarities: f) intracellular insertions formation (Babech Negri corpuscles); g) demielinisation and fast development; h) hemorrhagic vasculitis; i) detachable neurons affection; j) disturbance of the acethilcholine synthesis in synapses. 477. What way of transmittion is not typical for HSV infection: f) contact; g) sexual; h) through the blood; i) transplacentar; j) parinatal. 478. What are the typical course and prognosis of the herpetic encephalitis: f) mild; g) moderate; h) severe; i) favorable; j) unfavorable; 479. What are the typical course and prognosis of the herpetic meningitis: f) mild; g) moderate; h) severe; i) favorable; j) unfavorable; 480. What is the main way of transmission of Varicella zoster infection: f) through blood; g) air droplet; h) alimentary; i) sexual; j) tick bites. 481. The Epstain Barr virus has tendency to affect: f) neurocytes; g) B lymphocytes; h) T lymphocytes; i) skin and mucous layers; j) hepatocytes. 482. What are the two main antiherpetic medications: f) ribavirin; g) acyclovir; h) arbidol; i) remantadin;

j) gancyclovir. 483. What is the main course of CMV infection clinical manifestations: f) vitamin insufficiency; g) overcooling; h) immunodifficiency; i) physical exhaustion; j) decayed food using. 484. CMV infection affects all mentioned below cells except: f) neutrophiles; g) monocytes; h) epiteliocytes of gastro intestinal tract; i) endocrine glands cells; j) B lymphocytes. 485. Clinical forms of CMV includes all manifestations except: f) retinitis; g) thireoiditis; h) uveltis; i) hepatitis; j) pneumonia. 486. Clinical forms of CMV includes all manifestations except (2): f) prolonged fever; g) endocarditis; h) esophagitis; i) nephritis; j) meningitis. 487. What kind of antibody in high titers shows the affection of newborn with CMV infection: f) IgA; g) IgG; h) IgM; i) IgS; j) IgF. 488. What are the medication for CMV infection: f) cricsivan; g) foscarnet; h) azidotimidin; i) ganciclovir; j) zovirax. 489. Human herpesvirus HHV 6 can be the agent of all mentioned diseases except: f) exanthema of newborns; g) chronic fatigue syndrome; h) meningoencephalitis; i) B cell lymphoma; j) All mentioned. 490. What two symptoms are not typical for herpes zoster: f) fever; g) polyadenitis; h) acute pain syndrome; i) vesicular rush on the skin streaming along dermatom; j) herpetic tonsillitis. 491. What are two indications for prescription of asiclovir in case of herpetic infection: f) herpes labialis;

84

g) varicella zoster; h) primary herpes genitals; i) mumps; j) encephalitis. 492. The Camp cells, typical for herpes infection are: f) gigantic multinuclear with intranuclear insertion; g) cells with dark intranuclear insertions, encircled with enlightenment zone (owls eye); h) cells with cytoplasmic eosinophilic insertions; i) multinuclear syncytium with necrosis of the nuclei; j) polynuclear syncytium with bazophil incertions. 493. What methods are most acceptable for herpetic encephalitis diagnostics: f) clinical and epidemic; g) typical cells detection in blood and urine samples; h) IgM antibodies detection; i) Monoclonal Immunofluorescence Reaction of brain cells; j) All mentioned. 494. A 56 year old female during the treatment of breast tumor noticed painful vesicles, in places with hemorrhagic content, on the lover part of the chest in the 9 10 ribs area, accompanied by fever 38C. a) What is the most likely diagnosis and form of the disease? b) What is the main cause of the disease? 495. A 5 year old child periodically suffers from vesiclous rush on the mucous of the mouth lusting for 3 weeks. Such rush was observed several times during lust year. a) What is the most likely diagnosis? b) What medications should be prescribed? 496. What is the mechanism of action of aciclovir: f) blocks viral nucleinic acid release; g) viral revertase inhibitor; h) viral proteinase inhibitor; i) cellular ribonuclease activator; j) DNA synthesis inhibitor; 497. The Hant syndrome is: d) combination of facial nervus paralysis, pain and vesicular rush in the ear area; e) combination of back passage and waterworks disturbances with pain and vesicular rush in the anal and genital area; f) combination of acute pain in the mandibular area with vesicular rush on the mucous of the mouth. 498. Herpetic infection is prone to affect all mentioned organs and tissues except: f) eyes mycoses; g) liver; h) suprarenal glands;

i) bone marrow; j) bronchial tubes. 499. Varicelloform eczema Kapoci occurs: f) accompanied by HIV infection; g) in aged persons; h) in a childhood together with eczema; i) in young parsons with allergoses; j) in all age groups together with helminthes affection. 500. What drugs can not be used for herpetic infection treatment: f) zoviracs; g) biovir; cicloferon; h) retabolil; i) thebrofen ointment; j) all mentioned. PLAGUE. TULAREMIA 501. What is the main way of transmission of bubonic form of plague? f) insect transmission; g) alimentary transmission; h) aerial transmission; i) horizontal transmission; j) livestock transmission. 502.What is the main way of transmission of cutaneus form of plague? f) insect transmission; g) water way transmission; h) aerial transmission; i) horizontal transmission; j) livestock transmission. 503. How many days are incubation period of plague? f) 1-6; g) 3-7; h) 5-10; i) 7-14; j) 11-21; 504. How many days are incubation period of tularemia? f) 1-6; g) 3-7; h) 5-10; i) 7-14; j) 11-21; 505. What of the following forms of plague is externally disseminated? f) secondary septic; g) primary septic; h) bubonic; i) secondary pneumonic; j) primary pneumonic. 506. What of the following form of plague is localized? f) secondary septic; g) primary septic; h) bubonic; i) secondary pulmonic; j) primary pneumonic. 507. What of the following drugs are used for

85

emergency prevention in case of contact with a plague? f) erythromycin; g) penicillin; h) polymycxin; i) streptomycin; j) vancomicin. 508. What of the following group of lymph nodes are most likely affected with bubonic plague? f) cervical; g) inguinal; h) cubital; i) mesenteric; j) all mentioned. 509. What of the following group of lymph nodes are most likely affected in case of anginous form tularemia? f) cervical; g) inguinal; h) cubital; i) mesenteric; j) all mentioned. 510. In what period of pneumonic form of plague the patients are most dangerous? f) incubation; g) initial; h) climax; i) reconvalescention; j) no one from mentioned. 511. What clinical form of plague more often registered on a beginning of epidemic? e) primary pulmonic; f) bubonic; g) primary septic; h) cutaneus; 512. What is most often cause of death in case of septic form of plague? f) acute renal inflammation; g) acute respiratory inflammation; h) inflectional toxic shock; i) brain edema; j) edema of lungs. 513. What is the first step of management to patient with bubonic plague? f) prescription of immunoglobulin; g) local bubo therapy; h) desintoxication therapy; i) antibiotic prescription; j) dehydratation therapy. 514. What is the main method of plague treatment? f) antibiotic therapy; g) serotherapy; h) desintoxication therapy; i) dehydratation therapy; j) rehydratation therapy. 515. On examination of patient doctor has suspected a plague. What of the following facilities should be informed first? f) city center of state sanitary epidemiological surveillance; g) regional center of state sanitary epidemiological surveillance;

h) head of current medical facility; i) police office; j) relatives of a patient. 516. What skin phenomena are typical for skin form of plague: f) Affected places of skin are sharply outlined, jelly like fluctuation of skin during shaking. On center of edema painless black ulcer 34 sm. with yellow purulent edge. In outer zone of hyperemia - vesicula, filled by dark red contents. g) Skin is hyperemic with cyanotic shade, edema is moderate. On centre of edema black ulcer, 11 sm. without purulent content. Edema and ulcer are sharply painful. h) Moderate edema of skin, in centre of edema zone is crater - like ulcer under grey crusta, pus is excreted during pressing. Highly expressed infiltration, lymphadenit and lymphangit are presented. Touching to edges of ulcer is painful. i) Skin is markedly hyperemic, shining, form is smoothed, a zone of affection with precise festooned edges and a platen on end. On center affection hyperemia is less expressed, than on edges.

86

ANTHRAX 517. What is the source of the anthrax infection? k) livestock; l) man; m) insects; n) river fish; o) birds. 518. What inoculation way is most typical for anthrax? a) mucosa of respiratory tract; b) injured skin; c) mucosa of gastrointestinal tract; d) conjunctiva; e) all mentioned. 519. What kind of local inflammation is typical for anthrax? a) fibrinous; b) productive; c) serous - hemorrhagic; d) suppurative; e) granulematous. 520. What is contents of anthrax carbuncle cavity? a) serous liquor; b) pus; c) blood; d) all mentioned. 521. What is the main factor of anthrax pathogenesis? a) autoallergy; b) immunosuppression; c) toxemia; d) all mentioned; 522. What of the following are characterized ofr anthrax carbuncle? a) edema of tissue and acute pain; b) edema and painless of tissue; c) absence of tissues edema and acute pain; d) absence of tissues edema and pain. 523. What is the main method of anthrax diagnostics? a) bacteriological; b) biological; c) serological; d) bacterioscopy; e) allergiological; 524. What of the following is typical for anthrax regional lymphadenitis? a) painful lymph nodes and rapid purulence; b) painful lymph nodes and absence of purulence; c) painless lymph nodes and absence of purulence; d) painless lymph nodes and rapid prolypheration; e) all mentioned. 525. What is the most typical variant of cutaneous form of anthrax? a) erysipeloid - like; b) edematous; c) bullosis; d) carbunculosis;

e) bubonic. 526. What is the basic treatment method of cutaneous anthrax? a) bandage with ointment by Vishnevsky; b) antibacterial therapy; c) opening carbuncle and drainage; d) surgical revision of carbuncule; e) supportive therapy.

87

BRUCELLOSIS 527. What kind of brucellas are most pathogenic for human? f) bovis; g) suis; h) melitensis; i) ovis; j) canis; k) neotomae; 528. What factors influence on duration of incubation period of brucellosis? f) Pathogen activeness; g) Quantity of infecting dose; h) Way of transmission; i) Reinfection; j) All mentioned. 529. Which of the following is the reaction of sensibility tissue with specific brucellar antigen? a) reaction of Coombs; b) Wrights reaction; c) Raection of Heddlson. d) Burnets reaction. e) All mentioned. 530. What is migration of brucells: e) transfer of brucells from one organ to another of main host; f) transfer of brucells from one animal to another of the same species; g) transfer of brucells from main animal host to another of different species; h) all mentioned. 531. Inoculation gates of brucells into human organism: f) damaged skin; g) mucous; h) gastrointestinal tract; i) all mentioned. 532. What methods for diagnostics of brucellosis: f) Bacterioscopy; g) Bacteriological method (cultivation); h) Serological method; i) Allergic test; j) All, except bacterioscopy; 533. Is there a cross immunity after infection with different types of brucells possible? f) yes; g) no. 534. What are the sources of brucellosis? f) Affected human; g) Main hosts of different types of brucells; h) Another animais, affected with migrated brucells; i) Rodents; j) All, except human. 535. What are the ways of brucellosis transmission? f) Food borne; g) Air borne; h) Through damaged skin and mucosa;

i) Vertical; j) All, except vertical. 536. What are basic clinical symptoms of climax period of acute brucellosis: f) Constant remitting fever, chills, significant sweating, polylymphadenopathy, muscular pain, spleen and lever enlargement; g) Constant type of fever, weakness, adynamia, pale skin, poor roseolar rush on lateral surfaces of trunk, spleen and lever enlargement; h) Fever of remitting type, chills, significant weakness, petechial bleedings on conjunctiva, sometimes liver enlargement; i) Fever of intermitting type, on climax of fever fit feeling of heat with subsequent plentiful sweating, spleen and lever enlargement.

88

Q-FEVER 537. What is the drug for specific therapy of Q-fever? a) penicillin; b) tetracycline; c) furazolidone; d) streptomycin; e) cloroquin. 538. What cardiovascular disturbances are typical for climax period of Q-fever? f) tachycardia and hypotonia; g) relative bradicardia and moderate hypotonia; h) bradicardia and hypertonia; i) tachycardia and hypertonia; j) all mentioned. 539. How many days are maximal duration of Q -fever incubation period? f) 5; g) 14; h) 25; i) 45; j) 50 540. What is the basic method for specific laboratory diagnostic of Q - fever? f) bacteriological blood investigation; g) serum blood investigation; h) skin-allergic test; i) bacteriological investigation of faeces; j) bacteriological investigation of cerebrospinal liquor. 541. What of the following system are typically affected with Q-fever? f) nervous system; g) cardiovascular system; h) respiratory system; i) alimentary system; j) all mentioned.

89

TOXOPLASMOSIS 542. For what of parasitoses a cat is a main source of invasion? f) Amebiasis; g) Balantidiasis; h) Lambliosis; i) Toxoplasmosis; j) Pneumocystosis. 543. For what form mentioned pathogens transplacentar transmission is possible? f) Toxoplasma; g) Lamblia intestinalis; h) Balantidium; i) Coccidium; j) Negleria. 544. Where does the intestinal phase of development of toxoplasm occure? f) in human; g) in pigs; h) in large horned cuttle; i) in cats; j) all mentioned. 545. What is the source of toxoplasmosis infection? a) human; b) animals from feline family; c) large horned cuttle; d) pigs; e) insects. 546. What does the life cycle of toxoplasma development in the human organism include? a) sporogony; b) endodiogeny (internal budding); c) gametogony; d) schizogony; e) all mentioned. 547. What kind of the immunity from toxoplasmosis? a) stable, sterile; b) unstable, sterile; c) stable, unsterile; d) unstable unsterile; e) all mentioned. 548. In what form toxoplasms are kept in the organism of human in latent toxoplasmosis? a) oocysts; b) pseudocysts; c) tachyzoites (endozoites); d) tissure cysts; e) chizonts. 549. What is the drug for specific treatment of toxoplasmosis? a) chloridin (pyrimethamin); b) penicillin; c) chloroquine (delagil); d) metronidazole (trichopol); e) levomycetin.

90

SEPSIS. 550. Choose possible causative agent of sepsis from following: a) Sh. Sonnei; b) Str. Pneumonie; c) Fr. Tolarence; d) R. Prowazekii: e) S. Enteritidis. 551. What is the main route of transmission of sepsis? f) Air-droplet; g) Alimentary; h) Transplacentar; i) Contact; j) From supportive foci of organism. 552. Is recovering from sepsis possible without proper treatment? e) Yes, due to formation of stable immunity; f) Yes, unfavorable outcome possible only among children and old persons; g) No; h) Yes, unfavorable outcome possible only in HIV infected persons. 553. Clinical sings of sepsis are all following except: f) Petechial rash; g) Jaundice; h) High long temperature; i) Frequent liquid stool which contain mucus and blood; j) Hepatosplenomegaly. 554. Express methods of sepsis diagnosing is: f) Bacterioscopy of thick drop of blood; g) Blood culture; h) Serological methods; i) Immunofluorescent test; j) Allergic test. 555. In case staphylococcal sepsis the most effective antimicrobial drug is: f) Penicillin G in high dose; g) Cephalosporins of III generation; h) Vancomycin; i) Macrolides of III generation; j) Tetracyclines. 556. The main complications of sepsis are all except: f) ARDS; g) Septic shock; h) Acute renal failure; i) Edema of the brain; j) Degidrative shock. 557. Refractory septic shock it is condition when: f) Infusion therapy is not effective; g) Complex antibiotic treatment is not effective; h) Glucorticoides are not effective; i) Immune therapy is not effective. j) Infusion therapy and corticoids are not effective 558. Most mediators of inflammation act as: f) Activators of immune system; g) Vasodilatators; h) Suppressors of immune system; i) Factors increasing permeability of microvessels;

j) Vasodilatators and increasing permeability of microvessels. 559. Among the drugs which act as immunocorrectors in case of sepsis is most effective: a) Timalin; b) Interferon; c) Ronkoleikin; d) Inductors of endogenous interferon (amixin) e) T-activin.

TESTS AND CLINICAL CASES FOR 6TH YEAR STUDENTS

CLINICAL CASES CASE 1 The patient ., 44 years old, has admitted for 10-th day of disease with complaints on common weakness, irritability, loss of appetite, jaundice, darkening of urine. The disease began gradually with fatigue, nausea. On 4th day has noted darkening of urine, the feeling of weight in the right appear part of abdomen has appeared. Current state had connected with exhaustion. On 8th day the jaundice has appeared, the feces has brightened. Was guided to hospital by general practitioner. On examination: common state is moderate, temperature 36,8C. Moderate jaundice of mucous and skin was observed. Heart and lungs without singularities. Pulse - 60 1/min.. BP - 115/70 mm hg.. Tongue is a little edematous, dryish, coated by brownish fur. Meteorism of I degree. Liver was enlarged on + 2 sm., edge acute, surface was smooth, painful during palpation. f) What is the diagnosis? g) Give a plan of laboratory examinations and treatment. h) What outcomes of disease are possible. i) Make a differential diagnostics. CASE 2 The patient ., 35 years. Has admitted in clinic in a severe state, unconsciousness, periodically clonic cramps. From accompanied persons was found out, that patient was on business trip in Guinea. Come back one week ago, at once fell ill. All days before had a temperature, kept on high figures (39-40C) and only on mornings sometimes normal. Patient was complaining on sharp headache, chilling, feeling of fever. Had taken Analginum, but the state was not improved: the headache accrued, the sleepiness has appeared, and then had lost consciousness, the cramps began, excitement have appeared. On examination: skin and visual mucous slightly icteric, respiration of Chain Stocks, tachycardia. Pulse - 130 /min., weak filling and strain. BP - 90/40 mm hg.. Mild

91

wet variegated crepitating. The lien is palpated indistinctly. Pupillary reactions are flaccid, miosis, tendinal hyperreflexia. A mild rigidity of a neck muscles. k) What is the diagnosis? l) What is the form of disease? m) What are urgent states of current disease? n) Make a diagnostics and treatment plan. CASE 3 Patient fall ill acute with rising of temperature up to 39,0C, shill, headache in frontotemporal areas, filling of general unwell, muscle and joint pains. On the second day has joined block of nose breathing, moderate excretions from a nose, dry often cough, feel tickle in a throat and pain in throat and behind a breast bone during coughing. On examination by the doctor on 3-th day of disease temperature 38,2 C, injection of vessels of a conjunctiva and blepharons, scleras and soft palate hyperemia was observed. On auscultation - hard respiration, simple dry crepitation. a. What is most likely diagnosis? b. The plan of laboratory examination. c. What urgent states are characteristic for a current disease? d. What medical tactics are preferable? CASE 4 The patient 39 years old, admitted to the clinic of infectious diseases on 12th day of the disease after acute onset with startling shill, which was replaced by feeling of fever, temperature has raised up to 39,5C with indications of intoxication for 6 hours and critically decreased up to 36,5C with massive sweating. The attacks are repeated every 3-rd day. On examination: herpes labialis, sub icterus of scleras. The cardiac sounds are weak. The moderately expressed hepatolienal syndom was observed. In the clinical blood analysis moderate anemia, leukopenia, lymphomonositosis, ESR - 21 mm / hour. e. What is the diagnosis, laboratory diagnostics plan. f. Prescribe a treatment. g. What are possible urgent states during treatment? CASE 5 The patient 22 year old, student, admited in clinic on 4-th day of disease, with the complaints on general weakeness, spasmatic pain in abdomen, frequent liquid stool with slime and blood. The disease began from shill, headache, tenesms and a liquid stool. Patient lives in isolated apartment. All relatives are healthy. Week before disease has come back from village, where were cases of similar disease. A common

state is moderate. Temperature is 37,5C. A skin pale, tongue wet, coated with white fur. The cardiac sounds are weakened. The abdomen is moderately inflated, painful during palpation of thick sigmoid intestine. Stool up to 10 times per day, liquid, with slime and blood. h. Formulate and prove the clinical diagnosis. i. Prescribe laboratory diagnostics for acknowledgement of the diagnosis. j. What are possible complications and urgent states during the current disease? CASE 6 The patient 50 year old, watchman of a depot posed on edge of a forest. The disease began from rising of temperature up to 39C, shill, headache. From 2-th day of disease - back pain, from 3-rd - nasal bleeding. On 5-th day of disease, on date of entering in clinic, state severe - delirium, tremor of limbs, hyperemia of the face and neck. An abundant hemorrhagic rush on skin of a trunk and extremities. Oliguria was observed. k. Make your preliminary diagnosis, substantiation. l. Diagnostics. Possible urgent states? m. Medical tactics. CASE 7 The patient ., 25 years old, was admitted in infectious hospital on 2-th day of disease with the complaints on headache, back pain, in calf muscles, shivering. His state was moderate, scleras subicterical, mucous of pharynx is hiperemic. Tongue dry, coated with brown fur. The stomach is inflated. The liver is 2 sm lower than edge of a rib arc. A lien at edges of a rib arc. The palpation of muscles, especially calf is painful, dark urine, feces of usual color. a) Mention urgent states, which appropriate for this pathology. b) Their diagnostics and treatment. CASE 8 The inhabitant of the Tyumen area (Russia), frequently used a fresh fish, home made solty pike caviar. Within last months notes common weakness, dizziness, pain in tongue, muscles, weight loss and excretion with a feces particles of helminths. The scin is pale, edemas on shin and loins. The cardiac sounds are weakend, systolic murmur on the apex. BP 100 / 70 mm. hg. The liver enlarged on 2 sm. Tongue is bright red, with cracks. The analysis of a blood: erythrocytes - 1,3 * 1012 1/, haemoglobin - 50 g/l, CI - 1,2, in sample - megaloytes, poikilocytes, erythrocytes with corpuscles of Joli and rings of Kebot. a) What is most likely diagnosis? b) Make diagnostics and treatment plan.

92

c)

Complications. Their diagnostics and treatment.

CASE 9 Female patient, 45 year old, works as a train conductor. Was admitted on 10-th day of disease with the diagnosis of peritonsillar abscess which was by the otolaryngologist, but suppuration was not obtained. The disease began gradually from sore throat, then the rasping tussis accompanied, hoarse of a voise, further aphonia, noisy labored respiration has joined. Was admitted to hospital in a severe state. Cyanosis, dyspnea with participation of an auxiliary muscles. Frequency of respiration 52 /min., sweating, paroxysmal tachycardia. Cardiac sounds dummy. Periodic attacks of asphyxia, cyanosis, edema and hyperemia of a soft palate, tonsils; on tonsils - firm grey film. During laryngoscopy fibrinous grey color film was detected of a on a mucous of larynx. From Throat and the larynx dense film were extracted. a) Give your diagnosis, plan of diagnostics and treatment. b) What are possible complications, their diagnostics and treatment. CASE 10 The male patient, 36 year old, track driver, was admitted in a severe condition with complaints on headache, sleeplessness, impossibility of swallowing of liquids. From the anamnesis was found out that the last year on hunting has caught the fox, has brought it to the home, but it has escaped. There was no bites, but multiple scratches. The persons face hiperemic, with ingection of scleras, hipersalivation. Complaints on irritability from light and driving of air. Movement of the lower extremities labored; an abundant sweating. Periodic convulsive spastic of throat muscles and larynx, the respiration labored. exophtalm. Pulse -160 1/min. BP elevated. The noise of pouring water and movement of air is causing convulsive attacks, between them - the patient is adequate. Asks about the help. a) Make your diagnosis. b) What phase of the disease? c) The plan of laboratory diagnostics? Medical tactics? d) With what it is necessary to differentiate? e) Outcomes of current disease? CASE 11 The male patient G., pupil, is admitted in clinic of infectious diseases on 3-th day of disease , with the complaints to lack of motions in the lower extremities and sharp pain in muscles, high temperature, which began to reduce. However, with drop of temperature the pain in the lower extremities, loins, arms and neck has aggravated. Patient in consciousness. Within several hours for 3-th day of disease the flabby paralyses have appeared in the muscles of legs, arms with a dominance

of a proximal affection of extremities (femur, shoulders). The function of sphincters is reduced. The atony of muscles, areflexia has joined together with coldening and cianosis of legs. The sensitivity is saved. f) Make your diagnosis accounting phase of disease. g) The differential diagnosis? h) The plan of diagnostics? i) What complications are possible? j) Your therapeutic tactics? CASE 12 The male patient ., 48 year old. Works in a wool reception depot. Was admitted to clinic on 2 - th day of the disease in severe condition with complaints on dyspnea, expectoration of serum - hemorrhagic sputum (about 1 l/day), pain in a thorax, connected with breathing. The disease began suddenly with chill, weakness, the dry cough, then with expectoration of a great amount of serum and serum - hemorrhagic sputum. It is known from epidemic anamnesis, that during his work in a depot, accepted and unpacked bales with a wool delivered from other countries. Among the employees in a place cases of influenza took place. In the past history had had only common cold. On examination: general condition was serious, consciousness saved, but sometimes patient is inadequate. Skin without eruption, wet. A mucous of nasopharynx slightly hyperemic. Tongue was dry, coated with dirty fur. Respiration through a nose and mouth 40 / min. On percussion above lungs, in interscapular area shortening of a pulmonary sound, on auscultation there are a lot of dry and varied wet crepitation. Pulse - 120 /min.. Blood pressure - 80/40 mm hg.. The lien at edge of a rib arc. e) What is preliminary diagnosis? f) Tactics of the doctor in the anti epidemic ration? g) The plan of examination. h) What complications are typical for the disease? i) Medical measures. CASE 13 The patient ., 18 year old, has admitted in clinic on 8 - th day of the disease with the complaints to common fatigue, headache, pain in throat during swallowing, high temperature, sweating. The disease began with malaise, subfebrile temperature, pain in a throat. Frequent tonsillitis in past history. On examination: temperature 37,7C. Paleness of scin, without a rush. Mucous of throat moderately hyperemic. The tonsils enlarged in sizes, oedematous, without purulent corks, films. Axillary, submandibular, ugular lymph nodes were observed, size 0,8 1 sm., slightly painful, mobile. Cardiac sounds clear, rhythmic, pulse - 94 /min.. On auscultation vesicular sound. Tongue wet, coated with white fur. Abdomen soft, painless. The liver enlarged on 1 sm. Physiological excretions are normal.

93

f) Substantiate the diagnosis, laboratory and differential diagnostics. g) What complications are possible? h) Medical tactics? CASE 14 Female patient D., 30 year old. nurse of a children local clinic department, was admitted to infectious diseases hospital on 3 - th day of the disease with the complaints on pain in throat, high temperature and frequent liquid stool. 9 patients of teenage age from improving camp with similar symptoms were hospitalized in department. Bright hyperemia of mucous of throat and soft palate, palatine arcs, tonsils and back wall of a pharynx. On a mucous of soft palate group shallow vesicles with transparent contents, enclosed by a red crown, which in 2-3 days bursted with formation of erosion. The pain in throat were strengthened during swallowing. In 7 days the fever has decreased, mucous epitelisation was observed. General condition was improved, stool normalized. e) What is most likely diagnosis? f) The differential diagnosis? g) Methods of laboratory diagnostics? h) What complications are possible? i) Medical tactics. CASE 15 Female patient G, 38 year old admitted in hospital on 4 - th day of disease with the complaints on rising of temperature to 38,3C, headache, dizziness, nausea, weakness, pain in the left eye, massive suppuration from eye, tumescence and pain in the field of left ear and neck. Anamnesis of disease : Onset was acute: temperature up to 38 C, headache, weakness. On second day the dizziness has joined, a nausea, and to the end of 3 day has appeared pains in left eye, swelling of eyelids, next day - purulent discharge. Then patient noticed swelling and pain in the field of left ear and neck. Was hospitalized in a hospital. Past history - without singularities. Anamnesis epidemica: there was no contact with infectious patients, with animal also. Last 2,5 weeks work with threshing-mashine. On examination: moderate condition. Scin of face hyperemic, dry. Temperature 38,3C, S- 90 ./min., rhythmic, satisfactory qualities. Eyelids of the left eye dropsical, in an angle of left eye - white pus. The conjunctivas is sharply dropsical, plenty of small ulcers with purulent content. Asymmetry of the face and neck at the expense of enlarged subauricular and neck lymph nods, morbid, tough, with clear eges and movable. Mucous of throat are moderately hyperemic. Tonsils are not enlarged. Heart - tones clear. Respiration vesicular with a hard nuance. Tongue dryish, coated with fur. A stomach soft, painless during palpation. Liver and lien are not palpated.

The clinical blood test: red cell count - without singularities, white - neutrophil leukocytosis, ESR 25 mm/hr.. c) What is most likely diagnosis? d) Methods of laboratory diagnostics and therapy? CASE 16 Patient , 44 years old, was admitted to the hospital at the end of May with the complaints on a strong headache of pulsating character, photophobia, general muscle pains, chill, fever, sleeplessness, lack of appetite. 3-rd day of the disease. The disease began from a mild malaise, headaches, rising of temperature up to 37,4C, vomiting, photophobia, doesnt take any medicine. Anamnesis epidemica: 8 days before has come back from Western Siberia (Abakan district), lived there in a wood during 10 days. There were no contacts with ill people, animals, mosquitoes, ticks bites. OE: condition is moderate. The contact is entered well, consciousness clear, temperature 39,0C; pulse - 80 1/min., rhythmic, the cardiac sounds are muffled. On lung examination vesicular breathing. Tongue is dryish. Abdomen soft during palpation, painless. A liver, lien are not enlarged. Mucous of the throat hyperemic. A left-sided neck lymphadenitis was observed. In left parietal part of the head - a crust of brown color, a band of a tumescence around it, the palpation is painful. Neurologic status: pupillary reactions on light are sluggish, the convergence is broken. Mild assymetria of the face, slight rygudity of neck muscles. a) Your presumable diagnosis? Its substantiation? b) Therapeutic tactics? CASE 17 Patient M, 40 years, metal-turner of a factory, was admitted to the infectious hospital 4.06.79. with complaints on headache, strong weakness, temperature, pain in joints, inching abundant rush on skin. One week ago has received a wound in the field of the right foot joint, with the preventive purpose antitetanic serum was injected. On ehamination: 7th day of the disease : temperature 38,2C. Fase is dropsical and pale. The enlarged submandibular, cervical and axillar lymph nodes. Fingers of hands are dropsical. On a skin of trunk, arms, abdomen and lower extremities the maculo-papular pruritic rush. CVS: tachycardia, hypotonia. In a blood analysis: leukopenia with a relative lymphocytosis and accelerated ESR. a) Your diagnosis and tactics of treatment of the patient? CASE 18 Patient G, 38 years was admitted to the hospital on 4 th day from beginning of the disease with complains on temperature ap to 38,8C, headache, dizziness, nausea,

94

pain in the left eye, abundant suppuration from eye, tumescence and pain in region of left ear and neck. Anamnesis of disease : Onset was acute, with temperature up to 38C, headache, weakness. On the second day the giddiness has joined, a nausea, and to the end of 3th day has appeared pain in eye, tumescence of blepharons, on 4th day - a suppuration. Then patient has noted swelling and pain in the field of left ear, necks. Was admitted to the hospital. Anamnesis of life - without singularities. Anamnesis epidemica: there were no contacts with infectious patients, animals. During last 2,5 week worked with threshing machine. OE: a common state is moderate. The skin of the face is hyperemic, dry. Temperature 38,3C, uls- 90 1/min., rhythmic, satisfactory qualities. Eyelids of left eye hydropic, in an angle of an eye - white pus. The conjunctiva is sharply hyperemic, large number of ulcers with suppuration. Asymmetry of the fase and neck at the expense of enlarged auricular and anteriocervical lymph nodes, painful, tight, good contoured, relative frame, mobile. Mucous of throat is moderately hiperemic. Tonsils without significance. Heart - tones rhythmic, clear. Respiration: vesicular with a hard nuance. Tongue is dryish, coated with white fur. Abdomen soft, painless. A liver, lien are not enlarged. The analysis of a blood: red - without singularities, white -neuthrophil leukocytosis, elevated ESR. a) What is the diagnosis? b) Methods of laboratory diagnostics and therapy? CASE 19 The patient , 32 years, admitted to hospital after acute onset of the disease with pharyngalgias and painful swallowing, temperature 39,4C, headache. Penicillinum and sulfodimetoxin were prescribed. Soon the patient the felt a numbness of tongue, lips, face; giddiness, headache, nausea, shortness has appeared. On examination of the patient: the face is hyperemic, sweated, acrocianosis macular inching rush. Pulse often, thread-like. BP - 40/0 mm hg. Cardiac sounds dummy. Accent of 2th tone on a pulmonary arteria. The respiration often and superficial, later noisy also was listened apart. In lungs box-like pulmonary sound, during auscultation - dry, whistling crepitation. Patient excited, rushes in bed. A consensual urination and act of defecation. c) What is most likely diagnosis? d) Make a treatment plan? CASE 20 Patient ., 37 years, has complaints on weakness, sweating, pain in knees, foot joints. Temperature 37,6C. Anamnesis of disease: consider himself ill about a months. During this period bothered weakness, pain in large joints. Repeatedly sought for a medical help, was treated, but without effect. Anamnesis of life: a pneumonia, ARVI.

Anamnesis epidemica: patient a veterinarian by profession. Had contact with a ill animal - sheeps during lambing. Objective examination: temperature 37,6 C. A common condition satisfactory. Skin and visual mucous of usual colouring. Edema, morbidity of large joints was found; in the field of joints in a hypodermic fat, tendons, muscles the dense painful nodules are palpated. Vesicular sound on auscultation. The cardiac sounds rhythmic, dull. Pulse - 80 1/min., satisfactory qualities. BP - 120/80 mm. hg. Stomach during palpation soft, painless. The edge of a liver is palpated on 1,5 sm below of a rib arc. The lien is not enlarged. Stools and urine passage are normal. d) Make your presumable diagnosis? e) Make a plan of diagnostics? CASE 21 Patient K, 21 year old, school teacher was admitted to hospital on 5th day of the disease after acute beginning with fever up to 38,5C, running nose, frequent dry cough, fear of bright light, epiphora. After treatment at home temperature decreased to 37.5C. On 3rd day condition deteriorated, temperature increased to 39.5C, headache and nausea intensified; the same day rush raised on face, neck, upper part of thorax and then spread on whole body. On admission: temperature 39.5C, patient is languid. Face and eyelids are dropsical. Conjunctives are hyperemic, injection of sclera is observed. On skin of body and face lavish macular - papular rush, sometimes merged. Throat and palatal mucosa are hyperemic. On chick mucosa, opposite molars, spots of tender white fur observed. During auscultation rough breathing was found. Cardiac sounds are moderately dull, pulse is 92/min.. What is your diagnosis? b) Make a differential diagnosis. c) What complications are possible? d) First step treatment measures.

CASE 22 Patient K., 61-year-old pensioner, was admitted to hospital with complaints on headache, insomnia, chill, fever 39.3C, and weakness. Fell ill 5 days before, was treated by general practitioner with antipyretics and anti-inflammatory drugs. Today on skin of trunk roseolar and petechial rush rose. From past history was known that during II World War in partisan detachmen had louse-borne typhus (30 years ago). OE: General state moderate. Face skin is dropsical and hyperemic; excitement, glitter of eyes, moderate cyanosis of lips, pinch symptom is positive. On palatine mucosa fine enanthema was observed. Moderate dyspnoe. On lung auscultation vesicular breathing. Cardiac sounds are dull, systolic murmur on top projection. PB is 100/60 mm hg. Tongue is coated with

95

fur, trembling during examination. Abdomen is soft, painless, lever and spleen enlarged on 2 sm. e) What is the diagnosis? f) What complications are possible? g) Make diagnostics and treatment plan. CASE 23 Patient I. ., 35 years, was admitted in hospital with complaints on sweating, joint pain, weakness, headache, temperature 39,5C, chill. Anamnesis of disease: fell ill about 2 weeks ago. The disease began from a headache, weakness, sweating, temperature 37,5- 38,0C with chill. Each time chill and rising of temperature was finished with a profuse sweating. Paroxysms of temperature appeared in the afternoon. On 5th day of disease visited the doctor. Was treated with Aspirin, Analgin, but without effect. After second visit was hospitalized. Despite of remitting temperature with peak in the afternoon, common condition of the patient remained rather satisfactory. Anamnesis of life: without singularities. Anamnesis epidemica: during one year works on a farm as a milkmaid, had contact with the ill animals during calving. OE: common condition is satisfactory. Temperature 38C. The dermal integuments of the person are a little acyanotic, the eruption misses. The lymph nodes cervical, axillary, magnitude about a string bean, painless are palpated. On auscultation: lungs without singularities. The cardiac sounds rhythmic, are muffled. Pulse 80 /mines., satisfactory qualities. Tongue wet, pure. The stomach at a palpation weak, painless, is palpated edge of a liver of a below rib arc on 1. The lien is not palpated. A stools and waterworks is normal d) Your diagnosis? e) The plan of treatment? CASE 24 The young hunter, 25 years, was taken to a hospital in a severe state expressed with temperature 39,0C, repeated vomiting, sharp hyperemia of the face, acrocianosis, confused consciousness. From words of accompanied his wife was established, that the patient fell ill 3 days before. Onset of the disease was acute with chill, headache, vomiting, pain in muscles, especially right lower extremity. Temperature during a first day was 39,0C. The hallucinations have appeared. Was admitted in a hospital with a passing auto. All surrounding people were healthy. As a part of his work, majority of time he spent on a nature - in a wood, in steppe. Was accurately vaccinated. Objective examination: pulse - 140 1/min., weak filling. BP - 80/40 mm. hg. Cardiac sounds dull, RR - 26 in min.. Vesicular respiration above a lungs. Tongue was coated with fur. The abdomen soft, takes part in an act of respiration, does not react to palpation. The right leg was half bend. In inguinal area the formation with indistinct contours 15 25 sm. was visually defined. Skin above it was blue - crimson color. During

palpation of this conglomerate the patient reacts with a groan, tries to repel a hand of the doctor. d) What is your diagnosis? e) Make your tactics of treatment?

CASE 25 Patient G. B., 21 years, was taken in hospital on 7-th day of disease with complaints on chill, high temperature, headache, common weakness, sore throat. Onset of the disease was acute from rising temperature up to 38C and pain in a throat. At home was treated with Tetracyclin, Aspirin, but the condition without improvement. No experience of previous diseases. Anamnesis epidemica: quiet. On examination - state was moderate. Patient was sluggish. Skin was pale, without rush. Mucous of a throat is hyperemic, tonsils enlarged, crumbly with films of green-yellow color. Submandibular and back of neck lymph nodes were enlarged, size about a string bean, slightly painful during palpation. Pulse was 108118 1/min., rhythmic. The cardiac sounds are dull, tachycardia. In lungs harsh breathing on auscultation. Tongue was dryish, coated with white-grey fur. Abdomen was soft, a little painful during palpation in right hypochondrium. Liver at a level of a rib arc. Stool, urination were normal. The clinical analysis of blood: Eryth. 4 10 12/l, Leuk. 10 109 l, dr/st.- 8 %, cegments 52%, lymphs. - 31 %, monoc - 6 %, plasm. cells 2%. ESR -27 mm/h. Hemogram in 4 days: Leukoc. - 109 l, eosynoph. 0%, dr./st. - 8%, segment. - 47%, lymphs. - 38%, monoc. 9%, ESR - 32 mm/hr.. Lymphocytes with wide protoplasm. Pain in throat decreased in 5 days after admission have disappeared, temperature normalized in 15 days. To the moment of discharge 29th day of disease - the patient complained on fatigability, giddiness. In a haemogram limphomonocitosis with wide protoplasm lymphocytes. f) What is your diagnosis? g) What is your therapy?

CASE 26 Patient S, 35 years. Was admitted in clinic in a severe condition, the unconsciousness, periodically clonic cramps. In previous was on business trip in Guinea. Come back to Ukraine one week ago and fell ill at once. All days before entering temperature kept high (39,040,0C) and only in mornings sometimes normal. Sharp headache, chill, feeling of fever troubled. Patient was taking Analgin, but without effect: the headache accrued, sleepiness appeared, and then consciousness was lost, cramps, exaltation appeared. Anamnesis epidemica. In Guinea one month ago had a malaria, probably one month ago. OE: skin and visual mucous slightly subicterous, respiration of Shain - Stoks. Tachycardia, pulse - 130

96

1/min., weak filling and strain, BP - 90/40 mm. hg.. On auscultation of lungs - variegrated wet crepitations. The lien was palpated not legiblly. Pupillary tests sluggish, miosis was observed. Hyperreflexia of tendinal reflexes, mild rigidity of a neck muscles. a) What is your diagnosis? b) What is the form of disease? c) What is your therapeutic tactics?

CASE 27 Patient I. I., 26 years, was admitted in hospital in a severe condition with complaints on weakness, difficulty of swallowing a saliva and water, anxiety, temperature 38.8C. Anamnesis: Fell ill about 3 days before, when weakness, temperature 38C, then difficulty of swallowing of a saliva, and then water, hypersalivation has appeared. During first 2 days not appealed to doctor. On 3-rd day of disease the state has worsened sharply: appearance of water, driving of air have provoked spasts of pharynx, larynx. Anamnesis of life: without singularities. Anamnesis epidimica: During last 4 months before admission in hospital, during hunting was bitten by the fox for the right hand. Didnt received any vaccination. Objective examination: Condition of patient was severe, temperature 39.8C, hyperemia of a face. Skin was wet. In a reception ward the attack was developed, that was accompanied by motive exaltation, crick of muscles of a pharynx and larynx, abundant hypersalivation, disorder of respiration. With cry throws from itself a cup of water, throws back head and trunk. The face was distorted, cyanotic, expresses horror, the pupils dilated, view stuck in one point. The attack last some seconds. Then the attack has repeated as a result of a loud sound and switched on light. Lethal outcome had occurred during the fit. a) What is your diagnosis?

97

CLINICAL TASKS 1. Chose symptoms, typical for icteric form of viral hepatitis B: a) Gradual onset from nausea, gradual loss of appetite, increasing itch of skin. OE: jaundice of skin, with scratches traces. Skin turgor is reduced. Pulse is 86 /min.; liver sizes by Kurlov: 13128 sm, in gall - bladder projection elastic painless formation 33 is palpated. Spleen is not enlarged. b) Acute beginning from nausea, pain in a right part of abdomen, fever up to 38C. OE: moderate jaundice; tachicardia up to 104 /min. Liver sizes by Kurlov: 12127 sm., palpation of gall bladder is painful. Spleen is not enlarged. c) Beginning gradual from nausea, pain in joints, itch of skin, and loss of appetite. OE: skin is icteric, turgor is normal. Pulse is 68 /min.. Liver sizes by Kurlov: 13128 sm., edge of lever sensitive. Spleen is not enlarged. d) Acute onset with chill, fever up to 38,8C, Nausea, dizziness, weakness. OE: moderate jaundice, temperature is 39C. Tachicardia up to 122 /min. Liver sizes by Kurlov: 11108 sm., edge painless. Spleen enlarged on 3 sm. below left costal arch. Anuria about 10 hours. 2. Chose symptoms, typical for cholelitiasis: d) Gradual onset from nausea, gradual loss of appetite, increasing itch of skin. OE: jaundice of skin, with scratches traces. Skin turgor is reduced. Pulse is 86 /min.; liver sizes by Kurlov: 13128 sm, in gall - bladder projection elastic painless formation 33 is palpated. Spleen is not enlarged. e) Acute beginning from nausea, pain in a right part of abdomen, fever up to 38C. OE: moderate jaundice; tachicardia up to 104 /min. Liver sizes by Kurlov: 12127 sm., palpation of gall bladder is painful. Spleen is not enlarged. f) Beginning gradual from nausea, pain in joints, itch of skin, and loss of appetite. OE: skin is icteric, turgor is normal. Pulse is 68 /min.. Liver sizes by Kurlov: 13128 sm., edge of lever sensitive. Spleen is not enlarged. g) Acute onset with chill, fever up to 38,8C, Nausea, dizziness, weakness. OE: moderate jaundice, temperature is 39C. Tachicardia up to 122 /min. Liver sizes by Kurlov: 11108 sm., edge painless. Spleen enlarged on 3 sm. below left costal arch. Anuria about 10 hours. 3. Chose symptoms, typical for tumor of head of pancreas: a) Gradual onset from nausea, gradual loss of appetite, increasing itch of skin. OE: jaundice of skin, with scratches traces. Skin turgor is reduced. Pulse is 86 /min.; liver sizes by Kurlov: 13128 sm, in gall - bladder

projection elastic painless formation 33 is palpated. Spleen is not enlarged. b) Acute beginning from nausea, pain in a right part of abdomen, fever up to 38C. OE: moderate jaundice; tachicardia up to 104 /min. Liver sizes by Kurlov: 12127 sm., palpation of gall bladder is painful. Spleen is not enlarged. c) Beginning gradual from nausea, pain in joints, itch of skin, and loss of appetite. OE: skin is icteric, turgor is normal. Pulse is 68 /min. Liver sizes by Kurlov: 13128 sm., edge of lever sensitive. Spleen is not enlarged. d) Acute onset with chill, fever up to 38,8C, Nausea, dizziness, weakness. OE: moderate jaundice, temperature is 39C. Tachicardia up to 122 /min. Liver sizes by Kurlov: 11108 sm., edge painless. Spleen enlarged on 3 sm. below left costal arch. Anuria about 10 hours. 4. What pathogenetic mechanisms of diarrhea are typical for cholera: a) Dehydratation, hypovolemia, arterial pressure decreasing, reduction of renal filtration; b) Toxinemia, disturbance of peripheral microcirculation; c) Dehydratation, metabolic acidosis, hypokaliumemia; d) Increase of water and electrolytes secretion as a result of activation of adenilatcyclaza and accumulation of cAMP. 5. What basic clinical symptoms of colitic form of shigellosis: a) Gradual onset, absence of intoxication, moderate spasmatic pains in lower part of abdomen, alternation of constipation and diarrhea, sometimes slime in faeces; b) Acute onset with sharp pain in abdomen, symptoms of intoxication on the beginning are absent; stool is bloody, frequently without faeces; c) Acute onset, moderate intoxication, diffuse pain in abdomen, vomiting from onset, then liquid stool with admixture of slime (and sometimes of blood); d) Acute onset, moderate intoxication, spasmatic pain in left hypogastria, poor stool with slime and blood. 6. What basic clinical symptoms of cancer of rectum: a) Gradual onset, absence of intoxication, moderate spasmatic pains in lower part of abdomen, alternation of constipation and diarrhea, sometimes slime in faeces; b) Acute onset with sharp pain in abdomen, symptoms of intoxication on the beginning are absent; stool is bloody, frequently without faeces; c) Acute onset, moderate intoxication, diffuse pain in abdomen, vomiting from onset, then liquid stool with admixture of slime (and sometimes of blood);

98

d) Acute onset, moderate intoxication, spasmatic pain in left hypogastria, poor stool with slime and blood. 7. What basic clinical symptoms of thrombosis of mesenteric vessels: a) Gradual onset, absence of intoxication, moderate spasmatic pains in lower part of abdomen, alternation of constipation and diarrhea, sometimes slime in faeces; b) Acute onset with sharp pain in abdomen, symptoms of intoxication on the beginning are absent; stool is bloody, frequently without faeces; c) Acute onset, moderate intoxication, diffuse pain in abdomen, vomiting from onset, then liquid stool with admixture of slime (and sometimes of blood); d) Acute onset, moderate intoxication, spasmatic pain in left hypogastria, poor stool with slime and blood. 8. What are basic clinical symptoms of climax period of malaria vivax: e) Constant remitting fever, chills, significant sweating, polylymphadenopathy, muscular pain, spleen and lever enlargement; f) Constant type of fever, weakness, adynamia, pale skin, poor roseolar rush on lateral surfaces of trunk, spleen and lever enlargement; g) Fever of remitting type, chills, significant weakness, petechial bleedings on conjunctiva, sometimes liver enlargement; h) Fever of intermitting type, on climax of fever fit feeling of heat with subsequent plentiful sweating, spleen and lever enlargement. 9. What are basic clinical symptoms of climax period of acute brucellosis: a) Constant remitting fever, chills, significant sweating, polylymphadenopathy, muscular pain, spleen and lever enlargement; b) Constant type of fever, weakness, adynamia, pale skin, poor roseolar rush on lateral surfaces of trunk, spleen and lever enlargement; c) Fever of remitting type, chills, significant weakness, petechial bleedings on conjunctiva, sometimes liver enlargement; d) Fever of intermitting type, on climax of fever fit feeling of heat with subsequent plentiful sweating, spleen and lever enlargement. 10. What are basic clinical symptoms of climax period of staphylococcal sepsis: a) Constant remitting fever, chills, significant sweating, polylymphadenopathy, muscular pain, spleen and lever enlargement; b) Constant type of fever, weakness, adynamia, pale skin, poor roseolar rush on lateral surfaces of trunk, spleen and lever enlargement; c) Fever of remitting type, chills, significant weakness, petechial bleedings on conjunctiva, sometimes liver enlargement;

d) Fever of intermitting type, on climax of fever fit feeling of heat with subsequent plentiful sweating, spleen and lever enlargement. 11. What symptoms are most typical for staphylococcal food poisoning: a) Onset with chill, vomiting, short - term increase of temperature; diffuse abdominal pain, liquid stool with greenish shade is often (15-20 of time per day); b) Acute beginning with chill, repeated vomiting, sharp pains in epigastria, collapse is quite often, liquid stool 4-5 times per day, fast convalescence; c) Acute onset with liquid watery stools, subsequent vomiting connection; oliguria, normal body temperature, absence of abdominal pain; d) Gradual onset with fever, increasing weakness, pale skin, duration of fever about 2 weeks, from the end of first week - liquid stool 2-4 times per day; 12. What symptoms are most typical for gastrointestinal salmonellosis: a) Onset with chill, vomiting, short - term increase of temperature; diffuse abdominal pain, liquid stool with greenish shade is often (15-20 of time per day); b) Acute beginning with chill, repeated vomiting, sharp pains in epigastria, collapse is quite often, liquid stool 4-5 times per day, fast convalescence; c) Acute onset with liquid watery stools, subsequent vomiting connection; oliguria, normal body temperature, absence of abdominal pain; d) Gradual onset with fever, increasing weakness, pale skin, duration of fever about 2 weeks, from the end of first week - liquid stool 2-4 times per day; 13. What symptoms are most typical for moderate form of cholera: a) Onset with chill, vomiting, short - term increase of temperature; diffuse abdominal pain, liquid stool with greenish shade is often (15-20 of time per day); b) Acute beginning with chill, repeated vomiting, sharp pains in epigastria, collapse is quite often, liquid stool 4-5 times per day, fast convalescence; c) Acute onset with liquid watery stools, subsequent vomiting connection; oliguria, normal body temperature, absence of abdominal pain; d) Gradual onset with fever, increasing weakness, pale skin, duration of fever about 2 weeks, from the end of first week - liquid stool 2-4 times per day; 14. What symptoms are typical for initial period of typhoid fever;

99

d) Acute onset with fast increase of temperature within 1-2 days up to 39-40C; a persistent headache, sleeplessness, euphoria. Face is hyperemic, sclera are injected, spleen is increased since 3-4 days from onset; e) Gradual onset with slow (4-5 days) increase of body temperature, general weakness, slackness, pale skin; tongue is thickened, with teeth prints, bradicardia, spleen is enlarged since 6-7 days from onset; f) Acute onset, within several hours body temperature reaches 38-39C, accompanied with chill, headache in forehead and temple area, pain in eyes, nasal obstruction, dry cough, tracheal pain. Spleen is not enlarged; g) Acute onset with chill, fast increase of temperature up to 39-40C; weakness, sweating, muscular pain, excitation, headache, dry cough, hyperemia of face, injection of sclera, sometimes palatal enantema, relative bradicardia, spleen enlargement is quite often. 15. What symptoms are typical for initial period of louse - born typhus; a) Acute onset with fast increase of temperature within 1-2 days up to 39-40C; a persistent headache, sleeplessness, euphoria. Face is hyperemic, sclera are injected, spleen is increased since 3-4 days from onset; b) Gradual onset with slow (4-5 days) increase of body temperature, general weakness, slackness, pale skin; tongue is thickened, with teeth prints, bradicardia, spleen is enlarged since 6-7 days from onset; c) Acute onset, within several hours body temperature reaches 38-39C, accompanied with chill, headache in forehead and temple area, pain in eyes, nasal obstruction, dry cough, tracheal pain. Spleen is not enlarged; d) Acute onset with chill, fast increase of temperature up to 39-40C; weakness, sweating, muscular pain, excitation, headache, dry cough, hyperemia of face, injection of sclera, sometimes palatal enantema, relative bradicardia, spleen enlargement is quite often. 16. What symptoms are typical for initial period of Q fever; a) Acute onset with fast increase of temperature within 1-2 days up to 39-40C; a persistent headache, sleeplessness, euphoria. Face is hyperemic, sclera are injected, spleen is increased since 3-4 days from onset; b) Gradual onset with slow (4-5 days) increase of body temperature, general weakness, slackness, pale skin; tongue is thickened, with teeth prints, bradicardia, spleen is enlarged since 6-7 days from onset; c) Acute onset, within several hours body temperature reaches 38-39C, accompanied with chill, headache in forehead and temple

area, pain in eyes, nasal obstruction, dry cough, tracheal pain. Spleen is not enlarged; d) Acute onset with chill, fast increase of temperature up to 39-40C; weakness, sweating, muscular pain, excitation, headache, dry cough, hyperemia of face, injection of sclera, sometimes palatal enantema, relative bradicardia, spleen enlargement is quite often. 17. Choose symptoms, typical for diphtheria: a) Acute onset, general headache, pain in throat, fever 38 -40C, lymphadenopathy, mucous of throat is hyperemic, white - yellow purulent removed films in lacunas of tonsils; spleen enlargement; b) Acute beginning, strong headache, pain in throat as swallowing, fever 39-40C, mucous of throat is hyperemic, grey - yellow purulent removed films in lacunas of tonsils, jugular lymph nods are increased and sharply painful; c) Acute beginning, moderate pain in a throat, fever up to 38C, mucous of throat is hyperemic, white colored, hardly removed films, slightly increased and moderately painful jugularis lymph nodes; d) Acute onset, fever up to 38 - 40C, mucous of throat is moderate hyperemic, on left tonsil crater - like ulcer with purulent content, lymph nodes are not enlarged. 18. Choose symptoms, typical for infectious mononucleosis: a) Acute onset, general headache, pain in throat, fever 38 -40C, lymphadenopathy, mucous of throat is hyperemic, white - yellow purulent removed films in lacunas of tonsils; spleen enlargement; b) Acute beginning, strong headache, pain in throat as swallowing, fever 39-40C, mucous of throat is hyperemic, grey - yellow purulent removed films in lacunas of tonsils, jugular lymph nods are increased and sharply painful; c) Acute beginning, moderate pain in a throat, fever up to 38C, mucous of throat is hyperemic, white colored, hardly removed films, slightly increased and moderately painful jugularis lymph nodes; d) Acute onset, fever up to 38 - 40C, mucous of throat is moderate hyperemic, on left tonsil crater - like ulcer with purulent content, lymph nodes are not enlarged. 19. Choose symptoms, typical for acute tonsillitis: a) Acute onset, general headache, pain in throat, fever 38 -40C, lymphadenopathy, mucous of throat is hyperemic, white - yellow purulent removed films in lacunas of tonsils; spleen enlargement; b) Acute beginning, strong headache, pain in throat as swallowing, fever 39-40C, mucous of throat is hyperemic, grey - yellow purulent

100

removed films in lacunas of tonsils, jugular lymph nods are increased and sharply painful; c) Acute beginning, moderate pain in a throat, fever up to 38C, mucous of throat is hyperemic, white colored, hardly removed films, slightly increased and moderately painful jugularis lymph nodes; d) Acute onset, fever up to 38 - 40C, mucous of throat is moderate hyperemic, on left tonsil crater - like ulcer with purulent content, lymph nodes are not enlarged. 20. What symptoms are typical for initial period of typhoid fever: a) Acute beginning with fast increase of body temperature within 1-2 days up to 39-40C, persistent headache, sleeplessness, anxiety, euphoria. Face is hyperemic, sclera are injected, spleen is enlarged since 3-4 day of disease; b) Gradual onset with slow (4-5 days) increase of temperature, general adinamia, slackness, pale skin, tongue is tickened, with prints of teeth, relative bradicardia, spleen is enlarged since 6-7 day of disease; c) Acute beginning, within several hours temperature reaches 38-39C, accompanied with chill, headache in forehead and temple area, pain in eyes, stiffness of a nose, dry cough, morbidity on trachea projection. Spleen is not enlarged. d) Acute beginning with chill, fast increase of temperature up to 39-40C, weakness, sweating, muscular pain, excitation, headache, dry cough, hyperemia of face, injection of sclera, sometimes palatal enantema, relative bradicardia, spleen is enlarged quite often. 21. What symptoms are typical for initial period of louse - borne typhus: a) Acute beginning with fast increase of body temperature within 1-2 days up to 39-40C, persistent headache, sleeplessness, anxiety, euphoria. Face is hyperemic, sclera are injected, spleen is enlarged since 3-4 day of disease; b) Gradual onset with slow (4-5 days) increase of temperature, general adinamia, slackness, pale skin, tongue is tickened, with prints of teeth, relative bradicardia, spleen is enlarged since 6-7 day of disease; c) Acute beginning, within several hours temperature reaches 38-39C, accompanied with chill, headache in forehead and temple area, pain in eyes, stiffness of a nose, dry cough, morbidity on trachea projection. Spleen is not enlarged. d) Acute beginning with chill, fast increase of temperature up to 39-40C, weakness, sweating, muscular pain, excitation, headache, dry cough, hyperemia of face, injection of sclera, sometimes palatal enantema, relative bradicardia, spleen is enlarged quite often. 22. What symptoms are typical for initial period of Q fever:

a) Acute beginning with fast increase of body temperature within 1-2 days up to 39-40C, persistent headache, sleeplessness, anxiety, euphoria. Face is hyperemic, sclera are injected, spleen is enlarged since 3-4 day of disease; b) Gradual onset with slow (4-5 days) increase of temperature, general adinamia, slackness, pale skin, tongue is tickened, with prints of teeth, relative bradicardia, spleen is enlarged since 6-7 day of disease; c) Acute beginning, within several hours temperature reaches 38-39C, accompanied with chill, headache in forehead and temple area, pain in eyes, stiffness of a nose, dry cough, morbidity on trachea projection. Spleen is not enlarged. d) Acute beginning with chill, fast increase of temperature up to 39-40C, weakness, sweating, muscular pain, excitation, headache, dry cough, hyperemia of face, injection of sclera, sometimes palatal enantema, relative bradicardia, spleen is enlarged quite often. 23. What symptoms are typical for gastroenterocolitic form of shigellosis: a) Acute onset, increase of temperature up to 39C, repeated vomiting, diffuse abdominal pain, plentiful watery stool without slime and blood, spasm of led muscles, oliguria, hypotonia; b) Acute onset, increase of temperature up to 38,539C, vomiting 3-4 times, spasmatic abdominal pain in left hypogastrium, often plentiful liquid stool with admixture of blood and slime; c) Acute onset, normal body temperature, vomiting, often plentiful watery stool without slime and blood, spasm of leg muscles, expressed dryness mucous, decrease of skin elasticity, oliguria; d) Acute onset, increase of temperature up to 39C, spasmatic pain in the left half of abdomen, often liquid poor stool with slime and blood, tenesms. 24. What symptoms are typical for colitic form of shigellosis: a) Acute onset, increase of temperature up to 39C, repeated vomiting, diffuse abdominal pain, plentiful watery stool without slime and blood, spasm of led muscles, oliguria, hypotonia; b) Acute onset, increase of temperature up to 38,539C, vomiting 3-4 times, spasmatic abdominal pain in left hypogastrium, often plentiful liquid stool with admixture of blood and slime; c) Acute onset, normal body temperature, vomiting, often plentiful watery stool without slime and blood, spasm of leg muscles, expressed dryness mucous, decrease of skin elasticity, oliguria; d) Acute onset, increase of temperature up to 39C, spasmatic pain in the left half of abdomen, often liquid poor stool with slime and blood, tenesms. 25. What symptoms are typical for gastroenteritic form of salmonellosis: a) Acute onset, increase of temperature up to 39C, repeated vomiting, diffuse abdominal pain,

101

plentiful watery stool without slime and blood, spasm of led muscles, oliguria, hypotonia; b) Acute onset, increase of temperature up to 38,539C, vomiting 3-4 times, spasmatic abdominal pain in left hypogastrium, often plentiful liquid stool with admixture of blood and slime; c) Acute onset, normal body temperature, vomiting, often plentiful watery stool without slime and blood, spasm of leg muscles, expressed dryness mucous, decrease of skin elasticity, oliguria; d) Acute onset, increase of temperature up to 39C, spasmatic pain in the left half of abdomen, often liquid poor stool with slime and blood, tenesms. 26. What hemodynamics parameters of hemorrhagic fever with renal syndrome: e) Tachicardia, hypotonia, tendency to collapse; f) Relative bradicardia, dycrotia of pulse, hypotonia; g) Significant bradicardia, hypotonia; h) Tachicardia up to 120-160 /min., arrhythmia, hypotonia. 27. What hemodynamics parameters of typhoid fever: e) Tachicardia, hypotonia, tendency to collapse; f) Relative bradicardia, dycrotia of pulse, hypotonia; g) Significant bradicardia, hypotonia; h) Tachicardia up to 120-160 /min., arrhythmia, hypotonia. 28. What hemodynamics parameters of louse - borne typhus: a) Tachicardia, hypotonia, tendency to collapse; b) Relative bradicardia, dycrotia of pulse, hypotonia; c) Significant bradicardia, hypotonia; d) Tachicardia up to 120-160 /min., arrhythmia, hypotonia. 29. What are main symptoms of hepatic coma as complication of viral hepatitis B: e) Patient is unconsciousness. Temperature 39,8C. Skin pale, damp, jaundice. Tachicardia up to 148 /min.. Liver sizes by Kurlov: 12119 sm., spleen enlarged on 4 sm. below edge of costal arch; f) Patient is unconsciousness. Temperature 37,2C. Bright jaundice, individual petechia. Tachicardia up to 98 /min.. Liver sizes by Kurlov: 8 87 sm.. Spleen is not enlarged; g) Patient is unconsciousness. Temperature 36,1C. Skin pale, dry. Tachicardia up to 110 /min.. Liver sizes by Kurlov: 1098 sm. Spleen is not enlarged; h) Patient is unconsciousness. Temperature 38,5C. Skin of light pink color, plentiful hemorrhagic rash is presented. Tachicardia up to 130 /min.. Liver and spleen are not enlarged. 30. What are main symptoms of hyperglycemic coma as complication of sugar diabetes: a) Patient is unconsciousness. Temperature 39,8C. Skin pale, damp, jaundice. Tachicardia up to 148 /min.. Liver sizes by Kurlov: 12119 sm., spleen enlarged on 4 sm. below edge of costal arch; b) Patient is unconsciousness. Temperature 37,2C. Bright jaundice, individual petechia. Tachicardia up to 98 /min.. Liver sizes by Kurlov: 8 87 sm.. Spleen is not enlarged;

c) Patient is unconsciousness. Temperature 36,1C. Skin pale, dry. Tachicardia up to 110 /min.. Liver sizes by Kurlov: 1098 sm. Spleen is not enlarged; d) Patient is unconsciousness. Temperature 38,5C. Skin of light pink color, plentiful hemorrhagic rash is presented. Tachicardia up to 130 /min.. Liver and spleen are not enlarged. 31. What are main symptoms of malarial coma as complication of tropical malaria: a) Patient is unconsciousness. Temperature 39,8C. Skin pale, damp, jaundice. Tachicardia up to 148 /min.. Liver sizes by Kurlov: 12119 sm., spleen enlarged on 4 sm. below edge of costal arch; b) Patient is unconsciousness. Temperature 37,2C. Bright jaundice, individual petechia. Tachicardia up to 98 /min.. Liver sizes by Kurlov: 8 87 sm.. Spleen is not enlarged; c) Patient is unconsciousness. Temperature 36,1C. Skin pale, dry. Tachicardia up to 110 /min.. Liver sizes by Kurlov: 1098 sm. Spleen is not enlarged; d) Patient is unconsciousness. Temperature 38,5C. Skin of light pink color, plentiful hemorrhagic rash is presented. Tachicardia up to 130 /min.. Liver and spleen are not enlarged. 32. What are typical symptoms of initial period of pseudo-tuberculosis: e) Acute onset, in 2-6 hours body temperature reaches 38-39C, expressed headache in forehead and eyes, sore throat, dry cough. Lymph nodes and spleen are not enlarged. Urine passage is normal; f) Acute onset with chill, in 12-24 hours temperature reaches 40C, strong headache, are expressed muscular pain. Skin of face and neck hyperemic, scleritis, petechial rash, liver and spleen enlarged. Oliguria. g) Acute onset, on 2-3 day of disease temperature reaches 40C, headache, loss of appetite, pain in a throat, liquid stool 3-5 times per day. Skin of face, neck, palms and feet are hyperemic, pale nasal triangle. liver and spleen enlarged. Urine passage is normal; h) Acute onset, on 2-3 day of disease body temperature reaches 39,5-40C, strong headache, anxiety, quite often excitation, euphoria; hyperemia of face, sclera injected, spleen enlargement. Urine passage is normal. 33. What are typical symptoms of initial period of influenza: a) Acute onset, in 2-6 hours body temperature reaches 38-39C, expressed headache in forehead and eyes, sore throat, dry cough. Lymph nodes and spleen are not enlarged. Urine passage is normal; b) Acute onset with chill, in 12-24 hours temperature reaches 40C, strong headache, are expressed muscular pain. Skin of face and neck hyperemic, scleritis, petechial rash, liver and spleen enlarged. Oliguria. c) Acute onset, on 2-3 day of disease temperature reaches 40C, headache, loss of appetite, pain in a

102

throat, liquid stool 3-5 times per day. Skin of face, neck, palms and feet are hyperemic, pale nasal triangle. liver and spleen enlarged. Urine passage is normal; d) Acute onset, on 2-3 day of disease body temperature reaches 39,5-40C, strong headache, anxiety, quite often excitation, euphoria; hyperemia of face, sclera injected, spleen enlargement. Urine passage is normal. 34. What are typical symptoms of initial period of leptospyrosis: a) Acute onset, in 2-6 hours body temperature reaches 38-39C, expressed headache in forehead and eyes, sore throat, dry cough. Lymph nodes and spleen are not enlarged. Urine passage is normal; b) Acute onset with chill, in 12-24 hours temperature reaches 40C, strong headache, are expressed muscular pain. Skin of face and neck hyperemic, scleritis, petechial rash, liver and spleen enlarged. Oliguria. c) Acute onset, on 2-3 day of disease temperature reaches 40C, headache, loss of appetite, pain in a throat, liquid stool 3-5 times per day. Skin of face, neck, palms and feet are hyperemic, pale nasal triangle. liver and spleen enlarged. Urine passage is normal; d) Acute onset, on 2-3 day of disease body temperature reaches 39,5-40C, strong headache, anxiety, quite often excitation, euphoria; hyperemia of face, sclera injected, spleen enlargement. Urine passage is normal. 35. What symptoms are typical for meningococcal nasopharingitis: e) Granularity and hyperemia of a back wall of throat; f) hemprrhagic rash; g) stiffness of neck muscles; h) Plentiful purulent nasal excretion. 36. What symptoms are typical for meningococcal meningitis: e) Granularity and hyperemia of a back wall of throat; f) hemprrhagic rash; g) stiffness of neck muscles; h) Plentiful purulent nasal excretion. 37.What symptoms are typical for meningococcemia: a) Granularity and hyperemia of a back wall of throat; b) hemprrhagic rash; c) stiffness of neck muscles; d) Plentiful purulent nasal excretion. 38. What symptoms of CNS affection are typical for louse - borne typhus: a) hyperestesya, nausea and vomiting, quite often loss of consciousness, headache, positive symptom of Kernig, stiffness of neck muscles; b) Euphoria, excitation, anxiety, acoustical and visual hallucinations, persistent headache, symptoms of cranial nerves affection; c) adinamia, dormancy, dull headache; d) dizziness, noise in ears, adinamia, headache in forehead and temple areas.

39. What symptoms of CNS affection are typical for typhoid fever: a) hyperestesya, nausea and vomiting, quite often loss of consciousness, headache, positive symptom of Kernig, stiffness of neck muscles; b) Euphoria, excitation, anxiety, acoustical and visual hallucinations, persistent headache, symptoms of cranial nerves affection; c) adinamia, dormancy, dull headache; d) dizziness, noise in ears, adinamia, headache in forehead and temple areas. 40. What symptoms of CNS affection are typical for purulent meningitis: a) hyperestesya, nausea and vomiting, quite often loss of consciousness, headache, positive symptom of Kernig, stiffness of neck muscles; b) Euphoria, excitation, anxiety, acoustical and visual hallucinations, persistent headache, symptoms of cranial nerves affection; c) adinamia, dormancy, dull headache; d) dizziness, noise in ears, adinamia, headache in forehead and temple areas. 41. What rush type and localization is typical for pseudo-tuberculosis: a) Arises on 2-5 day of disease on top of trunk, then spreads on extremities and around joints, to a face, feet, roseolar and papular; b) Plentiful, roseolar and petechial, arises on 4-5 day, mainly on thorax, extremities, not typical on face, palms and feet; c) Arises on 2-6 day, little - pointed, on intact skin, located mainly on lateral surfaces of a trunk, axilar areas and abdomen, extremities, is concentrated in skin plaites, face is usually clear; frequently erythema of palms and feet as "gloves" and "socks"; d) Arises on 3-4 day on top of face, then on trunk, forearms and thighs, then on shins, spotty papulous, leaves pigmentation and peeling after recovery; 42. What rush type and localization is typical for louse borne typhus: a) Arises on 2-5 day of disease on top of trunk, then spreads on extremities and around joints, to a face, feet, roseolar and papular; b) Plentiful, roseolar and petechial, arises on 4-5 day, mainly on thorax, extremities, not typical on face, palms and feet; c) Arises on 2-6 day, little - pointed, on intact skin, located mainly on lateral surfaces of a trunk, axilar areas and abdomen, extremities, is concentrated in skin plaites, face is usually clear; frequently erythema of palms and feet as "gloves" and "socks"; d) Arises on 3-4 day on top of face, then on trunk, forearms and thighs, then on shins, spotty papules, leaves pigmentation and peeling after recovery; 43. What rush type and localization is typical for tick borne North Asian rickettsiosis:

103

a) Arises on 2-5 day of disease on top of trunk, then spreads on extremities and around joints, to a face, feet, roseolar and papular; b) Plentiful, roseolar and petechial, arises on 4-5 day, mainly on thorax, extremities, not typical on face, palms and feet; c) Arises on 2-6 day, little - pointed, on intact skin, located mainly on lateral surfaces of a trunk, axial areas and abdomen, extremities, is concentrated in skin plaits, face is usually clear; frequently erythema of palms and feet as "gloves" and "socks"; d) Arises on 3-4 day on top of face, then on trunk, forearms and thighs, then on shins, spotty papules, leaves pigmentation and peeling after recovery; 44. What mechanism of rash formation for meningococcemia: a) Damage of capillaries endothelium by microorganisms and toxins, circulating in blood; b) sedimentation of immune complexes in capillaries of skin; c) Drift of pathogen in lymph vessels of skin with subsequent development of productive inflammatory changes; d) generalized destructive and prolypherative vasculitis; 45. What mechanism of rash formation for typhoid fever: a) Damage of capillaries endothelium by microorganisms and toxins, circulating in blood; b) sedimentation of immune complexes in capillaries of skin; c) Drift of pathogen in lymph vessels of skin with subsequent development of productive inflammatory changes; d) generalized destructive and prolypherative vasculitis; 46. What mechanism of rash formation for louse - borne typhus. a) Damage of capillaries endothelium by microorganisms and toxins, circulating in blood; b) sedimentation of immune complexes in capillaries of skin; c) Drift of pathogen in lymph vessels of skin with subsequent development of productive inflammatory changes; d) generalized destructive and prolypherative vasculitis; 47. What abdominal symptoms are typical for rotaviral infection; a) Spleen is enlarged since 3-4 day of disease, soft, liver is enlarged; tendency to constipation, ishuria, oliguria; b) Liver and spleen are enlarged since 6-7 day, firm; stool liquid 3-4 times per day, meteorism; c) Liver and spleen are not enlarged, abdomen is soft, slightly painful around navel, stool is liquid from 2 up to 10 times per day;

d) liver enlarged since 3-4 day of disease, sensitive on palpation, spleen is enlarged, meteorism, mesenteric lymph nodes are palpated, stool 1-2 times a day. 48. What abdominal symptoms are typical for louse borne typhus; a) Spleen is enlarged since 3-4 day of disease, soft, liver is enlarged; tendency to constipation, ishuria, oliguria; b) Liver and spleen are enlarged since 6-7 day, firm; stool liquid 3-4 times per day, meteorism; c) Liver and spleen are not enlarged, abdomen is soft, slightly painful around navel, stool is liquid from 2 up to 10 times per day; d) liver enlarged since 3-4 day of disease, sensitive on palpation, spleen is enlarged, meteorism, mesenteric lymph nodes are palpated, stool 1-2 times a day. 49.What abdominal symptoms are typical for typhoid fever; a) Spleen is enlarged since 3-4 day of disease, soft, liver is enlarged; tendency to constipation, ishuria, oliguria; b) Liver and spleen are enlarged since 6-7 day, firm; stool liquid 3-4 times per day, meteorism; c) Liver and spleen are not enlarged, abdomen is soft, slightly painful around navel, stool is liquid from 2 up to 10 times per day; d) liver enlarged since 3-4 day of disease, sensitive on palpation, spleen is enlarged, meteorism, mesenteric lymph nodes are palpated, stool 1-2 times a day. 50. What symptoms are typical for initial period of leptostyrosis: a) Acute onset with increase of body temperature to 38C, loss of appetite, nausea, weakness and general unwell, sometimes dry cough. Since 2-4 day liver is enlarged, temperature of body is normalized; b) Acute onset with increase of body temperature to 39,5C, muscular and joint pains; a headache in forehead area, pain in eyes, dry cough, tracheal pain. Liver is not enlarged. c) Acute onset with increase of body temperature to 39C, headache, mialgias (mainly in leg muscles), weakness, pain in eyes. Injection of sclera, face is hyperemic, petechia in axillar areas. The liver is enlarged, dark urine, oliguria. d) Acute onset with increase of body temperature to 38C, weakness, running nose, dry cough, pain in eyes. Conjunctiva are hyperemic. Neck, axillary lymph nodes and lever are enlarged. Urine is normal. 51. What symptoms are typical for initial period of viral hepatitis A: a) Acute onset with increase of body temperature to 38C, loss of appetite, nausea, weakness and general unwell, sometimes dry cough. Since 2-4 day liver is enlarged, temperature of body is normalized;

104

b) Acute onset with increase of body temperature to 39,5C, muscular and joint pains; a headache in forehead area, pain in eyes, dry cough, tracheal pain. Liver is not enlarged. c) Acute onset with increase of body temperature to 39C, headache, mialgias (mainly in leg muscles), weakness, pain in eyes. Injection of sclera, face is hyperemic, petechia in axillar areas. The liver is enlarged, dark urine, oliguria. d) Acute onset with increase of body temperature to 38C, weakness, running nose, dry cough, pain in eyes. Conjunctiva are hyperemic. Neck, axillary lymph nodes and lever are enlarged. Urine is normal. 52. What symptoms are typical for initial period of adenoviral infection: a) Acute onset with increase of body temperature to 38C, loss of appetite, nausea, weakness and general unwell, sometimes dry cough. Since 2-4 day liver is enlarged, temperature of body is normalized; b) Acute onset with increase of body temperature to 39,5C, muscular and joint pains; a headache in forehead area, pain in eyes, dry cough, tracheal pain. Liver is not enlarged. c) Acute onset with increase of body temperature to 39C, headache, mialgias (mainly in leg muscles), weakness, pain in eyes. Injection of sclera, face is hyperemic, petechia in axillar areas. The liver is enlarged, dark urine, oliguria. d) Acute onset with increase of body temperature to 38C, weakness, running nose, dry cough, pain in eyes. Conjunctiva are hyperemic. Neck, axillary lymph nodes and lever are enlarged. Urine is normal. 53. What drug for specific therapy of shigellosis: a) erythromycin; b) cyprofloxacin; c) penicillin; d) metronidazol (trychopol); e) chlorochin; f) acyclovir; 54. What drug for specific therapy of intestinal amebiasis: a) erythromycin; b) cyprofloxacin; c) penicillin; d) metronidazol (trychopol); e) chlorochin; f) acyclovir; 55. What drug for specific therapy of malaria: a) erythromycin; b) cyprofloxacin; c) penicillin; d) metronidazol (trychopol); e) chlorochin; f) acyclovir; 56. What symptoms are typical for initial period of pseudo-tuberculosis:

a) Acute onset with fast increase of temperature of a body up to 39-40, strong headache, euphoria, sleeplessness; injection of sclera, insignificant hyperemia of the palatum and back wall of throat, lymph nodes are not enlarged, spleen is enlarged since 3-4 day of disease; b) Acute onset with increase of temperature to 38,540C, edema and pain in ankle and knee joints, pain in a throat, increase regional lymph nodes, bright hyperemia of the palatine with precise border, fur coated tongue. Spleen and lever enlargement. c) Gradual onset, from 3d day increase of temperature up to 38C, running nose, pain in a throat. Tonsils are enlarged, hyperemic, without fur. Neck, axillary lymph nodes are increased; conjunctivitis, joints are not changed, sometimes spleen enlargement; dry cough is quite often; d) Acute onset, after some hours temperature reaches 40C, headache in frontal area, pain in eyes, tracheal pain. Palatinal hyperemia. Lymph nodes and spleen are not enlarged, joints are normal. 57. What symptoms are typical for initial period of louse - borne typhus: a) Acute onset with fast increase of temperature of a body up to 39-40, strong headache, euphoria, sleeplessness; injection of sclera, insignificant hyperemia of the palatum and back wall of throat, lymph nodes are not enlarged, spleen is enlarged since 3-4 day of disease; b) Acute onset with increase of temperature to 38,540C, edema and pain in ankle and knee joints, pain in a throat, increase regional lymph nodes, bright hyperemia of the palatine with precise border, fur coated tongue. Spleen and lever enlargement. c) Gradual onset, from 3d day increase of temperature up to 38C, running nose, pain in a throat. Tonsils are enlarged, hyperemic, without fur. Neck, axillary lymph nodes are increased; conjunctivitis, joints are not changed, sometimes spleen enlargement; dry cough is quite often; d) Acute onset, after some hours temperature reaches 40C, headache in frontal area, pain in eyes, tracheal pain. Palatinal hyperemia. Lymph nodes and spleen are not enlarged, joints are normal. 58. What symptoms are typical for initial period of adenoviral infection: a) Acute onset with fast increase of temperature of a body up to 39-40, strong headache, euphoria, sleeplessness; injection of sclera, insignificant hyperemia of the palatum and back wall of throat, lymph nodes are not enlarged, spleen is enlarged since 3-4 day of disease; b) Acute onset with increase of temperature to 38,5-40C, edema and pain in ankle and knee

105

joints, pain in a throat, increase regional lymph nodes, bright hyperemia of the palatine with precise border, fur coated tongue. Spleen and lever enlargement. c) Gradual onset, from 3d day increase of temperature up to 38C, running nose, pain in a throat. Tonsils are enlarged, hyperemic, without fur. Neck, axillary lymph nodes are increased; conjunctivitis, joints are not changed, sometimes spleen enlargement; dry cough is quite often; d) Acute onset, after some hours temperature reaches 40C, headache in frontal area, pain in eyes, tracheal pain. Palatinal hyperemia. Lymph nodes and spleen are not enlarged, joints are normal.

106

SKIN CHANGES IN SOME INFECTIOUS DISEASES 59. What skin phenomena are typical for skin form of plague: e) Affected places of skin are sharply outlined, jelly - like fluctuation of skin during shaking. On center of edema painless black ulcer 34 sm. with yellow purulent edge. In outer zone of hyperemia - vesicula, filled by dark red contents. f) Skin is hyperemic with cyanotic shade, edema is moderate. On centre of edema black ulcer, 11 sm. without purulent content. Edema and ulcer are sharply painful. g) Moderate edema of skin, in centre of edema zone is crater - like ulcer under grey crusta, pus is excreted during pressing. Highly expressed infiltration, lymphadenit and lymphangit are presented. Touching to edges of ulcer is painful. h) Skin is markedly hyperemic, shining, form is smoothed, a zone of affection with precise festooned edges and a platen on end. On center affection hyperemia is less expressed, than on edges. 60. What skin phenomena are typical for staphylococcal infection, carbuncle: a) Affected places of skin are sharply outlined, jelly - like fluctuation of skin during shaking. On center of edema painless black ulcer 34 sm. with yellow purulent edge. In outer zone of hyperemia - vesicula, filled by dark red contents. b) Skin is hyperemic with cyanotic shade, edema is moderate. On centre of edema black ulcer, 11 sm. without purulent content. Edema and ulcer are sharply painful. c) Moderate edema of skin, in centre of edema zone is crater - like ulcer under grey crusta, pus is excreted during pressing. Highly expressed infiltration, lymphadenit and lymphangit are presented. Touching to edges of ulcer is painful. d) Skin is markedly hyperemic, shining, form is smoothed, a zone of affection with precise festooned edges and a platen on end. On center affection hyperemia is less expressed, than on edges. 61. What skin phenomena are typical for skin form of anthrax: a) Affected places of skin are sharply outlined, jelly - like fluctuation of skin during shaking. On center of edema painless black ulcer 34 sm. with yellow purulent edge. In outer zone of hyperemia - vesicula, filled by dark red contents. b) Skin is hyperemic with cyanotic shade, edema is moderate. On centre of edema black ulcer, 11 sm. without purulent content. Edema and ulcer are sharply painful.

Moderate edema of skin, in centre of edema zone is crater - like ulcer under grey crusta, pus is excreted during pressing. Highly expressed infiltration, lymphadenit and lymphangit are presented. Touching to edges of ulcer is painful. d) Skin is markedly hyperemic, shining, form is smoothed, a zone of affection with precise festooned edges and a platen on end. On center affection hyperemia is less expressed, than on edges. 62. What character and localization of rush is typical for pseudo-tuberculosis: g) Occurs on 4-5 days of disease on lateral surfaces of trunk, lateral surfaces of shoulders and forearm (intact face, palms, plants), rash in roseolar and petechial; h) Rash is plentiful, roseola and papula, sometimes eritematous (face, neck), arises on 2-3 days of disease, condensation in skin of joints, palms, feet is observed, on palms and feet quite often are eritema such "gloves" and "socks"; i) Single elements of roseolar rash arise on 8-10 day of disease, are located on forward surface of abdomen and thorax, some elements are roseola and papula. 63. What character and localization of rush is typical for typhoid fever: a) Occurs on 4-5 days of disease on lateral surfaces of trunk, lateral surfaces of shoulders and forearm (intact face, palms, plants), rash in roseolar and petechial; b) Rash is plentiful, roseola and papula, sometimes eritematous (face, neck), arises on 2-3 days of disease, condensation in skin of joints, palms, feet is observed, on palms and feet quite often are eritema such "gloves" and "socks"; c) Single elements of roseolar rash arise on 8-10 day of disease, are located on forward surface of abdomen and thorax, some elements are roseola and papula. 64. What character and localization of rush is typical for louse borne typhus: f) Occurs on 4-5 days of disease on lateral surfaces of trunk, lateral surfaces of shoulders and forearm (intact face, palms, plants), rash in roseolar and petechial; g) Rash is plentiful, roseola and papula, sometimes eritematous (face, neck), arises on 2-3 days of disease, condensation in skin of joints, palms, feet is observed, on palms and feet quite often are eritema such "gloves" and "socks"; h) Single elements of roseolar rash arise on 8-10 day of disease, are located on forward surface of abdomen and thorax, some elements are roseola and papula.

c)

107

LABORATORY METHOD OF DIAGNOSTICS 65. What methods are used for diagnostics of HIV/AIDS: a) Blood smear and tick drop microscopy; b) Clinical blood analysis; c) *Immune enzyme assay; d) *Immune blotting; e) Latex agglutination. 66. What tests are used for cytomegalovirus infection diagnostics: a) Reaction of compliment fixation; b) Absolute neutrophil count of Romanovsky Gimza stained blood smear; c) Immune enzyme detection of specific IgM and IgG in serum of blood; d) Cytological assay of urine sediment; e) Immunological assay (estimation of absolute and relative amount of lymphocytes subpopulation, immunoglobulins of M,A in serum of blood). 67. What methods are used for detection of replication activity of opportunistic infections pathogens and HIV on different stages of infectious process: a) Reaction of indirect agglutination; b) *PCR; c) Vidal reaction; d) Immunofluorescent microscopy with monoclonal antibodies; e) Avidity of antibodies detection. 68. What defect of immunoenzyme method in HIV/AIDS diagnostics: a) Low sensitivity; b) Low specificity; c) High cost; d) *Hypersensitivity; e) Long time performance; 69. How to treat positive result IEA in case of absence of clinical data of HIV/AIDS on examination: a) patient has protective antibodies level to HIV; b) *patient may be in incubation period, acute stage was asymtomatis or wasnt diagnosed; c) patient in incubation period; d) patient in incubation period; e) This is acute asymptomatic stage of infection. 70. What tactic of physician in case of negative result of IEA in patient with AIDS on stage of persistent generalized lymphadenopathy: a) Clinical analysis of blood performance; b) *Repeat assay with IEA test system of different firm; c) Exclude diagnosis of HIV infection; d) Recommend assay of enzyme activity indicating cytolysis in serum; e) Detect immunoglobulin E concentration in serum of blood. 71. What tactic of physician in case of negative result of IEA in patient with AIDS, asymptomatic on examination: a) Perform clinical analysis of blood; b) Prove HIV infection diagnosis;

c) *Repeat serum IEA, and in case of positive result to perform immune blotting; d) Recommend repeated IEA assay in one ear; e) Prescribe antiretroviral therapy. 72. Which methods from mentioned below are informative for sytomegalovirus infection: a) Blood smear and tick drop microscopy; b) Clinical blood sample; c) *Immunoenzyme assay; d) Immune blotting; e) Latex agglutination. 73. What laboratory methods should be applied for proving of etiology of herpesviral infections, phase of infectious process and respectively tactics of treatment: a) Immune blotting; b) Estimation of replicative capability of virus; c) Clinical blood analysis; d) Immunoenzyme assay; e) Reaction of Right; f) *One index is not enough, should be applied complex, including assay of antibodies Ig M, G, their dynamic titer, and (if necessary) in CSF, PCR. 74. Method of specific diagnostics of brucellosis: a) bacteriological; b) bacterioscopy; c) allergological; d) *serological; e) biological. 75. What material for specific diagnostics of pseudotuberculosis: a) blood; b) faeces; c) sputum; d) urine; e) all mentioned. 76 What are main data of CSF in meningococcal meningitis: a) CSF pressure elevation; b) High neutrophil pleositosis; c) Protein cell dissociation; d) Normal glucose amount; e) CSF is dimmed. 77. What laboratory methods are necessary for proving of latent or chronic toxoplasmosis: a) X ray examination of brain and muscles; b) Parasytological examination of faeces; c) Parasytological examination of blood; d) ECG; e) Aye bottom examination; 78. What laboratory data are proving diagnosis of viral hepatitis A: a) HBs AG in blood serum; b) High activeness ALT in blood serum; c) High level of cholesterol in blood serum; d) Anti HAV IgM in blood serum; e) Anti HCV IgM in blood serum. 79. Methods of specific laboratory diagnostics of food poisonings: a) bacteriological of vomiting masses; b) bacteriological of faeces;

108

c) bacteriological of blood; d) serological of blood serum. 80. What changes in blood count are typical for shigellosis: a) leukopenia; b) moderate neutrophil leukocytosis; c) relative lymph and monocytosis; d) drum stick shift to left. 81. What methods for specific diagnostics of ARVI: a) rinocytoscopia; b) virusological; c) serological; d) immunofluorescent. 82. What methods for specific diagnostics of toxoplasmosis: a) Reaction of compliment fixation; b) Skin allergic test; c) Parasytoscopy of faeces; d) RSF (Sabin Feldman reaction); e) RIF (reaction of immune fluorescence). 83. What laboratory data are proving diagnosis of viral hepatitis B: a) Anti HAV IgM in blood serum; b) HBs AG in blood serum; c) High activeness ALT in blood serum; d) High level of alkaline phosfatace in blood serum; e) Anti HBc IgM in blood serum. 84. What methods of specific laboratory diagnostics of shigellosis: a) bacteriological of faeces; b) bacteriological of blood; c) reaction of indirect hemmaglutination (RIH); d) reaction of immune fluorescence; e) immunoenzyme assay. 85. What methods of specific laboratory diagnostics of cholera: a) bacteriological; b) bacteriological of blood; c) serological of blood; d) bacteriological of faeces and food. 86. What methods of laboratory diagnostics of botulism: a) biological test on mice; b) bacteriological of blood; c) serological of blood; d) bacteriological of faeces and food. 87. What serological tests are typical for brucellosis: a) RCF; b) reaction of agglutination of Right, Heddelson; c) RIHA. d) RIF; e) Reaction of latex agglutination. 88. What methods for pseudotuberculosis diagnostics: a) epidemiological; b) clinical; c) bacteriological; d) serological. 89. Material for early bacteriological diagnostics of typhoid and paratyphoid fever is: a) faeces;

b) blood; c) CSF; d) urine; e) all mentioned. 90. In what case can be false negative result of immunoenzyme assay of HIV infection: a) malaria; b) patients, treated with corticosteroids; c) *in pregnant; d) in homosexual persons; e) in drug abused. 91. What peculiarities of blood cell count are typical for climax period of typhoid fever: a) Leucopenia, neutropenia, eosynopenia, relative lymphocytosis; b) Normal count or insignificant leukocytosis, tendency to neutrophil increase, drum - stick shift to the left, monocytosis, plasmatic Turk cells; c) Leucopenia, aneosynophilia, lymphocytosis, thrombocytopenia, ESR elevation; d) Leucocytosis, neutrophenia, lympho - monocytosis, atypical mononuclears. 92. What changes in BCC are typical for influenza: a) Leucocytosis, neutrophenia, lymphocytosis, monocytosis; b) Leucopenia, neutropenia, lymphocytosis; c) Leucocytosis, neutropenia, monocytopenia, lymphocytosis. d) Leucocytosis, neutrophilosis, drum - stick shift to the left. 93. What changes in BCC are typical for infectious mononucleosis: a) Leucocytosis, neutrophenia, lymphocytosis, monocytosis; b) Leucopenia, neutropenia, lymphocytosis; c) Leucocytosis, neutropenia, monocytopenia, lymphocytosis. d) Leucocytosis, neutrophilosis, drum - stick shift to the left. 94. What changes in BCC are typical for acute tonsillitis: a) Leucocytosis, neutrophenia, lymphocytosis, monocytosis; b) Leucopenia, neutropenia, lymphocytosis; c) Leucocytosis, neutropenia, monocytopenia, lymphocytosis. d) Leucocytosis, neutrophilosis, drum - stick shift to the left. 95. What methods of specific diagnostics of rotaviral gastroenteritis: a) Bacteriological of faeces; b) Faeces microscopy; c) Reaction of latex agglutination; d) Rectoromanoscopy; 96. What methods of specific diagnostics of gastrointestinal form of salmonellosis: a) Bacteriological of faeces; b) Faeces microscopy; c) Reaction of latex agglutination;

109

d) Rectoromanoscopy; 97. What methods of specific diagnostics of intestinal lambliosis: a) Bacteriological of faeces; b) Faeces microscopy; c) Reaction of latex agglutination; d) Rectoromanoscopy; 98. What changes in liquor are typical for meningococcal meningitis on microscopy: a) Purulent, muddy; b) Transparent or twinkling; c) hemorrhagical; 99. What changes in liquor are typical for meningeal form of tick encephalitis on microscopy: a) Purulent, muddy; b) Transparent or twinkling; c) hemorrhagical; 100. What blood cells count is typical for meningococcaemia: a) Neutrophil hyperleukocytosis with left shift, high ESR; b) Moderate neutrophil leucocytosis, ESR is moderately increased; c) Leucopenia, lymphomonocytosis, sometimes ESR increase; d) Leucopenia, relative lymphocytosis, increase of plasmatic cells number, ESR increase. 101. What blood sell count is typical for influenza: a) Neutrophil hyperleukocytosis with left shift, high ESR; b) Moderate neutrophil leucocytosis, ESR is moderately increased; c) Leucopenia, lymphomonocytosis, sometimes ESR increase; d) Leucopenia, relative lymphocytosis, increase of plasmatic cells number, ESR increase. 102. What blood sell count is typical for tick borne encephalitis: a) Neutrophil hyperleukocytosis with left shift, high ESR; b) Moderate neutrophil leucocytosis, ESR is moderately increased; c) Leucopenia, lymphomonocytosis, sometimes ESR increase; d) Leucopenia, relative lymphocytosis, increase of plasmatic cells number, ESR increase. 103. Choose methods of specific diagnostics of louse borne typhus, Brills disease: a) RCF, RIA with rickettsia Provazeky; b) RCF with rickettsia Burneti; c) RCF, RIA with rickettsia Sybirica; d) IEA with borrelia Burgdorferi. 104. Choose methods of specific diagnostics of NorthAsian rickettsiosis: a) RCF, RIA with rickettsia Provazeky; b) RCF with rickettsia Burneti; c) RCF, RIA with rickettsia Sybirica; d) IEA with borrelia Burgdorferi. 105. Choose methods of specific diagnostics of Q fever:

a) RCF, RIA with rickettsia Provazeky; b) RCF with rickettsia Burneti; c) RCF, RIA with rickettsia Sybirica; d) IEA with borrelia Burgdorferi. 106. Choose the analysis of CSF, typical for purulent meningitis: a) Colorless, transparent, cytosis - 0,003 109 /l., lymphocytes 100 %, protein 0,33 /. Sediment reaction is negative. Pressure is 250 mm of water; b) Colorless, transparent, cytosis 0,2 109 /l, lymphocytes 80 %, protein 1,0 g/l, sediment reaction positive, pressure is 300 mm of a water; c) Colorless, transparent, cytosis 0,01 109 /l, lymphocytes 85 %, protein 0,3 g/l, sediment reaction negative, pressure is 300 mm of a water; d) Muddy, white yellow color, cytosis 15,0 109 /l, neutrophils 100 %, protein 6,6 g/l, sediment reaction are sharply positive, pressure of 350 mm of a water; 107. Choose the analysis of CSF, typical for mentigismus: a) Colorless, transparent, cytosis - 0,003 109 /l., lymphocytes 100 %, protein 0,33 /. Sediment reaction is negative. Pressure is 250 mm of water; b) Colorless, transparent, cytosis 0,2 109 /l, lymphocytes 80 %, protein 1,0 g/l, sediment reaction positive, pressure is 300 mm of a water; c) Colorless, transparent, cytosis 0,01 109 /l, lymphocytes 85 %, protein 0,3 g/l, sediment reaction negative, pressure is 300 mm of a water; d) Muddy, white yellow color, cytosis 15,0 109 /l, neutrophils 100 %, protein 6,6 g/l, sediment reaction are sharply positive, pressure of 350 mm of a water; 108. Choose the analysis of CSF, typical for serous meningitis: a) Colorless, transparent, cytosis - 0,003 109 /l., lymphocytes 100 %, protein 0,33 /. Sediment reaction is negative. Pressure is 250 mm of water; b) Colorless, transparent, cytosis 0,2 109 /l, lymphocytes 80 %, protein 1,0 g/l, sediment reaction positive, pressure is 300 mm of a water; c) Colorless, transparent, cytosis 0,01 109 /l, lymphocytes 85 %, protein 0,3 g/l, sediment reaction negative, pressure is 300 mm of a water; d) Muddy, white yellow color, cytosis 15,0 109 /l, neutrophils 100 %, protein 6,6 g/l, sediment reaction are sharply positive, pressure of 350 mm of a water;

110

It is another pare of shoes.

URGENT STATES IN INFECTIOUS PATHOLOGY 109. For what kind of shock increasing of relative density of blood plasma is typical? f) infectious toxic; g) *hypovolemic; h) anaphylactic; i) cardiogenic; j) traumatic. 110. What preferable way of drugs administration in case of infectious - toxic shock development? f) per orally; g) subcutaneously; h) intramuscular; i) *intravenously; j) intraartreioly. 111. What one of the following solutions is most effective for initial treatment of infectious toxic shock? a) Trisol; b) 4% natrium hydrocarbonate solution; c) Polyglucine; d) Innunoglobuline; e) Regydrone. 112. Choose the clinical criteria of sopor (2): f) Contact with patient is absent; g) *Contact with patient is possible only on level of verbal-mimic reactions; h) *Directed reactions on pain irritations; i) Reactions of pupils on light are absent; j) Patients answers on questions are monosyllabic and not always adequate. 113. Mention the most typical clinical signs of brain edema: f) Hemorrhagical rash on the mucous membrane of soft palate; g) Cardiac arrhythmia; h) Psycho-motor excitation; i) Tonoclonic cramps; j) Tonic-and clonic cramps. 114. What level of systolic arterial pressure can result in blocking of renal filtration? f) 105/70 mm Hg; g) 100/60 mm Hg; h) 70/50 mm Hg; i) 95/60 mm Hg; j) 90/50 mm Hg. 115. What is drug of choise for treatment of anaphylactic shock? f) Rhiboxine; g) Infusion of 5% glucose solution; h) *Adrenaline; i) Glucocorticoids; j) Cordiamine. 116. Mention appropriate remedies for treatment of hypovolemic shock:

f) Ringer-Locks solution; g) Isotonic solution of sodium chloride; h) Trisol solution; i) 10% albumin solution; j) Rheopolyglucine. 117. Which clinical signs are typical for pulmonary edema? f) Crepitating rails on auscultation of lungs. g) Severe dyspnea while the patient is at rest in the bed, forced state of the patient in the bed h) Diffuse moist rails of different calibers on auscultation of the lung i) Bradycardia j) Noise of pleura friction 118. Mention the clinical signs of acute liver failure. f) Disorders of color sensation; g) Leucopoenia in CBC; h) Progressive decreasing of liver size; i) Disorders of consciousness; j) Low level of prothrombine index. 119. Choose the drug for treatment of comatose form of malaria (due to falciparum) caused with resistant to chloroquine plasmodia. f) Primaquine; g) Ribavirin; h) Zydovudin; i) Quinidine gluconate; j) Metronidazole. 120. What correlation of colloid and saline solutions for intensive treatment of septic shock is optimal? f) 1:1; g) 1:2; h) 2:1; i) 1:3; j) 3:1. 121. What daily dosage of dextrans is safe for treatment of infectious toxic shock? f) 3 liters; g) 500 ml; h) 10 ml; i) 800 ml; j) 2 liters. 122. Which complications are possible when dextrans are overdosed? f) Necrosis of large muscles; g) Decreasing of acuteness of hearing; h) Brain edema; i) Pulmonary edema; j) Necrosis of epithelium cells of renal tubules. 123. What speed of saline solutions infusion you must keep during the first stage of treatment of patient with hypovolemic shock? f) 1 l/h; g) 3 l/h; h) 100-120 ml/min; i) 60 drops/min; j) 50 ml/min. 124. What drugs should be given for patients with septic shock, refractory to volume replacing treatment and glucocorticoids?

111

To continue infusion of high doses of glucocorticoids (up to 1000-1500 mg); g) To begin intravenous infusion of dofamine; h) Treatment with enterosorbents; i) To begin transfusion of blood; j) To use lymposorbtion or plasmosorbtion. 125. Choose 2 of all laboratory signs typical for acute respiratory failure: f) *Pa v O2 >50 mm Hg; g) Pa v CO2 > 50 mm Hg; h) *Pa v O2 40 mm Hg; i) Pa v CO2 40 mm Hg; j) pH of the blood 7,38. 126. Choose clinical sign that typical for croup of II stage. f) Whistle dry rails over a lungs; g) *Inspiratory stridor; h) Comatose state; i) Discharging of plentiful foamy sputum; j) Diffuse moist rails of different calibers on auscultation of lungs. 127. Indicate the most characteristic clinical signs of acute broncoobstruction (2). f) *Whistle dry rails over the lung; g) Dry barking cough; h) *Tympanic sound on percussion of lungs; i) Discharging of abundant foamy sputum; j) Diffuse moist rails of different calibers on auscultation of lungs. 128. Indicate the most characteristic clinical signs of pulmonary edema (2): f) Whistle dry rails over a lungs; g) Severe pain in the chest on inhale; h) Comatose state; i) *Discharging of plentiful foamy sputum; j) *Diffuse moist rails of different calibers on auscultation of lungs. 129. Choose appropriate clinical and laboratory signs of acute respiratory failure (3): f) *Dyspnea; g) *Pa v O2 40 mm Hg; h) *Diffuse cyanosis; i) Pa v CO2 40 mm Hg j) The work of additional muscles on breathing 130. Which clinical and laboratory signs are typical for acute renal failure (ARF) (2)? f) *Dryness of mucous membranes; g) *Level of creatinine in the blood 0,20 mkmol/l; h) Level of urea in the blood 0,08 mkmol/l; i) High temperature; j) Pain in abdomen. 131. Oliguria it is state when volume of diuresis is: f) up to 1000 ml; g) up to 700 ml; h) *up to 500 ml; i) up to 300 ml; j) up to 100 ml. 132. Which volume of diuresis we can classify as anuria?

f)

f) 500 ml; g) *300 ml; h) 200 ml; i) 100 ml; j) 50 ml. 133. These are all clinical and laboratory signs of acute renal failure beside of: f) vomiting; g) diarrhea; h) dryness of mucous membranes; i) *urea of the blood 0,08 mmol/l; j) creatinine of the blood 0,27 mmol/l.; 134. Which infectious diseases can result in acute renal failure (3)? f) *Leptospirosis; g) Brucellosis; h) *Haemorrhagical fevers; i) Salmonellosis; j) *Cholera. 135.Indications for hemodialysis in patients with acute renal failure are all following beside of (2): f) *Potassium of plasma 5,7 mmol/l; g) Urea of plasma 0.24 mkmol/l; h) *Creatinine of plasma 0,5 mkmol/l; i) Creatinine of plasma 0,3 mkmol/l; j) *Longevity of anuria stage more than 10 days. 136.Which drugs are effective for treatment of acute respiratory failure (2)? f) Infusion of 10% solution of glucose; g) Infusion of 5% solution of glucose; h) *Glucocorticoids; i) Prozerinum; j) *Artificial pulmonary ventilation. 137. Which drugs can suppress formation of foam in patients with pulmonary edema (2)? f) *Inhaling of 78% ethanol alcohol; g) Pentaminum i/m; h) Inhaling oxygen mixed with air; i) Inhaling of ether; j) *Inhaling of antiphomsilanum. 138. What clinical signs are typical for diphtheric croup: e) Sudden beginning with fast development of laryngeal stridor and wavy course; f) Sudden development of laryngeal stridor during eating or game, absence of fever and catharal symptoms; g) Gradual development of laryngeal stridor, frequently normal body temperature, multiple of dry rales; h) Fever, harsh voice, consequent development of barking cough, dysphonia, lengthened sonorous breath, inspirator dispose, cyanosis of lips, tachicardia. 139. What clinical signs are typical for aspiration of foreign body: e) Sudden beginning with fast development of laryngeal stridor and wavy course;

112

Sudden development of laryngeal stridor during eating or game, absence of fever and catharal symptoms; g) Gradual development of laryngeal stridor, frequently normal body temperature, multiple of dry rales; h) Fever, harsh voice, consequent development of barking cough, dysphonia, lengthened sonorous breath, inspirator dispose, cyanosis of lips, tachicardia. 140. What clinical signs are typical for croup as complication of influenza and ARVI: e) Sudden beginning with fast development of laryngeal stridor and wavy course; f) Sudden development of laryngeal stridor during eating or game, absence of fever and catharal symptoms; g) Gradual development of laryngeal stridor, frequently normal body temperature, multiple of dry rales; h) Fever, harsh voice, consequent development of barking cough, dysphonia, lengthened sonorous breath, inspirator dispose, cyanosis of lips, tachicardia. 141. Chouse main pathogenetic mechanism of toxic shock: e) dehydratation; f) Primary decrease of cardiac input; g) Pathological increase of vascular capacity; h) Severe deficiency of steroid hormones; 142. Chouse main pathogenetic mechanism of cardiogenic shock: e) dehydratation; f) Primary decrease of cardiac input; g) Pathological increase of vascular capacity; h) Severe deficiency of steroid hormones; 143. Chouse main pathogenetic mechanism of hypovolemic shock: a) dehydratation; e) Primary decrease of cardiac input; f) Pathological increase of vascular capacity; g) Severe deficiency of steroid hormones; 144. Which of basic logical ( Kochs postulates) proof that disease is caused by a microorganism: a) Microorganism must be present in every case of disease, but absent in healthy individuals; b)suspected microorganism must be isolated from affected person and grown in culture; c)Same disease must resalt when isolated microorganism is introduced into healthy host; d)Same microorganism must be isolated again from second affected. e)* All mentioned

f)

ANSWERS FOR TESTS FOR 5th YEAR STUDENS N A N A N n n s s . . 1 A 6 B 1 4 D 2 7 2 C 6 A 1 5 C 2 8 3 A 6 A 1 6 2 9 4 C 6 B 1 7 3 0 5 E 6 B 1 8 3 1 6 B 6 1 9 3 2 7 C 7 C 1 0 3 3 8 B 7 A 1 1 3 4 9 C 7 D 1 2 3 5 1 7 A 1 0 3 3 6 1 B 7 D 1 1 D 4 3 7 1 E 7 C 1 2 5 3 8 1 B 7 B 1 3 6 3 9 1 E 7 A 1 4 7 4 0 1 B 7 B 1 5 E 8 4 1 1 C 7 A 1 6 9 4 2 1 B 8 E 1 7 C 0 4 3 A N A N n n s s . . D 1 A 2 9 5 0 3 B 1 D 2 9 5 1 4 D 1 C 2 9 5 2 5 B 1 D 2 9 5 3 6 E 1 D 2 9 5 4 7 C 1 B 2 9 5 5 8 B 1 C 2 9 5 6 9 B 1 D 2 9 6 7 0 B 1 C 2 9 6 8 1 C 1 D 2 9 6 9 2 A 2 D 2 0 6 0 3 B 2 B 2 0 6 1 4 A 2 C 2 0 6 2 5 C 2 D 2 0 6 3 6 B 2 B 2 0 6 4 7 A 2 C 2 0 6 5 8 A 2 B 2 0 6 6 9 A N A N n n s s . . A 3 E 3 1 7 6 9 C 3 E 3 1 8 7 0 B 3 B 3 1 C 8 8 1 D 3 A 3 1 B 8 9 D 2 B 3 C 3 2 8 0 3 B 3 A 3 2 8 1 4 C 3 B 3 2 8 2 5 A 3 C 3 2 8 3 6 C 3 A 3 2 8 4 7 B 3 D 3 2 8 5 8 C 3 A 3 2 8 6 9 C 3 B 3 2 9 7 0 A 3 B 3 2 9 8 1 A 3 D 3 2 9 9 2 A 3 B 3 3 9 0 3 B 3 C 3 3 9 1 4 B 3 A 3 3 9 2 5 A N A N n n s s . . B 4 A 5 4 D 0 2 5 A 4 C 5 4 D 0 3 6 C 4 A 5 4 0 4 7 A 4 D 5 4 0 5 8 B 4 B 5 4 0 6 9 D 4 C 5 4 1 7 0 B 4 B 5 4 F 1 8 1 B 4 B 5 4 1 9 2 C 4 C 5 5 1 0 3 B 4 A 5 5 E 1 1 4 B 4 D 5 5 1 2 5 A 4 B 5 5 1 3 6 E 4 A 5 5 1 4 7 A 4 B 5 5 1 5 8 A 4 D 5 5 1 6 9 B 4 E 5 5 2 7 0 B 4 D 5 5 2 8 1 A n s . D

113

1 A 8 B 1 8 D 1 4 4 1 B 8 D 1 9 2 4 5 2 C 8 A 1 0 3 4 6 2 B 8 B 1 1 4 E 4 7 2 A 8 B 1 2 5 4 8 2 C 8 B 1 3 6 4 9 2 C 8 1 4 7 5 0 2 A 8 1 5 8 5 1 2 C 8 D 1 6 9 5 2 2 B 9 B 1 7 0 5 3 2 D 9 E 1 8 1 5 4 2 A 9 D 1 9 2 5 5 3 9 C 1 0 3 5 6 3 B 9 A 1 1 4 5 7 3 D 9 C 1 2 5 5 8 3 C 9 A 1 3 6 5 9 3 A 9 D 1 4 7 6 0 3 B 9 B 1 5 8 D 6 1 3 B 9 B 1 6 9 C 6 2 3 A 1 A 1 7 0 B 6

B 2 0 7 E 2 J 0 8 B 2 0 9 A 2 1 0 C 2 1 1 E 2 1 2 B 2 1 3 B 2 1 4 C 2 D 1 5 C 2 1 6 A 2 1 7 B 2 1 8 D 2 1 9 B 2 2 0 C 2 2 1 C 2 2 2 E 2 2 3 C 2 2 4 A 2 2 5 C 2 2

A 2 7 0 C 2 7 1 D 2 7 2 B 2 7 3 A 2 7 4 C 2 7 5 A 2 7 6 B 2 7 7 B 2 7 8 C 2 7 9 C 2 8 0 D 2 8 1 B 2 8 2 D 2 8 3 E 2 8 4 C 2 8 5 B 2 8 6 D 2 8 7 B 2 8 8 C 2 8

C 3 3 3 C 3 3 4 D 3 3 5 B 3 3 6 A 3 3 7 B 3 3 8 E 3 3 9 D 3 4 0 E 3 4 1 A 3 4 2 E 3 4 3 B 3 4 4 C 3 4 5 C 3 4 6 D 3 4 7 A 3 4 8 D 3 4 9 C 3 5 0 B 3 5 1 C 3 5

D 3 9 6 A 3 9 7 B 3 9 8 C 3 9 9 A 4 0 0 A 4 0 1 B 4 0 2 D 4 0 3 B 4 0 4 A 4 0 5 D 4 0 6 B 4 0 7 B 4 0 8 D 4 0 9 C 4 1 0 B 4 1 1 B 4 1 2 C 4 1 3 A 4 1 4 D 4 1

A 4 5 9 B 4 6 0 E 4 6 1 A 4 6 2 A 4 6 3 B 4 6 4 C 4 E 6 5 A 4 6 6 B 4 6 7 A 4 6 8 B 4 6 9 B 4 7 0 C 4 7 1 A 4 7 2 A 4 7 3 A 4 7 4 A 4 7 5 A 4 7 6 B 4 7 7 C 4 7

A 5 2 2 C 5 2 3 B 5 2 4 A 5 2 5 D 5 2 6 C 5 2 7 B 5 2 8 A 5 2 9 C 5 3 0 C 5 3 1 D 5 3 2 B 5 3 3 A 5 3 4 B 5 3 5 B 5 3 6 C 5 3 7 A 5 3 8 B 5 3 9 C 5 4 0 E 5 4

0 3 C 1 8 0 1 3 B 1 9 0 2 4 A 1 0 0 3 4 B 1 1 0 4 4 D 1 2 0 5 4 B 1 3 0 6 4 C 1 4 0 7 4 A 1 5 0 8 4 C 1 6 0 9 4 C 1 7 1 0 4 C 1 8 1 1 4 A 1 9 1 2 5 A 1 0 1 3 5 A 1 1 1 4 5 B 1 2 1 5 5 C 1 3 1 6 5 B 1 4 1 7 5 D 1 5 1 8 5 C 1 6 1 9 5 A 1

3 D 1 6 4 C 1 6 5 E 1 6 6 C 1 6 7 D 1 6 8 B 1 C 6 9 E 1 7 0 C 1 E 7 1 A 1 B 7 2 B 1 D 7 3 A 1 C 7 4 A 1 B 7 5 E 1 7 6 D 1 E 7 7 E 1 7 8 B 1 7 9 C 1 8 0 A 1 8 1 A 1 D 8 2 A 1

6 A 2 2 7 B 2 2 8 D 2 2 9 A 2 3 0 C 2 3 1 B 2 3 2 B 2 3 3 C 2 3 4 A 2 3 5 A 2 3 6 B 2 3 7 B 2 3 8 C 2 3 9 B 2 4 0 C 2 4 1 C 2 4 2 E 2 4 3 A 2 4 4 B 2 4 5 A 2

9 B 2 9 0 C 2 9 1 D 2 9 2 D 2 9 3 B 2 9 4 F 2 9 5 A 2 9 6 D 2 9 7 B 2 9 8 D 2 9 9 A 3 0 0 C 3 0 1 D 3 0 2 C 3 0 3 A 3 0 4 C 3 0 5 D 3 0 6 B 3 0 7 B 3 0 8 A 3

2 C 3 5 3 C 3 5 4 A 3 5 5 C 3 5 6 C 3 5 7 B 3 5 8 A 3 5 9 A 3 6 0 A 3 6 1 B 3 6 2 B 3 6 3 E 3 6 4 E 3 F 6 5 D 3 E 6 6 C 3 6 7 E 3 6 8 A 3 6 9 B 3 7 0 D 3 7 1 C 3

5 C 4 1 6 D 4 1 7 B 4 1 8 A 4 1 9 C 4 2 0 C 4 2 1 B 4 2 2 D 4 2 3 C 4 2 4 A 4 2 5 C 4 2 6 C 4 2 7 B 4 2 8 D 4 2 9 C 4 3 0 B 4 3 1 B 4 3 2 B 4 3 3 C 4 3 4 A 4

8 B 4 7 9 C 4 8 0 D 4 8 1 A 4 8 2 C 4 8 3 B 4 8 4 B 4 D 8 5 C 4 8 6 C 4 8 7 D 4 E 8 8 B 4 8 9 C 4 9 0 B 4 9 1 D 1 9 2 C 4 9 3 B 4 9 4 C 4 9 5 C 4 9 6 C 4 9 7 B 4

1 D 5 4 2 B 5 4 3 B 5 4 4 B 5 E 4 5 C 5 4 6 C 5 4 7 B 5 4 8 B 5 C 4 9 C 5 5 0 D 5 5 1 C 5 5 2 B 5 E 5 3 C 5 5 4 B 5 5 5 C 5 5 6 5 5 7 5 5 8 A 5 5 9 A

114

7 5 A 8 5 C 9 6 B 0 6 D 1 6 A 2 6 C 3

2 0 1 2 1 1 2 2 1 2 3 1 2 4 1 2 5 1 2 6

B 8 E 3 D 1 A 8 4 D 1 A 8 5 B 1 A E 8 6 C 1 B 8 7 C 1 C 8 8 A 1 C 8 9

4 6 2 4 7 2 4 8 2 4 9 2 5 0 2 5 1 2 5 2

0 9 3 1 0 3 1 1 3 1 2 3 1 3 3 1 4 3 1 5

D E

7 2 3 7 3 3 7 4 3 7 5 3 7 6 3 7 7 3 7 8

3 5 4 3 6 4 3 7 4 3 8 4 3 9 4 4 0 4 4 1

A E C E

9 8 4 9 9 5 0 0 5 0 1 5 0 2 5 0 3 5 0 4

ANSWERS FOR CLINICAL CASES AND TESTS FOR 6th YEAR STUDENS

28. Diagnosis: viral hepatitis Differential diagnosis: hepatitis B, C, D, leptospirosis. 29. Diagnisis: tropical malaria, malarial coma. Differential diagnosis: sepsis, typhoid fever, influenza, leptospirosis, hemorrhagic fevers. 30. Diagnosis: influenza. Differential diagnosis: parainfluenza, adenoviral infection, measles, leptospirosis, paratyphoid A. 31. Diagnosis malaria tertiana. Differential diagnosis: influenza, recurrent typhus. 32. Diagnosis: Shigellosis. Differential diagnosis: amebiazis, salmonellosis, echerichiosis, intestinal iersiniosis. 33. Diagnosis: hemorrhagic fever with renal syndrome. Differential diagnosis: leptospirosis. 34. Diagnosis: Leptospirsis. Differential diagnosis: viral hepatitis, hemorrhagic fever with renal syndrome, pseudotuberculosis. 35. Diagnosis: opistorchosis. Differential diagnosis: typhoid and paratyphoid A, B; viral hepatitis C, B, acute cholecistitis. 36. Diagnosis: diphtheria of larynx; Differential diagnosis: parainfluenza. 37. Diagnosis: rabies. Hydrophobic phase.

Differential diagnosis: tetanus, arboviral encephalitis, atropine or strychnine poisoning, delirium tremens. 38. Diagnosis: polyomielitis. Differential diagnosis: botulism, arboviral encephalitis, lymphocytar meningitis; ECHO and Kocksaki diseases. 39. Diagnosis: plague, pulmonary form. Differential diagnosis: pulmonary form of antrax; pneumococcal pneumonia, influenza, complicated with edema of lungs. 40. Diagnosis: Epstine Burr infectious mononucleosis. Differential diagnosis: cytomegaloviral infection, felinosis, acute tonsillitis, localized form of diphtheria, acute HIV infection, leucosis. 41. Diagnosis: enteroviral disease. Differential diagnosis: rotaviral infection, intestinal iersiniosis, echerichiosis, salmonellosis. 42. Diagnosis: tularemia, eye bubonic form. Differential diagnosis: plague, sodocu, felinosis, purulent lymphadenitis. 43. Diagnosis: arboviral tick- borne encephalitis. Differential diagnosis: polyomielitis, rabies, meningococcal meningoencephalitis. 44. Diagnosis: serum disease. Differential diagnosis: measles, iersiniosis. 45. Diagnosis: tularemia, eye bubonic form. Differential diagnosis: plague, sodocu, felinosis, purulent lymphadenitis, infectious mononucleosis. 46. Diagnosis: anaphylactic shock Differential diagnosis: toxic infectious shock, acute poisoning. 47. Diagnosis: acute brucellosis. Differential diagnosis: typhoid fever, tuberculosis, lymphogranulematosis, reumatic fever, iersiniosis. 48. Diagnosis: measles. Differential diagnosis: rubella, enteroviral exantema, serum disease, infectious mononucleosis. 49. Diagnosis: Brills disease (endemic louse born typhus). Differential diagnosis: influenza, arboviral encephalitis, meningococcal infection. 50. Diagnosis: brucellosis. Differential diagnosis: tuberculosis, sepsis, lymphogranulematosis, pseudotuberculosis, reumatic fever, malaria. 51. Diagnosis: plague, bubonic form. Differential diagnosis: tularemia, sodocu, purulent lymphadenitis. 52. Diagnosis: infectious mononucleosis. Differential diagnosis: cytomegaloviral infection, felinosis, acute tonsillitis, localized form of diphtheria, acute HIV infection, leucosis, viral hepatitis A.

115

53. Diagnosis: tropical malaria Differential diagnosis: sepsis, typhoid fever, influenza, leptospirosis, hemorrhagic fevers. 54. : rabies, hydrophobic form. Differential diagnosis: tetanus, tick born encephalitis, delirium tremens.

116

Vous aimerez peut-être aussi